+ All Categories
Home > Documents > Insurance Meeting 6 Cases

Insurance Meeting 6 Cases

Date post: 03-Jun-2018
Category:
Upload: jealousmistress
View: 213 times
Download: 0 times
Share this document with a friend
29
MEETING 6 CASES - INSURANCE G.R. No. L-24899 March 19, 1928 BERNARDO ARGENTE, plaintiff-appellant, vs. WEST COAST LIFE INSURANCE CO., defendant-appelle e. Abad Santos, Camus, Delgado & Recto for appellant. Gibbs & McDonough and Roman Ozaeta for appellee. MALCOLM, J.: This is an action upon a joint life insurance policy for P15,000 issued by the defendant, the West Coast Life Insurance Co., on May 15, 1925, in favor of the plaintiff, Bernardo Argente, and his wife, Vicenta de Ocampo, the latter having died on November 18, 1925. Fraud in obtaining the policy was pleaded by way of special defense. On the issue thus suggested, the court adopted the theory of the defendant, and held the insurance policy null and void, with the result that the complaint was dismissed, with costs. On February 9, 1925, Bernardo Argente signed an application for joint insurance with his wife in the sum of P2,000. The wife, Vicenta de Ocampo, signed a like application for the same policy. Both applications, with the exception of the names and the signatures of the applicants, were written by Jose Geronimo del Rosario, an agent for the West Coast Life Insurance Co. But all the information contained in the applications was furnished the agent by Bernardo Argente. Pursuant to his application, Bernardo Argente was examined by Dr. Cesareo Sta. Ana, a medical examiner for the West Coast Life Insurance Co., on February 10, 1925, in the office of the Customs House. The result of such examination was recorded in the Medical Examiner's Report, and with the exception of the signature of Bernardo Argente, was in the hand-writing of Doctor Sta. Ana. But the information or answers to the questions contained on the face of the Medical Examiner's Report were furnished the doctor by the applicant, Bernardo Argente. Pursuant to her application, Vicenta de Ocampo, wife of the plaintiff, was examined by Dr. Cesareo Sta. Ana on February 10, 1925, at her residence in Manila. The result of the medical examinatio n, including among other things, the answers given by Vicenta de Ocampo to the questions propounded to her by the physician, appears in the Medical Examiner's Report. On May 9, 1925, Bernardo Argente and his wife submitted to the West Coast Life Insurance Co. an amended application for insurance, increasing the amount thereof to P15,000, and asked that the policy be dated May 15, 1925. The amended application was accompanied by the documents entitled "Short Form Medical Report." In both of these documents appear certain questions and answers.  A temporary policy for P15,00 0 was issued to Bernardo Arge nte and his wife as of May 15, but it was not delivered to Bernardo Argente until July 2, 1925, when the first quarterly premium on the policy was paid. In view of the fact that more than thirty days had elapsed since the applicants were examined by the company's physician, each of them was required to file a certificate of health before the policy was delivered to them. On November 18, 1925, Vicenta de Ocampo died of cerebral apoplexy. Thereafter Bernardo Argente presented a claim in due form to the West Coast Life Insurance Co. for the payment of the sum of P15,000 the amount of the  joint life Insurance policy. Following investigation cond ucted by the Manager of the Manila office of the insurance company , it was apparently disclosed that the answers given by the insured in their medical examinations with regard to their health and previous illness and medical attendance were untrue. For that reason, the West Coast Life Insurance Co. refused to pay the claim of Bernardo Argente, and on May 25, 1926, wrote him to the effect that the claim was rejected because the insurance was obtained through fraud and misrepresentation. It is admitted that it appears in the Medical Examiner's Report that Bernardo  Argente, in response to the question asked by the medical examiner, "Have you ever consulted a physician for, or have you ever suffered from any ailment or disease of, the brain or nervous system?" answered "No." To the question, "Have you consulted a physician for any ailment or disease not included in your above answer," answered "Yes. Nature of Ailment, Disease or Injury. Scabies, Number of attacks 1, Date 1911. Duration 1 month, Severity Fair, results and, if within five years, name and address of every physician consulted. Dr. P. Guazon. Cured. Dr. Guazon is dead now." And to the question, "What physician or physicians, if any, not named above, have you examiner, "How frequently, if at all, and in what quantity do you use beer, wine, spirits or other intoxicants?" answered "Beer only in small quantities occasionally." To the question, "Have you ever consulted a physician for or have you ever suffered from any ailment or disease of the brain or nervous system?" answered "No." To the question, "What physician or physicians, if any, not named above, have you consulted or been treated by, within the last five years and for what illness or ailment? (If none, so state)" answered "None." And to the question, "Are you in good health as far as you know and believe?" answered "Yes." It is, however, not disputed that Vicenta de Ocampo was taken by a patrolman, at the request of her husband, Bernardo  Argente, on May 19, 1924, to t he Meisic police station, and from there was transferred to the San Lazaro Hospital. In San Lazaro Hospital, her case was diagnosed by the admitting physician as "alcoholism," but later Doctor Domingo made a diagnosis of probable "manic-depressive psychosis," and still, later in Mary Chiles Hospital, made a final diagnosis of "phycho-neur osis." The plaintiff, Bernardo Argente, while readily conceding most of the facts herein narrated, yet alleges that both he and his wife revealed to the company's physician. Doctor Sta. Ana, all the facts concerning the previous illnesses and medical attendan ce, but that Doctor Sta. Ana, presumably acting in collusion, with the insurance agent, Jose Geronimo del Rosario, failed to record them in the medical reports. The evidence on these points consists of the testimony of the plaintiff and his subordinate clerk, Apolonio Espiritu, on the one hand, and of the testimony of Doctor Sta. Ana and Jose Geronimo del Rosario on the other. On the question of fact thus raised, the trial judge found with the insurance company. In so doing, we believe that His Honor gave proper inclination to the weight of the proof. There appears no motive whatever on the part of Doctor Sta. Ana to falsify the Medical Examiner's Reports and thereby not only jeopardize his career as a physician, but also gravely implicate himself criminally. What has heretofore been stated in this decision is gleaned to a great extent the carefully prepared decision of the trial judge, the Honorable George R. Harvey. The court found from the evidence that the representations made by Bernardo Argente and his wife in their applications to the defendant for life insurance were false with respect to their estate of health during the period of five years preceding the date of such applications, and that they knew the representations made by them in their applications were false. The court further found from the evidence that the answers given by Bernardo Argente and his wife at the time of the medical examination by Doctor Sta. Ana were false with respect to the condition of their health at that time and for a period of several years prior thereto. Based on these findings which must here be accepted since the stenographic transcript is incomplete, the question arises as to the estate of the law in relation thereto. One ground for the rescission of a contract of insurance under the Insurance  Act is "a concealme nt," which in section 25 is defined as "A neglect to communicate that which a party knows and ought to communicate." Appellant argues that the alleged concealment was immaterial and insufficient to avoid the policy. We cannot agree. In an action on a life insurance policy where the evidence conclusively shows that the answers to questions concerning diseases were untrue, the truth of falsity of the answers become the determining factor. In the policy was procured by fraudulent representations, the contract of insurance apparently set forth therein was never legally existent. It can fairly be assumed that had the true facts been disclosed by the assured, the insurance would never have been granted. In Joyce, The Law of Insurance, second edition, volume 3, Chapter LV, is found the following: Concea lment exists where the assured has knowledge of a fact material to the risk, and honesty, good faith, and fair dealing requires that he should communicate it t o the assured, but he designated and intentionally with holds the same.  Another rule is that if the assured undertakes to state all the circumstances affecting the risk, a full and fair statement of all is required. It is also held that the concealment must, in the absence of inquiries, be not only material, but fraudulent, or the fact must have been intentionally withheld; so it is held under English law that if no inquiries are made and no fraud or design to conceal enters into the concealment the contract is not avoided. And it is determined that even though silence may constitute misrepresentatio n or concealment it is not itself necessarily so as it is a question of fact. Nor is there a concealment justifying a forfeiture where the fact of insanity is not disclosed no questions being asked conce rning the same. . . . But it would seem that if a material fact is actually known to the assured, its concealm ent must of itself necessa rily be a f raud, and if the fact is one which
Transcript

8/12/2019 Insurance Meeting 6 Cases

http://slidepdf.com/reader/full/insurance-meeting-6-cases 1/28

MEETING 6 CASES - INSURANCE

1

G.R. No. L-24899 March 19, 1928

BERNARDO ARGENTE, plaintiff-appellant,vs.WEST COAST LIFE INSURANCE CO., defendant-appellee.

Abad Santos, Camus, Delgado & Recto for appellant.Gibbs & McDonough and Roman Ozaeta for appellee.

MALCOLM, J.:

This is an action upon a joint life insurance policy for P15,000 issued by thedefendant, the West Coast Life Insurance Co., on May 15, 1925, in favor of theplaintiff, Bernardo Argente, and his wife, Vicenta de Ocampo, the latter havingdied on November 18, 1925. Fraud in obtaining the policy was pleaded by wayof special defense. On the issue thus suggested, the court adopted the theoryof the defendant, and held the insurance policy null and void, with the resultthat the complaint was dismissed, with costs.

On February 9, 1925, Bernardo Argente signed an application for jointinsurance with his wife in the sum of P2,000. The wife, Vicenta de Ocampo,signed a like application for the same policy. Both applications, with theexception of the names and the signatures of the applicants, were written byJose Geronimo del Rosario, an agent for the West Coast Life Insurance Co.But all the information contained in the applications was furnished the agent byBernardo Argente.

Pursuant to his application, Bernardo Argente was examined by Dr. CesareoSta. Ana, a medical examiner for the West Coast Life Insurance Co., onFebruary 10, 1925, in the office of the Customs House. The result of suchexamination was recorded in the Medical Examiner's Report, and with theexception of the signature of Bernardo Argente, was in the hand-writing ofDoctor Sta. Ana. But the information or answers to the questions contained onthe face of the Medical Examiner's Report were furnished the doctor by theapplicant, Bernardo Argente.

Pursuant to her application, Vicenta de Ocampo, wife of the plaintiff, was

examined by Dr. Cesareo Sta. Ana on February 10, 1925, at her residence inManila. The result of the medical examination, including among other things,the answers given by Vicenta de Ocampo to the questions propounded to herby the physician, appears in the Medical Examiner's Report.

On May 9, 1925, Bernardo Argente and his wife submitted to the West CoastLife Insurance Co. an amended application for insurance, increasing theamount thereof to P15,000, and asked that the policy be dated May 15, 1925.The amended application was accompanied by the documents entitled "ShortForm Medical Report." In both of these documents appear certain questionsand answers.

A temporary policy for P15,000 was issued to Bernardo Argente and his wifeas of May 15, but it was not delivered to Bernardo Argente until July 2, 1925,when the first quarterly premium on the policy was paid. In view of the fact that

more than thirty days had elapsed since the applicants were examined by thecompany's physician, each of them was required to file a certificate of healthbefore the policy was delivered to them.

On November 18, 1925, Vicenta de Ocampo died of cerebral apoplexy.Thereafter Bernardo Argente presented a claim in due form to the West CoastLife Insurance Co. for the payment of the sum of P15,000 the amount of the joint life Insurance policy. Following investigation conducted by the Managerof the Manila office of the insurance company, it was apparently disclosed thatthe answers given by the insured in their medical examinations with regard totheir health and previous illness and medical attendance were untrue. For thatreason, the West Coast Life Insurance Co. refused to pay the claim ofBernardo Argente, and on May 25, 1926, wrote him to the effect that the claimwas rejected because the insurance was obtained through fraud andmisrepresentation.

It is admitted that it appears in the Medical Examiner's Report that Bernardo Argente, in response to the question asked by the medical examiner, "Haveyou ever consulted a physician for, or have you ever suffered from any ailmentor disease of, the brain or nervous system?" answered "No." To the question,"Have you consulted a physician for any ailment or disease not included inyour above answer," answered "Yes. Nature of Ailment, Disease or Injury.Scabies, Number of attacks 1, Date 1911. Duration 1 month, Severity Fair,results and, if within five years, name and address of every physicianconsulted. Dr. P. Guazon. Cured. Dr. Guazon is dead now." And to thequestion, "What physician or physicians, if any, not named above, have youconsulted or been treated by, within the last five years and for what illness orailment? (If none, so state)" answered "No." It is, however, not disputed that onJanuary 10, 11, and 13, 1923, Bernardo Argente was confined in thePhilippine General Hospital where he was treated by Dr. Agerico B. M. Sisonfor cerebral congestion and Bell's Palsy.

It is further admitted that it appears in the Medical Examiner's Report thatVicenta de Ocampo, in response to the question asked by the medical

examiner, "How frequently, if at all, and in what quantity do you use beer, wine,spirits or other intoxicants?" answered "Beer only in small quantitiesoccasionally." To the question, "Have you ever consulted a physician for orhave you ever suffered from any ailment or disease of the brain or nervoussystem?" answered "No." To the question, "What physician or physicians, ifany, not named above, have you consulted or been treated by, within the lastfive years and for what illness or ailment? (If none, so state)" answered"None." And to the question, "Are you in good health as far as you know and

believe?" answered "Yes." It is, however, not disputed that Vicenta deOcampo was taken by a patrolman, at the request of her husband, Bernardo Argente, on May 19, 1924, to the Meisic police station, and from there wastransferred to the San Lazaro Hospital. In San Lazaro Hospital, her case wasdiagnosed by the admitting physician as "alcoholism," but later DoctorDomingo made a diagnosis of probable "manic-depressive psychosis," andstill, later in Mary Chiles Hospital, made a final diagnosis of "phycho-neurosis."

The plaintiff, Bernardo Argente, while readily conceding most of the factsherein narrated, yet alleges that both he and his wife revealed to thecompany's physician. Doctor Sta. Ana, all the facts concerning the previousillnesses and medical attendance, but that Doctor Sta. Ana, presumably actingin collusion, with the insurance agent, Jose Geronimo del Rosario, failed torecord them in the medical reports. The evidence on these points consists ofthe testimony of the plaintiff and his subordinate clerk, Apolonio Espiritu, on

the one hand, and of the testimony of Doctor Sta. Ana and Jose Geronimo delRosario on the other. On the question of fact thus raised, the trial judge foundwith the insurance company. In so doing, we believe that His Honor gaveproper inclination to the weight of the proof. There appears no motivewhatever on the part of Doctor Sta. Ana to falsify the Medical Examiner'sReports and thereby not only jeopardize his career as a physician, but alsogravely implicate himself criminally.

What has heretofore been stated in this decision is gleaned to a great extentthe carefully prepared decision of the trial judge, the Honorable George R.Harvey. The court found from the evidence that the representations made byBernardo Argente and his wife in their applications to the defendant for lifeinsurance were false with respect to their estate of health during the period offive years preceding the date of such applications, and that they knew therepresentations made by them in their applications were false. The courtfurther found from the evidence that the answers given by Bernardo Argenteand his wife at the time of the medical examination by Doctor Sta. Ana werefalse with respect to the condition of their health at that time and for a period ofseveral years prior thereto. Based on these findings which must here beaccepted since the stenographic transcript is incomplete, the question arisesas to the estate of the law in relation thereto.

One ground for the rescission of a contract of insurance under the Insurance Act is "a concealment," which in section 25 is defined as "A neglect tocommunicate that which a party knows and ought to communicate." Appellantargues that the alleged concealment was immaterial and insufficient to avoidthe policy. We cannot agree. In an action on a life insurance policy where theevidence conclusively shows that the answers to questions concerningdiseases were untrue, the truth of falsity of the answers become thedetermining factor. In the policy was procured by fraudulent representations,the contract of insurance apparently set forth therein was never legallyexistent. It can fairly be assumed that had the true facts been disclosed by theassured, the insurance would never have been granted.

In Joyce, The Law of Insurance, second edition, volume 3, Chapter LV, isfound the following:

Concealment exists where the assured has knowledge of a fact material to therisk, and honesty, good faith, and fair dealing requires that he shouldcommunicate it to the assured, but he designated and intentionally with holdsthe same.

Another rule is that if the assured undertakes to state all the circumstancesaffecting the risk, a full and fair statement of all is required.

It is also held that the concealment must, in the absence of inquiries, be notonly material, but fraudulent, or the fact must have been intentionally withheld;so it is held under English law that if no inquiries are made and no fraud ordesign to conceal enters into the concealment the contract is not avoided. Andit is determined that even though silence may constitute misrepresentation orconcealment it is not itself necessarily so as it is a question of fact. Nor is therea concealment justifying a forfeiture where the fact of insanity is not disclosedno questions being asked concerning the same. . . .

But it would seem that if a material fact is actually known to the assured, itsconcealment must of itself necessarily be a fraud, and if the fact is one whichthe assured ought to know, or is presumed to know, the presumption ofknowledge ought to place the assured in the same position as in the formercase with relation to material facts; and if the jury in such cases find the fact

material, and one tending to increase the risk, it is difficult to see how theinference of a fraudulent intent or intentional concealment can be avoided. And it is declared that if a material fact concealed by assured it is equivalent toa false representation that it does not exist and that the essentials are the truth

8/12/2019 Insurance Meeting 6 Cases

http://slidepdf.com/reader/full/insurance-meeting-6-cases 2/28

MEETING 6 CASES - INSURANCE

2

of the representations whether they were intended to mislead and did insureraccept them as true and act upon them to his prejudice. So it is decided thatunder a stipulation voiding the policy for concealment or misrepresentation ofany material fact or if his interest is not truly stated or is either than the soleand unconditional ownership the facts are unimportant that insured did notintend to deceive or withhold information as to encumbrances even though noquestions were asked. And if insured while being examined for life insuranceand knowing that she had heart disease, falsely stated that she was in good

health, and though she could not read the application, it was explained to herand the questions asked through an interpreter, and the application like thepolicy contained and provision that no liability should be incurred unless thepolicy was delivered while the insured was in good health, the court properlydirected a verdict for the insurer, though a witness who was present at theexamination testified that the insured was not asked whether she had heartdisease.

x x x x x x x x x

The basis of the rule vitiating the contract in case of concealment is that itmisleads or deceives the insurer into accepting the risk, or accepting it at therate of premium agreed upon. The insurer, relying upon the belief that theassured will disclose every material within his actual or presumed knowledge,is misled into a belief that the circumstance withheld does not exist, and he is

thereby induced to estimate the risk upon a false basis that it does not exist.The principal question, therefore, must be, Was the assurer misled ordeceived into entering a contract obligation or in fixing the premium ofinsurance by a withholding of material information of facts within the assured'sknowledge or presumed knowledge?

It therefore follows that the assurer in assuming a risk is entitled to know everymaterial fact of which the assured has exclusive or peculiar knowledge, as wellas all material facts which directly tend to increase the hazard or risk which areknown by the assured, or which ought to be or are presumed to be known byhim. And a concealment of such facts vitiates the policy. "It does not seem tobe necessary . . . that the . . . suppression of the truth should have beenwillful." If it were but an inadvertent omission, yet if it were material to the riskand such as the plaintiff should have known to be so, it would render the policyvoid. But it is held that if untrue or false answers are given in response toinquiries and they relate to material facts the policy is avoided without regardto the knowledge or fraud of assured, although under the statute statementsare representations which must be fraudulent to avoid the policy. So undercertain codes the important inquiries are whether the concealment was willfuland related to a matter material to the risk.

x x x x x x x x x

If the assured has exclusive knowledge of material facts, he should fully andfairly disclose the same, whether he believes them material or not. Butnotwithstanding this general rule it will not infrequently happen, especially inlife risks, that the assured may have a knowledge actual or presumed ofmaterial facts, and yet entertain an honest belief that they are not material. . . .The determination of the point whether there has or has not been a materialconcealment must rest largely in all cases upon the form of the questionspropounded and the exact terms of the contract. Thus, where in addition tospecifically named diseases the insured was asked whether he had anysickness within ten years, to which he answered "No," and it was proven thatwithin that period he had a slight of pharyngitis, it was held a question properlyfor the jury whether such an inflammation of the throat was a "sickness" withinthe intent of the inquiry, and the court remarked on the appeal decision that if itcould be held as a matter of law that the policy was thereby avoided, then itwas a mere device on the part of insurance companies to obtain moneywithout rendering themselves liable under the policy. . . .

. . . The question should be left to the jury whether the assured trulyrepresented the state of his health so as not mislead or deceive the insurer;and if he did not deal a good faith with insurer in that matter, that the inquiryshould be made, Did he know the state of his health so as to be able to furnisha proper answer to such questions as are propounded? A Massachusettscase, if construed as it is frequently cited, would be opposed to the aboveconclusion; but, on the contrary, it sustains it, for the reason that symptoms ofconsumption had so far developed themselves within a few months prior toeffecting the insurance as to induce a reasonable belief that the applicant hadthat fatal disease, and we should further construe this case as establishing therule that such a matter cannot rest alone upon the assured's belief irrespectiveof what is a reasonable belief, but that it ought to be judged by the criterionwhether the belief is one fairly warranted by the circumstances. A case inIndiana, however, holds that if the assured has some affection or ailment ofone or more of the organs inquired about so well-defined and marked as tomaterially derange for a time the functions of such organ, as in the case ofBright's disease, the policy will be avoided by a nondisclosure, irrespective ofthe fact whether the assured knew of such ailment or not. . . .

Lastly, appellant contends that even if the insurance company had a right torescind the contract, such right cannot now be enforced in view of theprovisions of section 47 of the Insurance Act providing "Whenever a right torescind a contract of insurance is given to their insurer by provision of this

chapter, such right must be exercised previous to the commencement of anaction on the contract." This section was derived from section 2583 of theCalifornia Civil Code, but in contrast thereto, makes use of the imperative"must" instead of the permissive "may." Nevertheless, there are two answersto the problem as propounded. The first is that the California law as construedby the code examiners, at whose recommendation it was adopted, concededthat "A failure to exercise the right (of rescission), cannot, of course, prejudiceany defense to the action which the concealment may furnish." (Codes of

California annotated; Tan Chay Heng vs. West Coast Life Insurance Company[1927], p. 80, ante.) The second answer is that the insurance company morethan one month previous to the commencement of the present action wrotethe plaintiff and informed him that the insurance contract was void because ithad been procured through fraudulent representations, and offered to refundto the plaintiff the premium which the latter had paid upon the return of thepolicy for cancellation. As held in California as to a fire insurance policy, whereany of the material representations are false, the insurer's tender of thepremium and notice that the policy is canceled, before the commencement ofsuit thereon, operate to rescind the contract of insurance. (Rankin vs. AmazonInsurance Co. [1891], 89 Cal., 203.)

We are content to rest our judgment on the findings of the trial court, and onthe law governing those facts, with the result that the various assignments oferror are found to be without persuasive merit.

Judgment affirmed, with the costs of this instance against the appellant.

Villamor, Ostrand, Johns, Romualdez and Villa-Real, JJ., concur.

SECOND DIVISION[G.R. No. 113899. October 13, 1999]

GREAT PACIFIC LIFE ASSURANCE CORP., petitioner vs. COURT OFAPPEALS AND MEDARDA V. LEUTERIO, respondents.D E C I S I O NQUISUMBING, J.:

This petition for review, under Rule 45 of the Rules of Court, assails theDecision[1] dated May 17, 1993, of the Court of Appeals and its Resolution[2]dated January 4, 1994 in CA-G.R. CV No. 18341. The appellate courtaffirmed in toto the judgment of the Misamis Oriental Regional Trial Court,Branch 18, in an insurance claim filed by private respondent against GreatPacific Life Assurance Co. The dispositive portion of the trial court‘s decisionreads:

―WHEREFORE, judgment is rendered adjudging the defendant GREATPACIFIC LIFE ASSURANCE CORPORATION as insurer under its Grouppolicy No. G-1907, in relation to Certification B-18558 liable and ordered topay to the DEVELOPMENT BANK OF THE PHILIPPINES as creditor of theinsured Dr. Wilfredo Leuterio, the amount of EIGHTY SIX THOUSAND TWOHUNDRED PESOS (P86,200.00); dismissing the claims for damages,attorney‘s fees and litigation expenses in the complaint and counterclaim, withcosts against the defendant and dismissing the complaint in respect to theplaintiffs, other than the widow-beneficiary, for lack of cause of action.‖[3]

The facts, as found by the Court of Appeals, are as follows:

A contract of group life insurance was executed between petitioner GreatPacific Life Assurance Corporation (hereinafter Grepalife) and DevelopmentBank of the Philippines (hereinafter DBP). Grepalife agreed to insure thelives of eligible housing loan mortgagors of DBP.

On November 11, 1983, Dr. Wilfredo Leuterio, a physician and a housingdebtor of DBP applied for membership in the group life insurance plan. In anapplication form, Dr. Leuterio answered questions concerning his healthcondition as follows:

―7. Have you ever had, or consulted, a physician for a heart condition, highblood pressure, cancer, diabetes, lung, kidney or stomach disorder or anyother physical impairment?

Answer: No. If so give details ___________.

8. Are you now, to the best of your knowledge, in good health?

Answer: [ x ] Yes [ ] No.‖[4]

On November 15, 1983, Grepalife issued Certificate No. B-18558, asinsurance coverage of Dr. Leuterio, to the extent of his DBP mortgageindebtedness amounting to eighty-six thousand, two hundred (P86,200.00)pesos.

On August 6, 1984, Dr. Leuterio died due to ―massive cerebral hemorrhage.‖Consequently, DBP submitted a death claim to Grepalife. Grepalife deniedthe claim alleging that Dr. Leuterio was not physically healthy when he appliedfor an insurance coverage on November 15, 1983. Grepalife insisted that Dr.

8/12/2019 Insurance Meeting 6 Cases

http://slidepdf.com/reader/full/insurance-meeting-6-cases 3/28

MEETING 6 CASES - INSURANCE

3

Leuterio did not disclose he had been suffering from hypertension, whichcaused his death. Allegedly, such non-disclosure constituted concealmentthat justified the denial of the claim.

On October 20, 1986, the widow of the late Dr. Leuterio, respondent MedardaV. Leuterio, filed a complaint with the Regional Trial Court of Misamis Oriental,Branch 18, against Grepalife for ―Specific Performance with Damages.‖[5] During the trial, Dr. Hernando Mejia, who issued the death certificate, was

called to testify. Dr. Mejia‘s findings, based partly from the information givenby the respondent widow, stated that Dr. Leuterio complained of headachespresumably due to high blood pressure. The inference was not conclusivebecause Dr. Leuterio was not autopsied, hence, other causes were not ruledout.

On February 22, 1988, the trial court rendered a decision in favor ofrespondent widow and against Grepalife. On May 17, 1993, the Court of Appeals sustained the trial court‘s decision. Hence, the present petition.Petitioners interposed the following assigned errors:

"1. THE LOWER COURT ERRED IN HOLDING DEFENDANT-APPELLANTLIABLE TO THE DEVELOPMENT BANK OF THE PHILIPPINES (DBP)WHICH IS NOT A PARTY TO THE CASE FOR PAYMENT OF THEPROCEEDS OF A MORTGAGE REDEMPTION INSURANCE ON THE LIFE

OF PLAINTIFF‘S HUSBAND WILFREDO LEUTERIO ONE OF ITS LOANBORROWERS, INSTEAD OF DISMISSING THE CASE AGAINSTDEFENDANT-APPELLANT [Petitioner Grepalife] FOR LACK OF CAUSE OF ACTION.

2. THE LOWER COURT ERRED IN NOT DISMISSING THE CASE FORWANT OF JURISDICTION OVER THE SUBJECT OR NATURE OF THE ACTION AND OVER THE PERSON OF THE DEFENDANT.

3. THE LOWER COURT ERRED IN ORDERINGDEFENDANT-APPELLANT TO PAY TO DBP THE AMOUNT OF P86,200.00IN THE ABSENCE OF ANY EVIDENCE TO SHOW HOW MUCH WAS THE ACTUAL AMOUNT PAYABLE TO DBP IN ACCORDANCE WITH ITSGROUP INSURANCE CONTRACT WITH DEFENDANT-APPELLANT.

4. THE LOWER COURT ERRED IN - HOLDING THAT THERE WAS NOCONCEALMENT OF MATERIAL INFORMATION ON THE PART OFWILFREDO LEUTERIO IN HIS APPLICATION FOR MEMBERSHIP IN THEGROUP LIFE INSURANCE PLAN BETWEEN DEFENDANT-APPELLANTOF THE INSURANCE CLAIM ARISING FROM THE DEATH OF WILFREDOLEUTERIO.‖[6]

Synthesized below are the assigned errors for our resolution:

1. Whether the Court of Appeals erred in holding petitioner liable to DBP asbeneficiary in a group life insurance contract from a complaint filed by thewidow of the decedent/mortgagor?

2. Whether the Court of Appeals erred in not finding that Dr. Leuterioconcealed that he had hypertension, which would vitiate the insurancecontract?

3. Whether the Court of Appeals erred in holding Grepalife liable in the amountof eighty six thousand, two hundred (P86,200.00) pesos without proof of theactual outstanding mortgage payable by the mortgagor to DBP.

Petitioner alleges that the complaint was instituted by the widow of Dr.Leuterio, not the real party in interest, hence the trial court acquired no jurisdiction over the case. It argues that when the Court of Appeals affirmedthe trial court‘s judgment, Grepalife was held liable to pay the proceeds ofinsurance contract in favor of DBP, the indispensable party who was not joinedin the suit.

To resolve the issue, we must consider the insurable interest in mortgagedproperties and the parties to this type of contract. The rationale of a groupinsurance policy of mortgagors, otherwise known as the ―mortgageredemption insurance,‖ is a device for the protection of both the mortgageeand the mortgagor. On the part of the mortgagee, it has to enter into suchform of contract so that in the event of the unexpected demise of themortgagor during the subsistence of the mortgage contract, the proceeds fromsuch insurance will be applied to the payment of the mortgage debt, therebyrelieving the heirs of the mortgagor from paying the obligation.[7] In a similarvein, ample protection is given to the mortgagor under such a concept so thatin the event of death; the mortgage obligation will be extinguished by theapplication of the insurance proceeds to the mortgage indebtedness.[8]Consequently, where the mortgagor pays the insurance premium under thegroup insurance policy, making the loss payable to the mortgagee, theinsurance is on the mortgagor‘s interest, and the mortgagor continues to be aparty to the contract. In this type of policy insurance, the mortgagee is simply

an appointee of the insurance fund, such loss-payable clause does not makethe mortgagee a party to the contract.[9]

Section 8 of the Insurance Code provides:

―Unless the policy provides, where a mortgagor of property effects insurancein his own name providing that the loss shall be payable to the mortgagee, orassigns a policy of insurance to a mortgagee, the insurance is deemed to beupon the interest of the mortgagor, who does not cease to be a party to theoriginal contract, and any act of his, prior to the loss, which would otherwiseavoid the insurance, will have the same effect, although the property is in thehands of the mortgagee, but any act which, under the contract of insurance, is

to be performed by the mortgagor, may be performed by the mortgageetherein named, with the same effect as if it had been performed by themortgagor.‖

The insured private respondent did not cede to the mortgagee all his rights orinterests in the insurance, the policy stating that: ―In the event of the debtor‘sdeath before his indebtedness with the Creditor [DBP] shall have been fullypaid, an amount to pay the outstanding indebtedness shall first be paid to thecreditor and the balance of sum assured, if there is any, shall then be paid tothe beneficiary/ies designated by the debtor.‖[10] When DBP submitted theinsurance claim against petitioner, the latter denied payment thereof,interposing the defense of concealment committed by the insured.Thereafter, DBP collected the debt from the mortgagor and took the necessaryaction of foreclosure on the residential lot of private respondent.[11] InGonzales La O vs. Yek Tong Lin Fire & Marine Ins. Co.[12] we held:

―Insured, being the person with whom the contract was made, is primarily theproper person to bring suit thereon. * * * Subject to some exceptions,insured may thus sue, although the policy is taken wholly or in part for thebenefit of another person named or unnamed, and although it is expresslymade payable to another as his interest may appear or otherwise. * * * Although a policy issued to a mortgagor is taken out for the benefit of themortgagee and is made payable to him, yet the mortgagor may sue thereon inhis own name, especially where the mortgagee‘s interest is less than the fullamount recoverable under the policy, * * *.‘

And in volume 33, page 82, of the same work, we read the following:

‗Insured may be regarded as the real party in interest, although he hasassigned the policy for the purpose of collection, or has assigned as collateralsecurity any judgment he may obtain.‖[13]

And since a policy of insurance upon life or health may pass by transfer, will orsuccession to any person, whether he has an insurable interest or not, andsuch person may recover it whatever the insured might have recovered,[14]the widow of the decedent Dr. Leuterio may file the suit against the insurer,Grepalife.

The second assigned error refers to an alleged concealment that the petitionerinterposed as its defense to annul the insurance contract. Petitionercontends that Dr. Leuterio failed to disclose that he had hypertension, whichmight have caused his death. Concealment exists where the assured hadknowledge of a fact material to the risk, and honesty, good faith, and fairdealing requires that he should communicate it to the assured, but hedesignedly and intentionally withholds the same.[15]

Petitioner merely relied on the testimony of the attending physician, Dr.Hernando Mejia, as supported by the information given by the widow of thedecedent. Grepalife asserts that Dr. Mejia‘s technical diagnosis of the causeof death of Dr. Leuterio was a duly documented hospital record, and that thewidow‘s declaration that her husband had ―possible hypertension severalyears ago‖ should not be considered as hearsay, but as part of res gestae.

On the contrary the medical findings were not conclusive because Dr. Mejiadid not conduct an autopsy on the body of the decedent. As the attendingphysician, Dr. Mejia stated that he had no knowledge of Dr. Leuterio‘s anyprevious hospital confinement.[16] Dr. Leuterio‘s death certificate stated thathypertension was only ―the possible cause of death.‖ The private respondent‘sstatement, as to the medical history of her husband, was due to her unreliablerecollection of events. Hence, the statement of the physician was properlyconsidered by the trial court as hearsay.

The question of whether there was concealment was aptly answered by theappellate court, thus:

―The insured, Dr. Leuterio, had answered in his insurance application that hewas in good health and that he had not consulted a doctor or any of theenumerated ailments, including hypertension; when he died the attendingphysician had certified in the death certificate that the former died of cerebralhemorrhage, probably secondary to hypertension. From this report, theappellant insurance company refused to pay the insurance claim. Appellantalleged that the insured had concealed the fact that he had hypertension.

Contrary to appellant‘s allegations, there was no sufficient proof that the

insured had suffered from hypertension. Aside from the statement of theinsured‘s widow who was not even sure if the medicines taken by Dr. Leuteriowere for hypertension, the appellant had not proven nor produced any witnesswho could attest to Dr. Leuter io‘s medical history...

8/12/2019 Insurance Meeting 6 Cases

http://slidepdf.com/reader/full/insurance-meeting-6-cases 4/28

MEETING 6 CASES - INSURANCE

4

x x x

Appellant insurance company had failed to establish that there wasconcealment made by the insured, hence, it cannot refuse payment of theclaim.‖[17]

The fraudulent intent on the part of the insured must be established to entitle

the insurer to rescind the contract.[18] Misrepresentation as a defense of theinsurer to avoid liability is an affirmative defense and the duty to establish suchdefense by satisfactory and convincing evidence rests upon the insurer.[19] Inthe case at bar, the petitioner failed to clearly and satisfactorily establish itsdefense, and is therefore liable to pay the proceeds of the insurance.

And that brings us to the last point in the review of the case at bar. Petitionerclaims that there was no evidence as to the amount of Dr. Leuterio‘soutstanding indebtedness to DBP at the time of the mortgagor‘s death.Hence, for private respondent‘s failure to establish the same, the action forspecific performance should be dismissed. Petitioner‘s claimis without merit. A life insurance policy is a valued policy.[20] Unless the interest of a personinsured is susceptible of exact pecuniary measurement, the measure ofindemnity under a policy of insurance upon life or health is the sum fixed in thepolicy.[21] The mortgagor paid the premium according to the coverage of his

insurance, which states that:

―The policy states that upon receipt of due proof of the Debtor‘s death duringthe terms of this insurance, a death benefit in the amount of P86,200.00 shallbe paid.

In the event of the debtor‘s death before his indebtedness with the creditorshall have been fully paid, an amount to pay the outstanding indebtednessshall first be paid to the Creditor and the balance of the Sum Assured, if thereis any shall then be paid to the beneficiary/ies designated by the debtor.‖[22](Emphasis omitted)

However, we noted that the Court of Appeals‘ decision was promulgated onMay 17, 1993. In private respondent‘s memorandum, she states that DBPforeclosed in 1995 their residential lot, in satisfaction of mortgagor‘soutstanding loan. Considering this supervening event, the insuranceproceeds shall inure to the benefit of the heirs of the deceased person or hisbeneficiaries. Equity dictates that DBP should not unjustly enrich itself at theexpense of another (Nemo cum alterius detrimenio protest). Hence, it cannotcollect the insurance proceeds, after it already foreclosed on the mortgage.The proceeds now rightly belong to Dr. Leuterio‘s heirs represented by hiswidow, herein private respondent Medarda Leuterio.

WHEREFORE, the petition is hereby DENIED. The Decision and Resolutionof the Court of Appeals in CA-G.R. CV 18341 is AFFIRMED withMODIFICATION that the petitioner is ORDERED to pay the insuranceproceeds amounting to Eighty-six thousand, two hundred (P86,200.00) pesosto the heirs of the insured, Dr. Wilfredo Leuterio (deceased), uponpresentation of proof of prior settlement of mortgagor‘s indebtedness toDevelopment Bank of the Philippines. Costs against petitioner.

SO ORDERED.

Mendoza, Buena, and De Leon Jr., JJ., concur.Bellosillo, (Chairman), J., on official leave.

G.R. No. L-30685 May 30, 1983

NG GAN ZEE, plaintiff-appellee,vs.ASIAN CRUSADER LIFE ASSURANCE CORPORATION,defendant-appellant.

Alberto Q. Ubay for plaintiff-appellee.

Santiago F. A lidio for defendant-appellant.

ESCOLIN, J.:

This is an appeal from the judgment of the Court of First Instance of Manila,ordering the appellant Asian-Crusader Life Assurance Corporation to pay theface value of an insurance policy issued on the life of Kwong Nam thedeceased husband of appellee Ng Gan Zee. Misrepresentation andconcealment of material facts in obtaining the policy were pleaded to avoid thepolicy. The lower court rejected the appellant's theory and ordered the latter topay appellee "the amount of P 20,000.00, with interest at the legal rate from

July 24, 1964, the date of the filing of the complaint, until paid, and the costs. "

The Court of Appeals certified this appeal to Us, as the same involves solely aquestion of law.

On May 12, 1962, Kwong Nam applied for a 20-year endowment insurance onhis life for the sum of P20,000.00, with his wife, appellee Ng Gan Zee asbeneficiary. On the same date, appellant, upon receipt of the requiredpremium from the insured, approved the application and issued thecorresponding policy. On December 6, 1963, Kwong Nam died of cancer ofthe liver with metastasis. All premiums had been religiously paid at the time ofhis death.

On January 10, 1964, his widow Ng Gan Zee presented a claim in due form toappellant for payment of the face value of the policy. On the same date, shesubmitted the required proof of death of the insured. Appellant denied theclaim on the ground that the answers given by the insured to the questionsappealing in his application for life insurance were untrue.

Appellee brought the matter to the attention of the Insurance Commissioner,the Hon. Francisco Y. Mandamus, and the latter, after conducting aninvestigation, wrote the appellant that he had found no material concealmenton the part of the insured and that, therefore, appellee should be paid the fullface value of the policy. This opinion of the Insurance Commissionernotwithstanding, appellant refused to settle its obligation.

Appellant alleged that the insured was guilty of misrepresentation when he

answered "No" to the following question appearing in the application for lifeinsurance-

Has any life insurance company ever refused your application for insurance orfor reinstatement of a lapsed policy or offered you a policy different from thatapplied for? If, so, name company and date.

In its brief, appellant rationalized its thesis thus:

... As pointed out in the foregoing summary of the essential facts in this case,the insured had in January, 1962, applied for reinstatement of his lapsed lifeinsurance policy with the Insular Life Insurance Co., Ltd, but this was declinedby the insurance company, although later on approved for reinstatement witha very high premium as a result of his medical examination. Thusnotwithstanding the said insured answered 'No' to the [above] questionpropounded to him. ... 1

The lower court found the argument bereft of factual basis; and We quote withapproval its disquisition on the matter-

On the first question there is no evidence that the Insular Life Assurance Co.,Ltd. ever refused any application of Kwong Nam for insurance. Neither is thereany evidence that any other insurance company has refused any applicationof Kwong Nam for insurance.

... The evidence shows that the Insular Life Assurance Co., Ltd. approvedKwong Nam's request for reinstatement and amendment of his lapsedinsurance policy on April 24, 1962 [Exh. L-2 Stipulation of Facts, Sept. 22,1965). The Court notes from said application for reinstatement andamendment, Exh. 'L', that the amount applied for was P20,000.00 only and notfor P50,000.00 as it was in the lapsed policy. The amount of the reinstated andamended policy was also for P20,000.00. It results, therefore, that when onMay 12, 1962 Kwong Nam answered 'No' to the question whether any lifeinsurance company ever refused his application for reinstatement of a lapsedpolicy he did not misrepresent any fact.

... the evidence shows that the application of Kwong Nam with the Insular Life Assurance Co., Ltd. was for the reinstatement and amendment of his lapsedinsurance policy-Policy No. 369531 -not an application for a 'new insurancepolicy. The Insular Life Assurance Co., Ltd. approved the said application on April 24, 1962. Policy No. 369531 was reinstated for the amount of P20,000.00as applied for by Kwong Nam [Exhs. 'L', 'L-l' and 'L-2']. No new policy wasissued by the Insular Life Assurance Co., Ltd. to Kwong Nam in connectionwith said application for reinstatement and amendment. Such being the case,the Court finds that there is no misrepresentation on this matter. 2

Appellant further maintains that when the insured was examined in connectionwith his application for life insurance, he gave the appellant's medicalexaminer false and misleading information as to his ailment and previousoperation. The alleged false statements given by Kwong Nam are as follows:

Operated on for a Tumor [mayoma] of the stomach. Claims that Tumor hasbeen associated with ulcer of stomach. Tumor taken out was hard and of ahen's egg size. Operation was two [2] years ago in Chinese General Hospitalby Dr. Yap. Now, claims he is completely recovered.

To demonstrate the insured's misrepresentation, appellant directs Ourattention to:

[1] The report of Dr. Fu Sun Yuan the physician who treated KwongNam at the Chinese General Hospital on May 22, 1960, i.e., about 2 yearsbefore he applied for an insurance policy on May 12, 1962. According to saidreport, Dr. Fu Sun Yuan had diagnosed the patient's ailment as 'peptic ulcer'

8/12/2019 Insurance Meeting 6 Cases

http://slidepdf.com/reader/full/insurance-meeting-6-cases 5/28

MEETING 6 CASES - INSURANCE

5

for which, an operation, known as a 'sub-total gastric resection was performedon the patient by Dr. Pacifico Yap; and

[2] The Surgical Pathology Report of Dr. Elias Pantangco showing thatthe specimen removed from the patient's body was 'a portion of the stomachmeasuring 12 cm. and 19 cm. along the lesser curvature with a diameter of 15cm. along the greatest dimension.

On the bases of the above undisputed medical data showing that the insuredwas operated on for peptic ulcer", involving the excision of a portion of thestomach, appellant argues that the insured's statement in his application that atumor, "hard and of a hen's egg size," was removed during said operation,constituted material concealment.

The question to be resolved may be propounded thus: Was appellant,because of insured's aforesaid representation, misled or deceived intoentering the contract or in accepting the risk at the rate of premium agreedupon?

The lower court answered this question in the negative, and We agree.

Section 27 of the Insurance Law [Act 2427] provides:

Sec. 27. Such party a contract of insurance must communicate to the other, ingood faith, all facts within his knowledge which are material to the contract,and which the other has not the means of ascertaining, and as to which hemakes no warranty. 3

Thus, "concealment exists where the assured had knowledge of a factmaterial to the risk, and honesty, good faith, and fair dealing requires that heshould communicate it to the assurer, but he designedly and intentionallywithholds the same." 4

It has also been held "that the concealment must, in the absence of inquiries,be not only material, but fraudulent, or the fact must have been intentionallywithheld." 5

Assuming that the aforesaid answer given by the insured is false, as claimedby the appellant. Sec. 27 of the Insurance Law, above-quoted, neverthelessrequires that fraudulent intent on the part of the insured be established toentitle the insurer to rescind the contract. And as correctly observed by thelower court, "misrepresentation as a defense of the insurer to avoid liability isan 'affirmative' defense. The duty to establish such a defense by satisfactoryand convincing evidence rests upon the defendant. The evidence before theCourt does not clearly and satisfactorily establish that defense."

It bears emphasis that Kwong Nam had informed the appellant's medicalexaminer that the tumor for which he was operated on was "associated withulcer of the stomach." In the absence of evidence that the insured hadsufficient medical knowledge as to enable him to distinguish between "pepticulcer" and "a tumor", his statement that said tumor was "associated with ulcerof the stomach, " should be construed as an expression made in good faith ofhis belief as to the nature of his ailment and operation. Indeed, such statementmust be presumed to have been made by him without knowledge of itsincorrectness and without any deliberate intent on his part to mislead theappellant.

While it may be conceded that, from the viewpoint of a medical expert, theinformation communicated was imperfect, the same was neverthelesssufficient to have induced appellant to make further inquiries about the ailmentand operation of the insured.

Section 32 of Insurance Law [Act No. 24271 provides as follows:

Section 32. The right to information of material facts maybe waived either bythe terms of insurance or by neglect to make inquiries as to such facts wherethey are distinctly implied in other facts of which information is communicated.

It has been held that where, upon the face of the application, a questionappears to be not answered at all or to be imperfectly answered, and theinsurers issue a policy without any further inquiry, they waive the imperfectionof the answer and render the omission to answer more fully immaterial. 6

As aptly noted by the lower court, "if the ailment and operation of Kwong Namhad such an important bearing on the question of whether the defendantwould undertake the insurance or not, the court cannot understand why thedefendant or its medical examiner did not make any further inquiries on suchmatters from the Chinese General Hospital or require copies of the hospitalrecords from the appellant before acting on the application for insurance. Thefact of the matter is that the defendant was too eager to accept the applicationand receive the insured's premium. It would be inequitable now to allow thedefendant to avoid liability under the circumstances."

Finding no reversible error committed by the trial court, the judgment appealedfrom is hereby affirmed, with costs against appellant Asian-Crusader life Assurance Corporation.

SO ORDERED.

Makasiar (Chairman), Aquino, Concepcion, Jr., Guerrero and De Castro), JJ.,concur.

Abad Santos, J., I reserve my vote.

G.R. No. 92492 June 17, 1993THELMA VDA. DE CANILANG, petitioner,vs.HON. COURT OF APPEALS and GREAT PACIFIC LIFE ASSURANCECORPORATION, respondents.

Simeon C. Sato for petitioner.

FELICIANO, J.:

On 18 June 1982, Jaime Canilang consulted Dr. Wilfredo B. Claudio and wasdiagnosed as suffering from "sinus tachycardia." The doctor prescribed thefollowing fro him: Trazepam, a tranquilizer; and Aptin, a beta-blocker drug. Mr.Canilang consulted the same doctor again on 3 August 1982 and this time was

found to have "acute bronchitis."

On next day, 4 August 1982, Jaime Canilang applied for a "non-medical"insurance policy with respondent Great Pacific Life Assurance Company("Great Pacific") naming his wife, Thelma Canilang, as his beneficiary. 1 JaimeCanilang was issued ordinary life insurance Policy No. 345163, with the facevalue of P19,700, effective as of 9 August 1982.

On 5 August 1983, Jaime Canilang died of "congestive heart failure,""anemia," and "chronic anemia." 2 Petitioner, widow and beneficiary of theinsured, filed a claim with Great Pacific which the insurer denied on 5December 1983 upon the ground that the insured had concealed materialinformation from it.

Petitioner then filed a complaint against Great Pacific with the InsuranceCommission for recovery of the insurance proceeds. During the hearing calledby the Insurance Commissioner, petitioner testified that she was not aware ofany serious illness suffered by her late husband 3 and that, as far as sheknew, her husband had died because of a kidney disorder. 4 A depositiongiven by Dr. Wilfredo Claudio was presented by petitioner. There Dr. Claudiostated that he was the family physician of the deceased Jaime Canilang 5 andthat he had previously treated him for "sinus tachycardia" and "acutebronchitis." 6 Great Pacific for its part presented Dr. Esperanza Quismorio, aphysicianand a medical underwriter working for Great Pacific. 7 She testified that thedeceased's insurance application had been approved on the basis of hismedical declaration. 8 She explained that as a rule, medical examinations arerequired only in cases where the applicant has indicated in his application forinsurance coverage that he has previously undergone medical consultationand hospitalization. 9

In a decision dated 5 November 1985, Insurance Commissioner Armando Ansaldo ordered Great Pacific to pay P19,700 plus legal interest andP2,000.00 as attorney's fees after holding that:

1. the ailment of Jaime Canilang was not so serious that, even if it hadbeen disclosed, it would not have affected Great Pacific's decision to insurehim;

2. Great Pacific had waived its right to inquire into the health conditionof the applicant by the issuance of the policy despite the lack of answers to"some of the pertinent questions" in the insurance application;

3. there was no intentional concealment on the part of the insuredJaime Canilang as he had thought that he was merely suffering from a minorailment and simple cold; 10 and

4. Batas Pambansa Blg. 847 which voids an insurance contract,whether or not concealment was intentionally made, was not applicable toCanilang's case as that law became effective only on 1 June 1985.

On appeal by Great Pacific, the Court of Appeals reversed and set aside thedecision of the Insurance Commissioner and dismissed Thelma Canilang'scomplaint and Great Pacific's counterclaim. The Court of Appealed found thatthe use of the word "intentionally" by the Insurance Commissioner in definingand resolving the issue agreed upon by the parties at pre-trial before theInsurance Commissioner was not supported by the evidence; that the issueagreed upon by the parties had been whether the deceased insured, JaimeCanilang, made a material concealment as the state of his health at the time of

the filing of insurance application, justifying respondent's denial of the claim.The Court of Appeals also found that the failure of Jaime Canilang to discloseprevious medical consultation and treatment constituted material informationwhich should have been communicated to Great Pacific to enable the latter to

8/12/2019 Insurance Meeting 6 Cases

http://slidepdf.com/reader/full/insurance-meeting-6-cases 6/28

MEETING 6 CASES - INSURANCE

6

make proper inquiries. The Court of Appeals finally held that the Ng Gan Zeecase which had involved misrepresentation was not applicable in respect ofthe case at bar which involves concealment.

Petitioner Thelma Canilang is now before this Court on a Petition for Reviewon Certiorari alleging that:

1. . . . the Honorable Court of Appeals, speaking with due respect,

erred in not holding that the issue in the case agreed upon between the partiesbefore the Insurance Commission is whether or not Jaime Canilang"intentionally" made material concealment in stating his state of health;

2. . . . at any rate, the non-disclosure of certain facts about hisprevious health conditions does not amount to fraud and private respondent isdeemed to have waived inquiry thereto. 11

The medical declaration which was set out in the application for insuranceexecuted by Jaime Canilang read as follows:

MEDICAL DECLARATION

I hereby declare that:

(1) I have not been confined in any hospital, sanitarium or infirmary,nor receive any medical or surgical advice/attention within the last five (5)years.

(2) I have never been treated nor consulted a physician for a heartcondition, high blood pressure, cancer, diabetes, lung, kidney, stomachdisorder, or any other physical impairment.

(3) I am, to the best of my knowledge, in good health.

EXCEPTIONS:

_____________________________________________________________ ___________________

GENERAL DECLARATION

I hereby declare that all the foregoing answers and statements are complete,true and correct. I hereby agree that if there be any fraud or misrepresentationin the above statements material to the risk, the INSURANCE COMPANYupon discovery within two (2) years from the effective date of insurance shallhave the right to declare such insurance null and void. That the liabilities of theCompany under the said Policy/TA/Certificate shall accrue and begin onlyfrom the date of commencement of risk stated in the Policy/TA/Certificate,provided that the first premium is paid and the Policy/TA/Certificate isdelivered to, and accepted by me in person, when I am in actual good health.

Signed at Manila his 4th day of August, 1992.

Illegible—————————— Signature of Applicant. 12

We note that in addition to the negative statements made by Mr. Canilang inparagraph 1 and 2 of the medical declaration, he failed to disclose in theappropriate space, under the caption "Exceptions," that he had twiceconsulted Dr. Wilfredo B. Claudio who had found him to be suffering from"sinus tachycardia" and "acute bronchitis."

The relevant statutory provisions as they stood at the time Great Pacific issuedthe contract of insurance and at the time Jaime Canilang died, are set out inP.D. No. 1460, also known as the Insurance Code of 1978, which went intoeffect on 11 June 1978. These provisions read as follows:

Sec. 26. A neglect to communicate that which a party knows and ought tocommunicate, is called a concealment.

xxx xxx xxx

Sec. 28. Each party to a contract of insurance must communicate to theother, in good faith, all factors within his knowledge which are material to thecontract and as to which he makes no warranty, and which the other has notthe means of ascertaining. (Emphasis supplied)

Under the foregoing provisions, the information concealed must beinformation which the concealing party knew and "ought to [have]communicate[d]," that is to say, information which was "material to thecontract." The test of materiality is contained in Section 31 of the InsuranceCode of 1978 which reads:

Sec. 31. Materially is to be determined not by the event, but solely by theprobable and reasonable influence of the facts upon the party to whom the

communication is due, in forming his estimate of the disadvantages of theproposed contract, or in making his inquiries. (Emphasis supplied)

"Sinus tachycardia" is considered present "when the heart rate exceeds 100beats per minute." 13 The symptoms of this condition include pounding in thechest and sometimes faintness and weakness of the person affected. Thefollowing elaboration was offered by Great Pacific and set out by the Court of Appeals in its Decision:

Sinus tachycardia is defined as sinus-initiated; heart rate faster than 100 beatsper minute. (Harrison' s Principles of Internal Medicine, 8th ed. [1978], p.1193.) It is, among others, a common reaction to heart disease, includingmyocardial infarction, and heart failure per se. (Henry J.L. Marriot, M.D.,Electrocardiography, 6th ed., [1977], p. 127.) The medication prescribed byDr. Claudio for treatment of Canilang's ailment on June 18, 1982, indicates thecondition that said physician was trying to manage. Thus, he prescribedTrazepam, (Philippine Index of Medical Specialties (PIMS), Vol. 14, No. 3,Dec. 1985, p. 112) which is anti-anxiety, anti-convulsant, muscle-relaxant; and Aptin, (Idem, p. 36) a cardiac drug, for palpitations and nervous heart. Suchtreatment could have been a very material information to the insurer indetermining the action to be take on Canilang's application for life insurancecoverage. 14

We agree with the Court of Appeals that the information which Jaime Canilangfailed to disclose was material to the ability of Great Pacific to estimate theprobable risk he presented as a subject of life insurance. Had Canilangdisclosed his visits to his doctor, the diagnosis made and medicinesprescribed by such doctor, in the insurance application, it may be reasonablyassumed that Great Pacific would have made further inquiries and would haveprobably refused to issue a non-medical insurance policy or, at the very least,required a higher premium for the same coverage. 15 The materiality of theinformation withheld by Great Pacific did not depend upon the state of mind ofJaime Canilang. A man's state of mind or subjective belief is not capable ofproof in our judicial process, except through proof of external acts or failure toact from which inferences as to his subjective belief may be reasonably drawn.Neither does materiality depend upon the actual or physical events whichensue. Materiality relates rather to the "probable and reasonable influence ofthe facts" upon the party to whom the communication should have been made,in assessing the risk involved in making or omitting to make further inquiriesand in accepting the application for insurance; that "probable and reasonableinfluence of the facts" concealed must, of course, be determined objectively,by the judge ultimately.

The insurance Great Pacific applied for was a "non-medical" insurance policy.In Saturnino v. Philippine-American Life Insurance Company, 16 this Courtheld that:

. . . if anything, the waiver of medical examination [in a non-medical insurancecontract] renders even more material the information required of the applicantconcerning previous condition of health and diseases suffered, for suchinformation necessarily constitutes an important factor which the insurer takesinto consideration in deciding whether to issue the policy or not . . . . 17(Emphasis supplied)

The Insurance Commissioner had also ruled that the failure of Great Pacific toconvey certain information to the insurer was not "intentional" in nature, for thereason that Jaime Canilang believed that he was suffering from minor ailmentlike a common cold. Section 27 of the Insurance Code of 1978 as it existedfrom 1974 up to 1985, that is, throughout the time range material for presentpurposes, provided that:

Sec. 27. A concealment entitles the injured party to rescind a contract ofinsurance.

The preceding statute, Act No. 2427, as it stood from 1914 up to 1974, hadprovided:

Sec. 26. A concealment, whether intentional or unintentional, entitles theinjured party to rescind a contract of insurance. (Emphasis supplied)

Upon the other hand, in 1985, the Insurance Code of 1978 was amended byB.P. Blg. 874. This subsequent statute modified Section 27 of the InsuranceCode of 1978 so as to read as follows:

Sec. 27. A concealment whether intentional or unintentional entitles theinjured party to rescind a contract of insurance. (Emphasis supplied)

The unspoken theory of the Insurance Commissioner appears to have beenthat by deleting the phrase "intentional or unintentional," the Insurance Codeof 1978 (prior to its amendment by B.P. Blg. 874) intended to limit the kinds ofconcealment which generate a right to rescind on the part of the injured partyto "intentional concealments." This argument is not persuasive. As a simple

matter of grammar, it may be noted that "intentional" and "unintentional"cancel each other out. The net result therefore of the phrase "whetherintentional or unitentional" is precisely to leave unqualified the term"concealment." Thus, Section 27 of the Insurance Code of 1978 is properly

8/12/2019 Insurance Meeting 6 Cases

http://slidepdf.com/reader/full/insurance-meeting-6-cases 7/28

MEETING 6 CASES - INSURANCE

7

read as referring to "any concealment" without regard to whether suchconcealment is intentional or unintentional. The phrase "whether intentional orunintentional" was in fact superfluous. The deletion of the phrase "whetherintentional or unintentional" could not have had the effect of imposing anaffirmative requirement that a concealment must be intentional if it is to entitlethe injured party to rescind a contract of insurance. The restoration in 1985 byB.P. Blg. 874 of the phrase "whether intentional or unintentional" merelyunderscored the fact that all throughout (from 1914 to 1985), the statute did

not require proof that concealment must be "intentional" in order to authorizerescission by the injured party.

In any case, in the case at bar, the nature of the facts not conveyed to theinsurer was such that the failure to communicate must have been intentionalrather than merely inadvertent. For Jaime Canilang could not have beenunaware that his heart beat would at times rise to high and alarming levels andthat he had consulted a doctor twice in the two (2) months before applying fornon-medical insurance. Indeed, the last medical consultation took place justthe day before the insurance application was filed. In all probability, JaimeCanilang went to visit his doctor precisely because of the discomfort andconcern brought about by his experiencing "sinus tachycardia."

We find it difficult to take seriously the argument that Great Pacific had waivedinquiry into the concealment by issuing the insurance policy notwithstanding

Canilang's failure to set out answers to some of the questions in the insuranceapplication. Such failure precisely constituted concealment on the part ofCanilang. Petitioner's argument, if accepted, would obviously erase Section27 from the Insurance Code of 1978.

It remains only to note that the Court of Appeals finding that the parties had notagreed in the pretrial before the Insurance Commission that the relevant issuewas whether or not Jaime Canilang had intentionally concealed materialinformation from the insurer, was supported by the evidence of record, i.e., thePre-trial Order itself dated 17 October 1984 and the Minutes of the Pre-trialConference dated 15 October 1984, which "readily shows that the word"intentional" does not appear in the statement or definition of the issue in thesaid Order and Minutes." 18

WHEREFORE, the Petition for Review is DENIED for lack of merit and theDecision of the Court of Appeals dated 16 October 1989 in C.A.-G.R. SP No.08696 is hereby AFFIRMED. No pronouncement as to the costs.

SO ORDERED.

Bidin, Davide, Jr., Romero and Melo, JJ., concur.

G.R. No. 105135 June 22, 1995

SUNLIFE ASSURANCE COMPANY OF CANADA, petitioner,vs.The Hon. COURT OF APPEALS and Spouses ROLANDO andBERNARDA BACANI, respondents.

QUIASON, J.:

This is a petition for review for certiorari under Rule 45 of the Revised Rules ofCourt to reverse and set aside the Decision dated February 21, 1992 of theCourt of Appeals in CA-G.R. CV No. 29068, and its Resolution dated April 22,1992, denying reconsideration thereof.

We grant the petition.

I

On April 15, 1986, Robert John B. Bacani procured a life insurance contract forhimself from petitioner. He was issued Policy No. 3-903-766-X valued atP100,000.00, with double indemnity in case of accidental death. Thedesignated beneficiary was his mother, respondent Bernarda Bacani.

On June 26, 1987, the insured died in a plane crash. Respondent BernardaBacani filed a claim with petitioner, seeking the benefits of the insurance policytaken by her son. Petitioner conducted an investigation and its findingsprompted it to reject the claim.

In its letter, petitioner informed respondent Bernarda Bacani, that the insureddid not disclose material facts relevant to the issuance of the policy, thusrendering the contract of insurance voidable. A check representing the totalpremiums paid in the amount of P10,172.00 was attached to said letter.

Petitioner claimed that the insured gave false statements in his application

when he answered the following questions:

5. Within the past 5 years have you:

a) consulted any doctor or other health practitioner?

b) submitted to:

EGG?X-rays?blood tests?other tests?

c) attended or been admitted to any hospital or other medical facility?

6. Have you ever had or sought advice for:

xxx xxx xxx

b) urine, kidney or bladder disorder? (Rollo, p. 53)

The deceased answered question No. 5(a) in the affirmative but limited hisanswer to a consultation with a certain Dr. Reinaldo D. Raymundo of theChinese General Hospital on February 1986, for cough and flu complications.The other questions were answered in the negative (Rollo, p. 53).

Petitioner discovered that two weeks prior to his application for insurance, the

insured was examined and confined at the Lung Center of the Philippines,where he was diagnosed for renal failure. During his confinement, thedeceased was subjected to urinalysis, ultra-sonography and hematologytests.

On November 17, 1988, respondent Bernarda Bacani and her husband,respondent Rolando Bacani, filed an action for specific performance againstpetitioner with the Regional Trial Court, Branch 191, Valenzuela, MetroManila. Petitioner filed its answer with counterclaim and a list of exhibitsconsisting of medical records furnished by the Lung Center of the Philippines.

On January 14, 1990, private respondents filed a "Proposed Stipulation withPrayer for Summary Judgment" where they manifested that they "have noevidence to refute the documentary evidence ofconcealment/misrepresentation by the decedent of his health condition (Rollo,p. 62).

Petitioner filed its Request for Admissions relative to the authenticity and dueexecution of several documents as well as allegations regarding the health ofthe insured. Private respondents failed to oppose said request or reply thereto,thereby rendering an admission of the matters alleged.

Petitioner then moved for a summary judgment and the trial court decided infavor of private respondents. The dispositive portion of the decision isreproduced as follows:

WHEREFORE, judgment is hereby rendered in favor of the plaintiffs andagainst the defendant, condemning the latter to pay the former the amount ofOne Hundred Thousand Pesos (P100,000.00) the face value of insured'sInsurance Policy No. 3903766, and the Accidental Death Benefit in theamount of One Hundred Thousand Pesos (P100,000.00) and further sum ofP5,000.00 in the concept of reasonable attorney's fees and costs of suit.

Defendant's counterclaim is hereby Dismissed (Rollo, pp. 43-44).

In ruling for private respondents, the trial court concluded that the factsconcealed by the insured were made in good faith and under a belief that theyneed not be disclosed. Moreover, it held that the health history of the insuredwas immaterial since the insurance policy was "non-medical".

Petitioner appealed to the Court of Appeals, which affirmed the decision of thetrial court. The appellate court ruled that petitioner cannot avoid its obligationby claiming concealment because the cause of death was unrelated to thefacts concealed by the insured. It also sustained the finding of the trial courtthat matters relating to the health history of the insured were irrelevant sincepetitioner waived the medical examination prior to the approval and issuanceof the insurance policy. Moreover, the appellate court agreed with the trialcourt that the policy was "non-medical" (Rollo, pp. 4-5).

Petitioner's motion for reconsideration was denied; hence, this petition.

II

We reverse the decision of the Court of Appeals.

The rule that factual findings of the lower court and the appellate court arebinding on this Court is not absolute and admits of exceptions, such as whenthe judgment is based on a misappreciation of the facts (Geronimo v. Court of Appeals, 224 SCRA 494 [1993]).

In weighing the evidence presented, the trial court concluded that indeed therewas concealment and misrepresentation, however, the same was made in

8/12/2019 Insurance Meeting 6 Cases

http://slidepdf.com/reader/full/insurance-meeting-6-cases 8/28

MEETING 6 CASES - INSURANCE

8

"good faith" and the facts concealed or misrepresented were irrelevant sincethe policy was "non-medical". We disagree.

Section 26 of The Insurance Code is explicit in requiring a party to a contract ofinsurance to communicate to the other, in good faith, all facts within hisknowledge which are material to the contract and as to which he makes nowarranty, and which the other has no means of ascertaining. Said Sectionprovides:

A neglect to communicate that which a party knows and ought tocommunicate, is called concealment.

Materiality is to be determined not by the event, but solely by the probable andreasonable influence of the facts upon the party to whom communication isdue, in forming his estimate of the disadvantages of the proposed contract orin making his inquiries (The Insurance Code, Sec. 31).

The terms of the contract are clear. The insured is specifically required todisclose to the insurer matters relating to his health.

The information which the insured failed to disclose were material and relevantto the approval and issuance of the insurance policy. The matters concealedwould have definitely affected petitioner's action on his application, either by

approving it with the corresponding adjustment for a higher premium orrejecting the same. Moreover, a disclosure may have warranted a medicalexamination of the insured by petitioner in order for it to reasonably assess therisk involved in accepting the application.

In Vda. de Canilang v. Court of Appeals, 223 SCRA 443 (1993), we held thatmateriality of the information withheld does not depend on the state of mind ofthe insured. Neither does it depend on the actual or physical events whichensue.

Thus, "goad faith" is no defense in concealment. The insured's failure todisclose the fact that he was hospitalized for two weeks prior to filing hisapplication for insurance, raises grave doubts about his bonafides. It appearsthat such concealment was deliberate on his part.

The argument, that petitioner's waiver of the medical examination of theinsured debunks the materiality of the facts concealed, is untenable. Wereiterate our ruling in Saturnino v. Philippine American Life InsuranceCompany, 7 SCRA 316 (1963), that " . . . the waiver of a medical examination[in a non-medical insurance contract] renders even more material theinformation required of the applicant concerning previous condition of healthand diseases suffered, for such information necessarily constitutes animportant factor which the insurer takes into consideration in deciding whetherto issue the policy or not . . . "

Moreover, such argument of private respondents would make Section 27 ofthe Insurance Code, which allows the injured party to rescind a contract ofinsurance where there is concealment, ineffective (See Vda. de Canilang v.Court of Appeals, supra).

Anent the finding that the facts concealed had no bearing to the cause of deathof the insured, it is well settled that the insured need not die of the disease hehad failed to disclose to the insurer. It is sufficient that his non-disclosuremisled the insurer in forming his estimates of the risks of the proposedinsurance policy or in making inquiries (Henson v. The Philippine AmericanLife Insurance Co., 56 O.G. No. 48 [1960]).

We, therefore, rule that petitioner properly exercised its right to rescind thecontract of insurance by reason of the concealment employed by the insured.It must be emphasized that rescission was exercised within the two-yearcontestability period as recognized in Section 48 of The Insurance Code.

WHEREFORE, the petition is GRANTED and the Decision of the Court of Appeals is REVERSED and SET ASIDE.

SO ORDERED.

Padilla, Davide, Jr., Bellosillo and Kapunan, JJ., concur.

DIGEST

SUN LIFE V. CA - CONCEALMENT IN INSURANCE245 SCRA 268 (1995)Facts:> On April 15, 1986, Bacani procured a life insurance contract for himselffrom Sun Life. He was issued a life insurance policy with double indemnity incase of accidental death. The designated beneficiary was his mother,Bernarda.> On June 26, 1987, the insured died in a plane crash. Bernarda Bacani filed

a claim with Sun Life, seeking the benefits of the insurance. Sun Lifeconducted an investigation and its findings prompted it to reject the claim.> Sun Life discovered that 2 weeks prior to his application, Bacani wasexamined and confined at the Lung Center of the Philippines, where he was

diagnosed for renal failure. During his confinement, the deceased wassubjected to urinalysis, ultra-sonography and hematology tests. He did notreveal such fact in his application.> In its letter, Sun Life informed Berarda, that the insured did not disclosedmaterial facts relevant to the issuance of the policy, thus rendering thecontract of insurance voidable. A check representing the total premiums paidin the amount of P10,172.00 was attached to said letter.> Bernarda and her husband, filed an action for specific performance against

Sun Life. RTC ruled for Bernarda holding that the facts concealed by theinsured were made in good faith and under the belief that they need not bedisclosed. Moreover, it held that the health history of the insured wasimmaterial since the insurance policy was "non-medical." CA affirmed.Issue:Whether or not the beneficiary can claim despite the concealment.

Held:NOPE.Section 26 of the Insurance Code is explicit in requiring a party to a contract ofinsurance to communicate to the other, in good faith, all facts within hisknowledge which are material to the contract and as to which he makes nowarranty, and which the other has no means of ascertaining.

Materiality is to be determined not by the event, but solely by the probable and

reasonable influence of the facts upon the party to whom communication isdue, in forming his estimate of the disadvantages of the proposed contract orin making his inquiries (The Insurance Code, Sec 31)

The terms of the contract are clear. The insured is specifically required todisclose to the insurer matters relating to his health. The information which theinsured failed to disclose were material and relevant to the approval and theissuance of the insurance policy. The matters concealed would have definitelyaffected petitioner's action on his application, either by approving it with thecorresponding adjustment for a higher premium or rejecting the same.Moreover, a disclosure may have warranted a medical examination of theinsured by petitioner in order for it to reasonably assess the risk involved inaccepting the application.

Thus, "good faith" is no defense in concealment. The insured's failure todisclose the fact that he was hospitalized for two weeks prior to filing hisapplication for insurance, raises grave doubts about his bonafides. It appearsthat such concealment was deliberate on his part.

G.R. No. L-31845 April 30, 1979

GREAT PACIFIC LIFE ASSURANCE COMPANY, petitioner,vs.HONORABLE COURT OF APPEALS, respondents.

G.R. No. L-31878 April 30, 1979

LAPULAPU D. MONDRAGON, petitioner,vs.HON. COURT OF APPEALS and NGO HING, respondents.

Siguion Reyna, Montecillo & Ongsiako and Sycip, Salazar, Luna &Manalo for petitioner Company.

Voltaire Garcia for petitioner Mondragon.

Pelaez, Pelaez & Pelaez for respondent Ngo Hing.

DE CASTRO, J.:

The two above-entitled cases were ordered consolidated by the Resolution ofthis Court dated April 29, 1970, (Rollo, No. L-31878, p. 58), because thepetitioners in both cases seek similar relief, through these petitions forcertiorari by way of appeal, from the amended decision of respondent Court of Appeals which affirmed in toto the decision of the Court of First Instance ofCebu, ordering "the defendants (herein petitioners Great Pacific Ligfe Assurance Company and Mondragon) jointly and severally to pay plaintiff(herein private respondent Ngo Hing) the amount of P50,000.00 with interestat 6% from the date of the filing of the complaint, and the sum of P1,077.75,without interest.

It appears that on March 14, 1957, private respondent Ngo Hing filed anapplication with the Great Pacific Life Assurance Company (hereinafterreferred to as Pacific Life) for a twenty-year endownment policy in the amountof P50,000.00 on the life of his one-year old daughter Helen Go. Saidrespondent supplied the essential data which petitioner Lapulapu D.Mondragon, Branch Manager of the Pacific Life in Cebu City wrote on the

corresponding form in his own handwriting (Exhibit I-M). Mondragon finallytype-wrote the data on the application form which was signed by privaterespondent Ngo Hing. The latter paid the annual premuim the sum ofP1,077.75 going over to the Company, but he reatined the amount of

8/12/2019 Insurance Meeting 6 Cases

http://slidepdf.com/reader/full/insurance-meeting-6-cases 9/28

MEETING 6 CASES - INSURANCE

9

P1,317.00 as his commission for being a duly authorized agebt of Pacific Life.Upon the payment of the insurance premuim, the binding deposit receipt(Exhibit E) was issued to private respondent Ngo Hing. Likewise, petitionerMondragon handwrote at the bottom of the back page of the application formhis strong recommendation for the approval of the insurance application. Thenon April 30, 1957, Mondragon received a letter from Pacific Life disapprovingthe insurance application (Exhibit 3-M). The letter stated that the said lifeinsurance application for 20-year endowment plan is not available for minors

below seven years old, but Pacific Life can consider the same under theJuvenile Triple Action Plan, and advised that if the offer is acceptable, theJuvenile Non-Medical Declaration be sent to the company.

The non-acceptance of the insurance plan by Pacific Life was allegedly notcommunicated by petitioner Mondragon to private respondent Ngo Hing.Instead, on May 6, 1957, Mondragon wrote back Pacific Life again stronglyrecommending the approval of the 20-year endowment insurance plan tochildren, pointing out that since 1954 the customers, especially the Chinese,were asking for such coverage (Exhibit 4-M).

It was when things were in such state that on May 28, 1957 Helen Go died ofinfluenza with complication of bronchopneumonia. Thereupon, privaterespondent sought the payment of the proceeds of the insurance, but havingfailed in his effort, he filed the action for the recovery of the same before the

Court of First Instance of Cebu, which rendered the adverse decision asearlier refered to against both petitioners.

The decisive issues in these cases are: (1) whether the binding deposit receipt(Exhibit E) constituted a temporary contract of the life insurance in question;and (2) whether private respondent Ngo Hing concealed the state of healthand physical condition of Helen Go, which rendered void the aforesaid ExhibitE.

1. At the back of Exhibit E are condition precedents required before a depositis considered a BINDING RECEIPT. These conditions state that:

A. If the Company or its agent, shan have received the premium deposit ... andthe insurance application, ON or PRIOR to the date of medical examination ...said insurance shan be in force and in effect from the date of such medicalexamination, for such period as is covered by the deposit ..., PROVIDED thecompany shall be satisfied that on said date the applicant was insurable onstandard rates under its rule for the amount of insurance and the kind of policyrequested in the application.

D. If the Company does not accept the application on standard rate for theamount of insurance and/or the kind of policy requested in the application butissue, or offers to issue a policy for a different plan and/or amount ..., theinsurance shall not be in force and in effect until the applicant shall haveaccepted the policy as issued or offered by the Company and shall have paidthe full premium thereof. If the applicant does not accept the policy, the depositshall be refunded.

E. If the applicant shall not have been insurable under Condition A above, andthe Company declines to approve the application the insurance applied forshall not have been in force at any time and the sum paid be returned to theapplicant upon the surrender of this receipt. (Emphasis Ours).

The aforequoted provisions printed on Exhibit E show that the binding depositreceipt is intended to be merely a provisional or temporary insurance contractand only upon compliance of the following conditions: (1) that the companyshall be satisfied that the applicant was insurable on standard rates; (2) that ifthe company does not accept the application and offers to issue a policy for adifferent plan, the insurance contract shall not be binding until the applicantaccepts the policy offered; otherwise, the deposit shall be reftmded; and (3)that if the applicant is not ble according to the standard rates, and thecompany disapproves the application, the insurance applied for shall not be inforce at any time, and the premium paid shall be returned to the applicant.

Clearly implied from the aforesaid conditions is that the binding deposit receiptin question is merely an acknowledgment, on behalf of the company, that thelatter's branch office had received from the applicant the insurance premiumand had accepted the application subject for processing by the insurancecompany; and that the latter will either approve or reject the same on the basisof whether or not the applicant is "insurable on standard rates." Sincepetitioner Pacific Life disapproved the insurance application of respondentNgo Hing, the binding deposit receipt in question had never become in force atany time.

Upon this premise, the binding deposit receipt (Exhibit E) is, manifestly,merely conditional and does not insure outright. As held by this Court, wherean agreement is made between the applicant and the agent, no liability shallattach until the principal approves the risk and a receipt is given by the agent.The acceptance is merely conditional and is subordinated to the act of the

company in approving or rejecting the application. Thus, in life insurance, a"binding slip" or "binding receipt" does not insure by itself (De Lim vs. Sun Life Assurance Company of Canada, 41 Phil. 264).

It bears repeating that through the intra-company communication of April 30,1957 (Exhibit 3-M), Pacific Life disapproved the insurance application inquestion on the ground that it is not offering the twenty-year endowmentinsurance policy to children less than seven years of age. What it offeredinstead is another plan known as the Juvenile Triple Action, which privaterespondent failed to accept. In the absence of a meeting of the minds betweenpetitioner Pacific Life and private respondent Ngo Hing over the 20-yearendowment life insurance in the amount of P50,000.00 in favor of the latter's

one-year old daughter, and with the non-compliance of the abovequotedconditions stated in the disputed binding deposit receipt, there could havebeen no insurance contract duly perfected between thenl Accordingly, thedeposit paid by private respondent shall have to be refunded by Pacific Life.

As held in De Lim vs. Sun Life Assurance Company of Canada, supra, "acontract of insurance, like other contracts, must be assented to by both partieseither in person or by their agents ... The contract, to be binding from the dateof the application, must have been a completed contract, one that leavesnothing to be dione, nothing to be completed, nothing to be passed upon, ordetermined, before it shall take effect. There can be no contract of insuranceunless the minds of the parties have met in agreement."

We are not impressed with private respondent's contention that failure ofpetitioner Mondragon to communicate to him the rejection of the insurance

application would not have any adverse effect on the allegedly perfectedtemporary contract (Respondent's Brief, pp. 13-14). In this first place, therewas no contract perfected between the parties who had no meeting of theirminds. Private respondet, being an authorized insurance agent of Pacific Lifeat Cebu branch office, is indubitably aware that said company does not offerthe life insurance applied for. When he filed the insurance application indispute, private respondent was, therefore, only taking the chance that PacificLife will approve the recommendation of Mondragon for the acceptance andapproval of the application in question along with his proposal that theinsurance company starts to offer the 20-year endowment insurance plan forchildren less than seven years. Nonetheless, the record discloses that PacificLife had rejected the proposal and recommendation. Secondly, having aninsurable interest on the li fe of his one-year old daughter, aside from being aninsurance agent and an offense associate of petitioner Mondragon, privaterespondent Ngo Hing must have known and followed the progress on theprocessing of such application and could not pretend ignorance of theCompany's rejection of the 20-year endowment life insurance application.

At this juncture, We find it fit to quote with approval, the very apt observation ofthen Appellate Associate Justice Ruperto G. Martin who later came up to thisCourt, from his dissenting opinion to the amended decision of the respondentcourt which completely reversed the original decision, the following:

Of course, there is the insinuation that neither the memorandum of rejection(Exhibit 3-M) nor the reply thereto of appellant Mondragon reiterating thedesire for applicant's father to have the application considered as one for a20-year endowment plan was ever duly communicated to Ngo; Hing, father ofthe minor applicant. I am not quite conninced that this was so. Ngo Hing, asfather of the applicant herself, was precisely the "underwriter who wrote thiscase" (Exhibit H-1). The unchallenged statement of appellant Mondragon inhis letter of May 6, 1957) (Exhibit 4-M), specifically admits that said Ngo Hingwas "our associate" and that it was the latter who "insisted that the plan beplaced on the 20-year endowment plan." Under these circumstances, it isinconceivable that the progress in the processing of the application was notbrought home to his knowledge. He must have been duly apprised of therejection of the application for a 20-year endowment plan otherwiseMondragon would not have asserted that it was Ngo Hing himself who insistedon the application as originally filed, thereby implictly declining the offer toconsider the application under the Juvenile Triple Action Plan. Besides, theassociate of Mondragon that he was, Ngo Hing should only be presumed toknow what kind of policies are available in the company for minors below 7years old. What he and Mondragon were apparently trying to do in thepremises was merely to prod the company into going into the business ofissuing endowment policies for minors just as other insurance companiesallegedly do. Until such a definite policy is however, adopted by the company,it can hardly be said that it could have been bound at all under the binding slipfor a plan of insurance that it could not have, by then issued at all. (AmendedDecision, Rollo, pp- 52-53).

2. Relative to the second issue of alleged concealment. this Court is of the firmbelief that private respondent had deliberately concealed the state of healthand piysical condition of his daughter Helen Go. Wher private regpondeitsupplied the required essential data for the insurance application form, he wasfully aware that his one-year old daughter is typically a mongoloid child. Sucha congenital physical defect could never be ensconced nor disguished.Nonetheless, private respondent, in apparent bad faith, withheld the factmateral to the risk to be assumed by the insurance compary. As an insuranceagent of Pacific Life, he ought to know, as he surely must have known. his dutyand responsibility to such a material fact. Had he diamond said significant fact

in the insurance application fom Pacific Life would have verified the same andwould have had no choice but to disapprove the application outright.

8/12/2019 Insurance Meeting 6 Cases

http://slidepdf.com/reader/full/insurance-meeting-6-cases 10/28

MEETING 6 CASES - INSURANCE

10

The contract of insurance is one of perfect good faith uberrima fides meaninggood faith, absolute and perfect candor or openness and honesty; theabsence of any concealment or demotion, however slight [Black's LawDictionary, 2nd Edition], not for the alone but equally so for the insurer (Fieldman's Insurance Co., Inc. vs. Vda de Songco, 25 SCRA 70). Concealment is aneglect to communicate that which a partY knows aDd Ought to communicate(Section 25, Act No. 2427). Whether intentional or unintentional theconcealment entitles the insurer to rescind the contract of insurance (Section

26, Id.: Yu Pang Cheng vs. Court of Appeals, et al, 105 Phil 930; Satumino vs.Philippine American Life Insurance Company, 7 SCRA 316). Privaterespondent appears guilty thereof.

We are thus constrained to hold that no insurance contract was perfectedbetween the parties with the noncompliance of the conditions provided in thebinding receipt, and concealment, as legally defined, having been comraittedby herein private respondent.

WHEREFORE, the decision appealed from is hereby set aside, and in lieuthereof, one is hereby entered absolving petitioners Lapulapu D. Mondragonand Great Pacific Life Assurance Company from their civil liabilities as foundby respondent Court and ordering the aforesaid insurance company toreimburse the amount of P1,077.75, without interest, to private respondent,Ngo Hing. Costs against private respondent.

SO ORDERED.

Teehankee (Chairman), Makasiar, Guerrero and Melencio-Herrera, JJ.,concur.

Fernandez, J., took no part.

YU PANG CHENG V. CA- LIFE INSURANCE POLICY105 PHIL 1930Facts:> Yu Pang Eng obtained a life insurance policy naming his brother Yu PangCheng as beneficiary.> Eng subsequently died of medullary carcinoma, Grade 4, advanced andlesser curvature.> Cheng claims the proceeds of the policy.> Insurance co. refused payment on the ground that the policy was void dueto the concealment.

Issue:Whether or not the policy is void.

Held:YES.In the application for the policy, Eng was asked whether he had been ill or hadconsulted a doctor due to symptoms or illnesses enumerated in thequestionnaire. He answered ― No‖, when in fact he was hospitalized sevenmonths prior to his application for the said policy.

G.R. No. 48049 June 29, 1989

EMILIO TAN, JUANITO TAN, ALBERTO TAN and ARTURO TAN,petitioners,vs.THE COURT OF APPEALS and THE PHILIPPINE AMERICAN LIFEINSURANCE COMPANY, respondents.

O.F. Santos & P.C. Nolasco for petitioners.

Ferry, De la Rosa and Associates for private respondent.

GUTIERREZ, JR., J.:

This is a petition for review on certiorari of the Court of Appeals' decisionaffirming the decision of the Insurance Commissioner which dismissed thepetitioners' complaint against respondent Philippine American Life InsuranceCompany for the recovery of the proceeds from their late father's policy. Thefacts of the case as found by the Court of Appeals are:

Petitioners appeal from the Decision of the Insurance Commissionerdismissing herein petitioners' complaint against respondent Philippine American Life Insurance Company for the recovery of the proceeds of PolicyNo. 1082467 in the amount of P 80,000.00.

On September 23,1973, Tan Lee Siong, father of herein petitioners, appliedfor life insurance in the amount of P 80,000.00 with respondent company. Saidapplication was approved and Policy No. 1082467 was issued effective

November 6,1973, with petitioners the beneficiaries thereof (Exhibit A).

On April 26,1975, Tan Lee Siong died of hepatoma (Exhibit B). Petitionersthen filed with respondent company their claim for the proceeds of the life

insurance policy. However, in a letter dated September 11, 1975, respondentcompany denied petitioners' claim and rescinded the policy by reason of thealleged misrepresentation and concealment of material facts made by thedeceased Tan Lee Siong in his application for insurance (Exhibit 3). Thepremiums paid on the policy were thereupon refunded .

Alleging that respondent company's refusal to pay them the proceeds of thepolicy was unjustified and unreasonable, petitioners filed on November 27,

1975, a complaint against the former with the Office of the InsuranceCommissioner, docketed as I.C. Case No. 218.

After hearing the evidence of both parties, the Insurance Commissionerrendered judgment on August 9, 1977, dismissing petitioners' complaint.(Rollo, pp. 91-92)

The Court of Appeals dismissed ' the petitioners' appeal from the InsuranceCommissioner's decision for lack of merit

Hence, this petition.

The petitioners raise the following issues in their assignment of errors, to wit:

A. The conclusion in law of respondent Court that respondent insurer

has the right to rescind the policy contract when insured is already dead is notin accordance with existing law and applicable jurisprudence.

B. The conclusion in law of respondent Court that respondent insurermay be allowed to avoid the policy on grounds of concealment by thedeceased assured, is contrary to the provisions of the policy contract itself, aswell as, of applicable legal provisions and established jurisprudence.

C. The inference of respondent Court that respondent insurer wasmisled in issuing the policy are manifestly mistaken and contrary to admittedevidence. (Rollo, p. 7)

The petitioners contend that the respondent company no longer had the rightto rescind the contract of insurance as rescission must allegedly be doneduring the lifetime of the insured within two years and prior to thecommencement of action.

The contention is without merit.

The pertinent section in the Insurance Code provides:

Section 48. Whenever a right to rescind a contract of insurance is given to theinsurer by any provision of this chapter, such right must be exercised previousto the commencement of an action on the contract.

After a policy of life insurance made payable on the death of the insured shallhave been in force during the lifetime of the insured for a period of two yearsfrom the date of its issue or of its last reinstatement, the insurer cannot provethat the policy is void ab initio or is rescindable by reason of the fraudulentconcealment or misrepresentation of the insured or his agent.

According to the petitioners, the Insurance Law was amended and the secondparagraph of Section 48 added to prevent the insurance company fromexercising a right to rescind after the death of the insured.

The so-called "incontestability clause" precludes the insurer from raising thedefenses of false representations or concealment of material facts insofar ashealth and previous diseases are concerned if the insurance has been in forcefor at least two years during the insured's lifetime. The phrase "during thelifetime" found in Section 48 simply means that the policy is no longerconsidered in force after the insured has died. The key phrase in the secondparagraph of Section 48 is "for a period of two years."

As noted by the Court of Appeals, to wit:

The policy was issued on November 6,1973 and the insured died on April26,1975. The policy was thus in force for a period of only one year and fivemonths. Considering that the insured died before the two-year period hadlapsed, respondent company is not, therefore, barred from proving that thepolicy is void ab initio by reason of the insured's fraudulent concealment ormisrepresentation. Moreover, respondent company rescinded the contract ofinsurance and refunded the premiums paid on September 11, 1975, previousto the commencement of this action on November 27,1975. (Rollo, pp.99-100)

xxx xxx xxx

The petitioners contend that there could have been no concealment ormisrepresentation by their late father because Tan Lee Siong did not have to

buy insurance. He was only pressured by insistent salesmen to do so. Thepetitioners state:

8/12/2019 Insurance Meeting 6 Cases

http://slidepdf.com/reader/full/insurance-meeting-6-cases 11/28

MEETING 6 CASES - INSURANCE

11

Here then is a case of an assured whose application was submitted becauseof repeated visits and solicitations by the insurer's agent. Assured did notknock at the door of the insurer to buy insurance. He was the object ofsolicitations and visits.

Assured was a man of means. He could have obtained a bigger insurance, not just P 80,000.00. If his purpose were to misrepresent and to conceal hisailments in anticipation of death during the two-year period, he certainly could

have gotten a bigger insurance. He did not.Insurer Philamlife could have presented as witness its Medical Examiner Dr.Urbano Guinto. It was he who accomplished the application, Part II, medical.Philamlife did not.

Philamlife could have put to the witness stand its Agent Bienvenido S. Guinto,a relative to Dr. Guinto, Again Philamlife did not. (pp. 138139, Rollo)

xxx xxx xxx

This Honorable Supreme Court has had occasion to denounce the pressureand practice indulged in by agents in selling insurance. At one time or anothermost of us have been subjected to that pressure, that practice. This court took judicial cognizance of the whirlwind pressure of insurance selling-especially of

the agent's practice of 'supplying the information, preparing and answering theapplication, submitting the application to their companies, concluding thetransactions and otherwise smoothing out all difficulties.

We call attention to what this Honorable Court said in Insular Life v. Feliciano,et al., 73 Phil. 201; at page 205:

It is of common knowledge that the selling of insurance today is subjected tothe whirlwind pressure of modern salesmanship.

Insurance companies send detailed instructions to their agents to solicit andprocure applications.

These agents are to be found all over the length and breadth of the land. Theyare stimulated to more active efforts by contests and by the keen competitionoffered by the other rival insurance companies.

They supply all the information, prepare and answer the applications, submitthe applications to their companies, conclude the transactions, and otherwisesmooth out all difficulties.

The agents in short do what the company set them out to do.

The Insular Life case was decided some forty years ago when the pressure ofinsurance salesmanship was not overwhelming as it is now; when thepopulation of this country was less than one-fourth of what it is now; when theinsurance companies competing with one another could be counted by thefingers. (pp. 140-142, Rollo)

xxx xxx xxx

In the face of all the above, it would be unjust if, having been subjected to thewhirlwind pressure of insurance salesmanship this Court itself has longdenounced, the assured who dies within the two-year period, should standcharged of fraudulent concealment and misrepresentation." (p. 142, Rollo)

The legislative answer to the arguments posed by the petitioners is the"incontestability clause" added by the second paragraph of Section 48.

The insurer has two years from the date of issuance of the insurance contractor of its last reinstatement within which to contest the policy, whether or not,the insured still lives within such period. After two years, the defenses ofconcealment or misrepresentation, no matter how patent or well founded, nolonger lie. Congress felt this was a sufficient answer to the various tacticsemployed by insurance companies to avoid liability. The petitioners'interpretation would give rise to the incongruous situation where thebeneficiaries of an insured who dies right after taking out and paying for a l ifeinsurance policy, would be allowed to collect on the policy even if the insuredfraudulently concealed material facts.

The petitioners argue that no evidence was presented to show that themedical terms were explained in a layman's language to the insured. Theystate that the insurer should have presented its two medical field examiners aswitnesses. Moreover, the petitioners allege that the policy intends that themedical examination must be conducted before its issuance otherwise theinsurer "waives whatever imperfection by ratification."

We agree with the Court of Appeals which ruled:

On the other hand, petitioners argue that no evidence was presented byrespondent company to show that the questions appearing in Part II of theapplication for insurance were asked, explained to and understood by thedeceased so as to prove concealment on his part. The same is not well taken.

The deceased, by affixing his signature on the application form, affirmed thecorrectness of all the entries and answers appearing therein. It is but to beexpected that he, a businessman, would not have affixed his signature on theapplication form unless he clearly understood its significance. For, thepresumption is that a person intends the ordinary consequence of hisvoluntary act and takes ordinary care of his concerns. [Sec. 5(c) and (d), Rule131, Rules of Court].

The evidence for respondent company shows that on September 19,1972, thedeceased was examined by Dr. Victoriano Lim and was found to be diabeticand hypertensive; that by January, 1973, the deceased was complaining ofprogressive weight loss and abdominal pain and was diagnosed to besuffering from hepatoma, (t.s.n. August 23, 1976, pp. 8-10; Exhibit 2). Anotherphysician, Dr. Wenceslao Vitug, testified that the deceased came to see himon December 14, 1973 for consolation and claimed to have been diabetic forfive years. (t.s.n., Aug. 23,1976, p. 5; Exhibit 6) Because of the concealmentmade by the deceased of his consultations and treatments for hypertension,diabetes and liver disorders, respondent company was thus misled intoaccepting the risk and approving his application as medically standard (Exhibit5- C) and dispensing with further medical investigation and examination(Exhibit 5-A). For as long as no adverse medical history is revealed in theapplication form, an applicant for insurance is presumed to be healthy andphysically fit and no further medical investigation or examination is conducted

by respondent company. (t.s.n., April 8,1976, pp. 6-8). (Rollo, pp. 96-98)

There is no strong showing that we should apply the "fine print" or "contract ofadhesion" rule in this case. (Sweet Lines, Inc. v. Teves, 83 SCRA 361 [1978]).The petitioners cite:

It is a matter of common knowledge that large amounts of money are collectedfrom ignorant persons by companies and associations which adopt highsounding titles and print the amount of benefits they agree to pay in largeblack-faced type, following such undertakings by fine print conditions whichdestroy the substance of the promise. All provisions, conditions, or exceptionswhich in any way tend to work a forfeiture of the policy should be construedmost strongly against those for whose benefit they are inserted, and mostfavorably toward those against whom they are meant to operate. (Trinidad v.Orient Protective Assurance Assn., 67 Phil. 184)

There is no showing that the questions in the application form for insuranceregarding the insured's medical history are in smaller print than the rest of theprinted form or that they are designed in such a way as to conceal from theapplicant their importance. If a warning in bold red letters or a boxed warningsimilar to that required for cigarette advertisements by the Surgeon General ofthe United States is necessary, that is for Congress or the InsuranceCommission to provide as protection against high pressure insurancesalesmanship. We are limited in this petition to ascertaining whether or not therespondent Court of Appeals committed reversible error. It is the petitioners'burden to show that the factual findings of the respondent court are not basedon substantial evidence or that its conclusions are contrary to applicable lawand jurisprudence. They have failed to discharge that burden.

WHEREFORE, the petition is hereby DENIED for lack of merit. Thequestioned decision of the Court of Appeals is AFFIRMED.

SO ORDERED.

Fernan, (C.J., Chairman), Bidin and Cortes, JJ., concur.

Feliciano, took no part.

EDILLON V. MANILA BANKERS LIFE INSURANCE CORP. -CONCEALMENT

117 SCRA 187Facts:> In Apr. 1969, Carmen Lapuz applied for insurance with Manila Bankers.In the application she stated the date of her birth as July 11, 1904 (around 64yrs old). The policy was thereafter issued.> Subsequently, in May 1969, Carmen died of a car accident. Her sister, asbeneficiary claimed the proceeds of the insurance.> Manila Bankers refused to pay because the certificate of insurancecontained a provision excluding it‘s liability to pay claims to persons under 16or over 60.

Issue:Whether or not the policy is void considering that the insured was over 60when she applied.

Held:NO.The age of Carmen was not concealed to the insurance company. Her

application form indicated her true age. Despite such information, ManilaBankers accepted the premium and issued the policy. It had all the time toprocess the application and notice the applicant‘s age. If it failed to act, it was

8/12/2019 Insurance Meeting 6 Cases

http://slidepdf.com/reader/full/insurance-meeting-6-cases 12/28

MEETING 6 CASES - INSURANCE

12

because Manila Bankers was willing to waive such disqualifications or it simplyoverlooked such fact. It is therefore estopped from disclaiming any liability.

G.R. No. L-41014 November 28, 1988

PACIFIC BANKING CORPORATION, petitioner,vs.COURT OF APPEALS and ORIENTAL ASSURANCE CORPORATION,

respondents.Flores, Ocampo, Dizon and Domingo Law Office for petitioner.

Cabochan and Reyes Law Office for respondents.

PARAS, J.:

This is a petition for review on certiorari of the decision of respondent Court of Appeals * in CA-G.R. No. 41735-R, entitled "Pacific Banking Corporation vs.Oriental Assurance Corporation", which set aside the decision of the Court ofFirst Instance (CFI) of Manila, ** which had in turn granted the complaint for asum of money in Civil Case No. 56889.

As gathered from the records, the undisputed facts of this case are as follows:

On October 21,1963, Fire Policy No. F-3770 (Exhibit "A"), an open policy, wasissued to the Paramount Shirt Manufacturing Co. (hereinafter referred to asthe insured, for brevity), by which private respondent Oriental AssuranceCorporation bound itself to indemnify the insured for any loss or damage, notexceeding P61,000.00, caused by fire to its property consisting of stocks,materials and supplies usual to a shirt factory, including furniture, fixtures,machinery and equipment while contained in the ground, second and thirdfloors of the building situated at number 256 Jaboneros St., San Nicolas,Manila, for a period of one year commencing from that date to October 21,1964.

The insured was at the time of the issuance of the policy and is up to this time,a debtor of petitioner in the amount of not less than Eight Hundred ThousandPesos (P800,000.00) and the goods described in the policy were held in trustby the insured for the petitioner under thrust receipts (Record on Appeal, p. 4).

Said policy was duly endorsed to petitioner as mortgagee/ trustor of theproperties insured, with the knowledge and consent of private respondent tothe effect that "loss if any under this policy is payable to the Pacific BankingCorporation".

On January 4, 1964, while the aforesaid policy was in full force and effect, afire broke out on the subject premises destroying the goods contained in itsground and second floors (Record on Appeal, p.5)

On January 24, 1964, counsel for the petitioner sent a letter of demand toprivate respondent for indemnity due to the loss of property by fire under theendorsement of said policy (Brief for Plaintiff-Appellee, pp. 16-17).

On January 28, 1964, private respondent informed counsel for the petitionerthat it was not yet ready to accede to the latter's demand as the former isawaiting the final report of the insurance adjuster, H.H. Bayne AdjustmentCompany (Brief for Plaintiff-Appellee, pp. 17-18).

On March 25, 1964, the said insurance adjuster notified counsel for thepetitioner that the insured under the policy had not filed any claim with it, norsubmitted proof of loss which is a clear violation of Policy Condition No.11, andfor which reason, determination of the liability of private respondent could notbe had (Supra, pp. 19-20).

On April 24, 1964, petitioner's counsel replied to aforesaid letter asking theinsurance adjuster to verify from the records of the Bureau of Customs theentries of merchandise taken into the customs bonded warehouse razed byfire as a reliable proof of loss (Supra, pp. 21-22). For failure of the insurancecompany to pay the loss as demanded, petitioner (plaintiff therein) on April 28,1 964, filed in the court a quo an action for a sum of money against the privaterespondent, Oriental Assurance Corporation, in the principal sum ofP61,000.00 issued in favor of Paramount Shirt Manufacturing Co. (Record on Appeal, pp. 1-36).

On May 25, 1964, private respondent raised the following defenses in itsanswer to wit: (a) lack of formal claim by insured over the loss and (b)premature filing of the suit as neither plaintiff nor insured had submitted anyproof of loss on the basis of which defendant would determine its liability andthe amount thereof, either to the private respondent or its ad . adjuster H.H.Bayne Adjustment Co., both in violation of Policy Condition No.11 (Record on

Appeal, pp. 37-38).

At the trial, petitioner presented in evidence Exhibit "H", which is acommunication dated December 22, 1965 of the insurance adjuster, H.H.

Bayne Adjustment Co. to Asian Surety Insurance Co., Inc., revealingundeclared co-insurances with the following: P30,000.00 with WellingtonInsurance; P25,000. 00 with Empire Surety and P250,000.00 with AsianSurety; undertaken by insured Paramount on the same property covered by itspolicy with private respondent whereas the only co-insurances declared in thesubject policy are those of P30,000.00 with Malayan P50,000.00 with SouthSea and P25.000.00 with Victory (Brief for the Defendant pp. 13-14).

It will be noted that the defense of fraud and/or violation of Condition No. 3 inthe Policy, in the form of non-declaration of co-insurances which was notpleaded in the answer was also not pleaded in the Motion to Dismiss.

At any rate, on June 30, 1967, the trial court denied private respondent'smotion on the ground that the defense of lack of proof of loss or defects thereinwas raised for the first time after the commencement of the suit and that itmust be deemed to have waived the requirement of proof of loss (Sections 83and 84, Insurance Act; Record on Appeal, p. 61).

On September 9, 1967, the case was considered submitted for decision fromwhich order private respondent filed a motion for reconsideration to set thecase or further reception of private respondent's additional evidence, "in orderto prove that 'insured has committed a violation of condition No. 3 of the policyin relation to the other Insurance Clause.' " (Record on Appeal, pp. 61-69).

On September 30,1967, the case was set for the continuation of the hearingfor the reception merely of the testimony of Alejandro Tan Gatue, Manager ofthe Adjustment Co., over the vehement opposition of the petitioner (Record on Appeal, p. 129).

On April 18, 1 968, the trial court rendered a decision adjudging privaterespondent liable to the petitioner under the said contract of insurance, thedispositive portion of which reads:

WHEREFORE, judgment is hereby rendered ordering the defendant to paythe plaintiff P61,000.00, with interest at the rate of 8% per annum fromJanuary 4, 1964, to April 28, 1964, and 12% from April 29, 1964, until theamount is fully paid, P6,100.00, as attorney's fees, and the costs.

SO ORDERED. (Record on Appeal, pp. 140-141)

On appeal, the Court of Appeals reversed the decision of the trial court(Decision promulgated on April 23, 1975, Rollo, pp. 21-33).

Petitioner filed a motion for reconsideration of the said decision of therespondent Court of Appeals, but this was denied on July 3,1975 for lack ofmerit (Rollo, pp. 54-67), resulting in this petition with the following assignederrors;

I

RESPONDENT COURT OF APPEALS COMMITTED A GRAVE ERROR OFLAW IN CONCLUDING FRAUD FROM THE BARE FACT THAT THEINSURED PARAMOUNT PROCURED ADDITIONAL INSURANCES OTHERTHAN THOSE STATED IN THE POLICY IN SPITE OF THE EXISTENCE OFCONTRARY PRESUMPTIONS AND ADMITTED FACT ANDCIRCUMSTANCES WHICH NEGATE THE CORRECTNESS OF SAIDCONCLUSION.

(a) The respondent Court did not consider the legal presumptionagainst the existence of fraud, which should be established with such quantumof proof as is required for any crime.

(b) The record of the case is bereft of proof of such fraud.

(c) The private respondent insurer did not even plead or in anywiseraise fraud as a defense in its answer or motion to dismiss and, therefore, itshould have been considered waived.

(d) The total amount of insurance procured by the insured from thedifferent companies amounted to hardly onehalf (½) of the value of the goodsinsured.

II

RESPONDENT COURT ERRED IN NOT HOLDING THAT CONSIDERINGTHE VOTING ON THE PARTICULAR QUESTION OF FRAUD, THE FINDINGOF THE TRIAL COURT THEREON SHOULD BE CONSIDERED AFFIRMED.

III

THE CONCURRING OPINION OF MR. JUSTICE CHANCO IS LEGALLYERRONEOUS IN HOLDING THAT THE ACTION WAS PREMATURELY

BROUGHT BECAUSE THE REQUIRED CLAIM UNDER THE INSURANCELAW HAS NOT BEEN FILED, NOTWITHSTANDING THE LETTER, (EXHIBIT"C") OF PETITIONER-APPELLANT'S LAWYER WHICH IS A SUBSTANTIALCOMPLIANCE OF THE LEGAL REQUIREMENTS AND NOT HOLDING

8/12/2019 Insurance Meeting 6 Cases

http://slidepdf.com/reader/full/insurance-meeting-6-cases 13/28

MEETING 6 CASES - INSURANCE

13

THAT PRIVATE RESPONDENT INSURER HAD ALREADY WAIVED THESUPPOSED DEFECTS IN THE CLAIM FILED BY PETITIONER-APPELLANTFOR ITS FAILURE TO CALL THE ATTENTION OF THE LAYER TO SUCH ALLEGED DEFECTS AND FOR ENDORSING THE CLAIM TO ITS ADJUSTER FOR PROCESSING.

IV

RESPONDENT COURT OF APPEALS COMMITTED A GRAVE ERROR OFLAW IN NOT INTERPRETING THE PROVISIONS OF THE POLICYLIBERALLY IN FAVOR OF THE HEREIN PETITIONER-APPELLANT, WHOIS NOT THE INSURED BUT ONLY THE ASSIGNEE/MORTGAGEE OF THEPROPERTY INSURED.

V

RESPONDENT COURT OF APPEALS COMMITTED A GRAVE ERROR OFLAW IN DISMISSING THE CASE AND IN NOT AFFIRMING THE APPEALEDDECISION OF THE TRIAL COURT. (Brief for Petitioners, pp. 1-3)

The crux of the controversy centers on two points: (a) unrevealedco-insurances which violated policy conditions No. 3 and (b) failure of theinsured to file the required proof of loss prior to court action. Policy Condition

No. 3 explicitly provides:

3. The Insured shall give notice to the Company of any insurancealready effected, or which may subsequently be effected, covering any of theproperty hereby insured, and unless such notice be given and the particularsof such insurance or insurances be stated in or endorsed on this Policy by oron behalf of the Company before the occurrence of any loss or damage, allbenefit under this policy shall be forfeited. (Record on Appeal, p. 12)

It is not disputed that the insured failed to reveal before the loss three otherinsurances. As found by the Court of Appeals, by reason of said unrevealedinsurances, the insured had been guilty of a false declaration; a clearmisrepresentation and a vital one because where the insured had been askedto reveal but did not, that was deception. Otherwise stated, had the insurerknown that there were many co-insurances, it could have hesitated or plainlydesisted from entering into such contract. Hence, the insured was guilty ofclear fraud (Rollo, p. 25).

Petitioner's contention that the allegation of fraud is but a mere inference orsuspicion is untenable. In fact, concrete evidence of fraud or false declarationby the insured was furnished by the petitioner itself when the facts alleged inthe policy under clauses "Co-Insurances Declared" and "Other InsuranceClause" are materially different from the actual number of co-insurances takenover the subject property. Consequently, "the whole foundation of the contractfails, the risk does not attach and the policy never becomes a contractbetween the parties. Representations of facts are the foundation of thecontract and if the foundation does not exist, the superstructure does not arise.Falsehood in such representations is not shown to vary or add to the contract,or to terminate a contract which has once been made, but to show that nocontract has ever existed (Tolentino, Commercial Laws of the Philippines, p.991, Vol. II, 8th Ed.) A void or inexistent contract is one which has no force andeffect from the very beginning, as if it had never been entered into, and whichcannot be validated either by time or by ratification Tongoy v. C.A., 123 SCRA99 [1983]; Avila v. C.A. 145 SCRA [1986]).

As the insurance policy against fire expressly required that notice should begiven by the insured of other insurance upon the same property, the totalabsence of such notice nullifies the policy (Sta. Ana v. Commercial Union Assurance Co., 55 Phil. 333 [1930]; Union Manufacturing Co., Inc. vs.Philippine Guaranty Co., Inc., 47 SCRA 276 [1972]; Pioneer Ins. & SuretyCorp., v. Yap, 61 SCRA 432 [1974]).

The argument that notice of co-insurances may be made orally ispreposterous and negates policy condition No. 20 which requires every noticeand other communications to the insurer to be written or printed.

Petitioner points out that Condition No. 3 in the policy in relation to the "otherinsurance clause" supposedly to have been violated, cannot certainly defeatthe right of the petitioner to recover the insurance as mortgagee/assignee.Particularly referring to the mortgage clause of the policy, petitioner arguesthat considering the purpose for which the endorsement or assignment wasmade, that is, to protect the mortgagee/assignee against any untoward act oromission of the insured, it would be absurd to hold that petitioner is barredfrom recovering the insurance on account of the alleged violation committedby the insured (Rollo, Brief for the petitioner, pp, 33-35).

It is obvious that petitioner has missed all together the import of subjectmortgage clause which specifically provides:

Mortgage Clause

Loss, if any, under this policy, shall be payable to the PACIFIC BANKINGCORPORATION Manila mortgagee/trustor as its interest may appear, it being

hereby understood and agreed that this insurance as to the interest of themortgagee/trustor only herein, shall not be invalidated by any act orneglect—except fraud or misrepresentation, or arson—of the mortgagor orowner/trustee of the property insured; provided, that in case the mortgagor orowner/ trustee neglects or refuses to pay any premium, the mortgagee/ trustorshall, on demand pay the same. (Rollo, p. 26)

The paragraph clearly states the exceptions to the general rule that insurance

as to the interest of the mortgagee, cannot be invalidated; namely: fraud, ormisrepresentation or arson. As correctly found by the Court of Appeals,concealment of the aforecitedco-insurances can easily be fraud, or in the very least, misrepresentation(Rollo, p. 27).

Undoubtedly, it is but fair and just that where the insured who is primarilyentitled to receive the proceeds of the policy has by its fraud and/ormisrepresentation, forfeited said right, with more reason petitioner which ismerely claiming as indorsee of said insured, cannot be entitled to suchproceeds.

Petitioner further stressed that fraud which was not pleaded as a defense inprivate respondent's answer or motion to dismiss, should be deemed to havebeen waived.

It will be noted that the fact of fraud was tried by express or at least impliedconsent of the parties. Petitioner did not only object to the introduction ofevidence but on the contrary, presented the very evidence that proved itsexistence.

Be that as it may, it is established that the Supreme Court has ample authorityto give beyond the pleadings where in the interest of justice and the promotionof public policy, there is a need to make its own finding to support itsconclusion. Otherwise stated, the Court can consider a fact which surfacedonly after trial proper (Maharlika Publishing Corp. v. Tagle, 142 SCRA 561[1986]).

Generally, the cause of action on the policy accrues when the loss occurs, Butwhen the policy provides that no action shall be brought unless the claim isfirst presented extrajudicially in the manner provided in the policy, the cause ofaction will accrue from the time the insurer finally rejects the claim for payment(Eagle Star Insurance v. Chia Yu, 55 Phil 701 [1955]).

In the case at bar, policy condition No. 11 specifically provides that the insuredshall on the happening of any loss or damage give notice to the company andshall within fifteen (15) days after such loss or damage deliver to the privaterespondent (a) a claim in writing giving particular account as to the articles orgoods destroyed and the amount of the loss or damage and (b) particulars ofall other insurances, if any. Likewise, insured was required "at his ownexpense to produce, procure and give to the company all such furtherparticulars, plans, specifications, books, vouchers, invoices, duplicates orcopies thereof, documents, proofs and information with respect to the claim".(Record on Appeal, pp. 18-20).

The evidence adduced shows that twenty-four (24) days after the fire,petitioner merely wrote letters to private respondent to serve as a notice ofloss, thereafter, the former did not furnish the latter whatever pertinentdocuments were necessary to prove and estimate its loss. Instead, petitionershifted upon private respondent the burden of fishing out the necessaryinformation to ascertain the particular account of the articles destroyed by fireas well as the amount of loss. It is noteworthy that private respondent and itsadjuster notified petitioner that insured had not yet filed a written claim norsubmitted the supporting documents in compliance with the requirements setforth in the policy. Despite the notice, the latter remained unheedful. Since therequired claim by insured, together with the preliminary submittal of relevantdocuments had not been complied with, it follows that private respondentcould not be deemed to have finally rejected petitioner's claim and thereforethe latter's cause of action had not yet arisen. Compliance with condition No.11 is a requirement sine qua non to the right to maintain an action as priorthereto no violation of petitioner's right can be attributable to privaterespondent. This is so, as before such final rejection, there was no realnecessity for bringing suit. Petitioner should have endeavored to file the formalclaim and procure all the documents, papers, inventory needed by privaterespondent or its adjuster to ascertain the amount of loss and after complianceawait the final rejection of its claim. Indeed, the law does not encourageunnecessary litigation (Eagle Star Insurance Co., Ltd., et al. v. Chia Yu, p. 701,supra).<äre||anº•1àw>

Verily, petitioner prematurely filed Civil Case No. 56889 and dismissal thereofwas warranted under the circumstances. While it is a cardinal principle ofinsurance law that a policy or contract of insurance is to be construed liberallyin favor of the insured and strictly as against the insurer company (Eagle StarInsurance Co., Ltd., et al. v. Chia Yu, p. 702, supra; Taurus Taxi Co., Inc. v.

The Capital Ins. & Surety Co., Inc., 24 SCRA 458 [1968]; National PowerCorp. v. CA, 145 SCRA 533 [1986]), yet, contracts of insurance, like othercontracts, are to be construed according to the sense and meaning of theterms which the parties themselves have used. If such terms are clear and

8/12/2019 Insurance Meeting 6 Cases

http://slidepdf.com/reader/full/insurance-meeting-6-cases 14/28

MEETING 6 CASES - INSURANCE

14

unambiguous, they must be taken and understood in their plain, ordinary andpopular sense (Young v. Midland Textile Ins. Co., 30 Phil. 617 [1919]; UnionManufacturing Co., Inc. v. Phil. Guaranty Co., Inc., p. 277 supra; Pichel v. Alonzo, III SCRA 341 [1982]; Gonzales v. CA, 124 SCRA 630 [1983]; GSIS v.CA, 145 SCRA 311 [1986]; Herrera v. Petrophil Corp., 146 SCRA 385 [1986]).

Contracts of insurance are contracts of indemnity upon the terms andconditions specified in the policy. The parties have a right to impose such

reasonable conditions at the time of the making of the contract as they maydeem wise and necessary. The agreement has the force of law between theparties. The terms of the policy constitute the measure of the insurer's liability,and in order to recover, the insured must show himself within those terms. Thecompliance of the insured with the terms of the policy is a condition precedentto the light of recovery (Stokes v. Malayan Insurance Co., Inc., 127 SCRA 766[1984]).

It appearing that insured has violated or failed to perform the conditions underNo. 3 and 11 of the contract, and such violation or want of performance hasnot been waived by the insurer, the insured cannot recover, much less theherein petitioner. Courts are not permitted to make contracts for the parties;the function and duty of the courts is simply to enforce and carry out thecontracts actually made (Young v. Midland Textile Ins. Co., 30 Phil. 617[1915]; Union Manufacturing Co. Inc. v. Phil. Guaranty Co. Inc., p. 276 supra).

Finally, the established rule in this jurisdiction that findings of fact of the Courtof Appeals when supported by substantial evidence, are not reviewable onappeal by certiorari, deserves reiteration. Said findings of the appellate courtare final and cannot be disturbed by the Supreme Court except in certaincases Lereos v. CA, 117 SCRA 395 [1985]; Dalida v. CA, 117 SCRA 480[1982] Director of Lands v. CA, 117 SCRA 346 [1982]; Montesa v. CA, 117SCRA 770 [1982]; Sacay v. Sandiganbayan, 142 SCRA 609 [1986]; Guita v.CA, 139 SCRA 576 [1985]; Manlapaz v. CA, 147 SCRA 238-239 [1987]).

PREMISES CONSIDERED, the petition is DISMISSED for lack of merit, andthe decision appealed from is AFFIRMED. No costs.

SO ORDERED.

Melencio-Herrera, (Chairman), Padilla, Sarmiento and Regalado, JJ., concur.

G.R. No. L-10436 January 24, 1916

FRANCISCA EGUARAS, plaintiff-appellee,vs.THE GREAT EASTERN LIFE ASSURANCE COMPANY, LTD., and WESTG. SMITH, defendants.THE GREAT EASTERN LIFE ASSURANCE COMPANY, LTD., appellant.

Crossfield and O'Brien for appellant.Pedro Guevara for appellee.

TORRES, J.:

This is an appeal filed through bill of exceptions by the counsel for thedefendant, the Great Eastern Life Assurance Company, Ltd., from the judgment of September 14, 1914, whereby the Court of First Instance ofLaguna sentenced it to pay to the plaintiff the sum of P5,000, the value of theinsurance policy in question, with legal interest from April 15, 1913, the datewhen the complaint was filed, and the costs. W.G. Smith was absolved fromthe complaint, and the claim fro damages dismissed, as they were not proven.

On April 14, 1913, counsel for Francisca Eguaras filed a written complaint inthe said Laguna court, alleging as a cause of action that about October 14,1912, her son-in-law Dominador Albay had applied in writing to the defendantinsurance company to insure his life for the sum of P5,000, naming as thebeneficiary in case of his death the plaintiff Francisca Eguaras; that aftercompliance with the requisites and the investigation carried on by thedefendant company, and it had been satisfied concerning the physicalcondition of the applicant, it accepted the application for insurance and onNovember 6, 1912, issued policy No. 5592, Exhibit A, which has been made apart of the complaint, whereby the said insurance company insured the life ofthe said Dominador Albay in the sum of P5,000, payable in the event of hisdeath to Francisca Eguaras; that on December 6, 1912, said policy No. 5592being in force, the insured Dominador Albay, died in the municipality of SantaCruz, Laguna, and despite the fact that the beneficiary submitted satisfactoryproofs of his death and that the defendant company investigated the event,still it refused and continues to refuse to pay to the plaintiff the value of thepolicy, Exhibit A, thereby causing damages estimated at P1,000. The courtwas therefore asked to render judgment against the Great Eastern Life Assurance Company, Ltd., and its general agent, West G. Smith, bysentencing them to pay to the plaintiff the sum of P5,000, the value of policyNo. 5592, plus the sum of P1,000 for damages inflicted upon them, in addition

to the costs of the suit.

The demurrer filed to the foregoing complaint having been overruled, counselfor the insurance company and for West G. Smith replied thereto, admitting

the allegations of the complaint with respect to the legal status of the parties bydenying all the rest, and setting forth in special defense that the insurancepolicy issued in the name of Dominador [Albay] had been obtained throughfraud and deceit known and consented to by the interested parties and istherefore completely illegal, void, and ineffective; wherefore he prayed that thedefendants be absolved from the complaint, with the costs against the plaintiff.

In answer to the reply of the defendants the plaintiff alleged that the grounds

set forth in the special defense had been made the basis of a criminalcomplaint against the plaintiff, Francisca Eguaras, and Ponciano Remegio forthe crime of frustrated estafa in the Court of First Instance of Laguna, but thatthey had been acquitted on said complaint, as is demonstrated by the copy ofthe judgment, marked Exhibit b, which was made an integral part of theanswer, and therefore the plaintiff prayed that the relief sought in hercomplaint be granted.

After trial and examination of the evidence submitted by both parties, the courtrendered the judgment that has been set forth, whereto the defendant, TheGreat Eastern Life Assurance Company, Ltd., saved its exception, and inwriting moved for a reopening of the case and a new trial. This motion wasdenied, with exception on appellant's part, so the corresponding bill ofexceptions was filed, approved and forwarded to the clerk of this court.

The question to be determined in this suit consists in whether the lifeinsurance obtained by Dominador Albay, with the assistance of the insuranceagent, Ponciano Remegio, is legal and valid or whether on the contrary it wasissued through fraud and deceit, and in such case, whether the defendant,The Great Eastern Life Assurance Company, Ltd., is still under obligation topay the value thereof to the plaintiff.

It is demonstrated in the case by evidence submitted by the plaintiff that onOctober 14, 1912, through the efforts of the defendant company's agent,Ponciano Remigio, Dominador Albay got the insurance company to insure hislife for the sum of P5,000 and that through the representations and statementsmade by said Dominador Albay in his application and the favorable medicalexamination made by Dr. Jose A. Vidal (record, p. 126), the company agreedto the life insurance sought, and on November 6,. 1912, issued the policy No.5592, the value whereof was payable to the insured's mother-in-law,Francisca Eguaras. One month after said insurance policy had been issued,that is, on December 6, 1912, the insured Dominador Albay died in themunicipality of Santa Cruz, Laguna, of intestinal occlusion, according to thecertificate of Dr. R. Kamatoy, after an illness of three days, wit medicalattendance. (Exhibit B, p. 154; Exhibit B, criminal case No. 2616.) Thedefendant company, according to the declaration of its own agent in theseIslands, despite having received satisfactory proofs of the death of theinsured, refused to pay the amount of the insurance, alleging that it had beensecured through fraud and deceit and was therefore illegal and void.

The contract of life insurance executed between The Great Eastern Life Assurance Company, Ltd., and Dominador Albay is set forth in the policy itselfand in the original and supplementary applications signed apparently byDominador Albay, it appearing to have been stipulated that "This insurance isgranted in consideration of the foregoing statements and agreement in theapplication presented to obtain this policy, which application forms a part ofthe present contract." This condition is repeated in Clause VIII of theconditions and the privileges granted to the insured, that "This policy and theapplication presented to secure it, taken together, constitute the wholecontract, which cannot be altered except in writing by the general manager orsome person expressly appointed therefor by the board of directors."

Now then, in the supplementary application presented by the insured onOctober 14, 1912, to the question: "Do you think that you are free from diseaseand that you have a good constitution?" he answered: "Yes;" and to anotherquestion: "Have you suffered from any affection of ... (c) Chest— Cough,asthma, spitting blood, pleurisy?" the applicant answered: "No." (p. 126.)

The physician of the insurance company in charge of the physical examinationof the person applying for insurance in Laguna, Dr. Jose A. Vidal, made thephysical examination of the person who presented himself to him asDominador Albay and in his report to the said company (pp. 126-127), datedOctober 14, 1912, he recorded that the development, expansion, percussion,and auscultation of the applicant's chest were "normal" and recommended tothe company that it could "take the risk" of insuring the applicant Dominador Albay and on said professional report a certain Lunn, who must be the medicalofficer of the company, placed his O.K. On the basis of these medical reportsand of the exact and faithful performance of its obligations on the part of theinsurance agent, Ponciano Remigio, The Great Eastern Life AssuranceCompany issued the corresponding policy in favor of Dominador Albay,insuring his life for P5,000. The first premium, amounting to P82.25, was paidon November 6, 1912, and while said policy was in full force and effect thedeath of the death of the insured Albay occurred the next month.

It appears from the record that the insured had knowledge of the false repliedcontained in the two applications for insurance and knowing permitted fraud tobe practised upon the insurance company, for in his acknowledgment andconsent his mother-in-law was designated as the beneficiary of the insurance,

8/12/2019 Insurance Meeting 6 Cases

http://slidepdf.com/reader/full/insurance-meeting-6-cases 15/28

MEETING 6 CASES - INSURANCE

15

despite the fact that he had children and his mother was still living. In thepresent case the fraud consisted in the fact that a healthy and robust personwas substituted in place of insured invalid when Dr. Vidal made the physicalexamination of the one who seeking to be insured, for the real person whodesired to be insured and who ought to have been examined was in bad healthon and before the date of executing the insurance contract of which facts theinsured Dominador Albay and the insurance agent Ponciano Remigio had fullknowledge.

The insurance company endeavored to prove by means of cross-examinationof Ponciano Remigio, while he was testifying as a witness for the plaintiff, andby means of the declaration of another insurance agent, Jose D. Arce, thatsaid Ponciano Remigio had always been in the habit of securing the insuranceof sick persons, who died shortly after it was issued, in fraud and to the seriousinjury of the defendant company; but at the request of the opposition party thecourt overruled this attempt of the defendant and did not permit proof ofspecific fraudulent acts performed by its agent Remegio. It is to be observedthat the said Remegio has already been convicted of the crime of estafa andsentenced to two months and one day of arresto and to the restitution of P20to The Insular Life Insurance Company, as stated in the copy of the sentence.(Record, p. 158.)

It is unquestionable that the person who on October 14, 1912, presented

himself to Dr. Vidal to be examined under the name of Dominador Albay, andwho signed the supplementary application before said physician, was not thereal Dominador Albay, who died on December 6, 1912. In case No. 2616,prosecuted against Ponciano Remegio, Castor Garcia, and FranciscaEguaras for frustrated estafa, part of which was exhibited at the hearing in thissuit, Dr. Vidal testified that on October 14, 1912, while he was in the employ ofthe defendant company as a physician, he proceeded to examine, in SanPablo, Laguna, a person presented to him by the insurance agent, PoncianoRemegio, who said that such person was named Dominador Albay anddesired to insured; that after he had conducted the examination for the spaceof about an hour the person examined by him signed the supplementaryapplication (Exhibit F) with the name of Dominador Albay; that the personwhom he examined and who signed the application with the name ofDominador Albay, if he were not mistaken, was the individual he saw beforehim, the accused Castor Garcia. When he saw urged to state positivelywhether he had any doubt that the person he had examined was the accusedCastor Garcia, he first asked permission to examine the latter's body, andfinally reaffirmed that, judging from the general appearance of the accused,Castor Garcia, the latter was the very person he had examined in San Pabloand that he had assured Major Grove of the Constabulary and the attorney ofthe defendant company that said Garcia was the person who had presentedhimself to him, saying that his name was Dominador Albay. He further statedthat about March 24, 1913, Ponciano Remegio had visited his house in Manilato request that he should testify in favor of said Remegio, who at the sametime had offered him P600 not to identify the person of Castor Garcia at thetrial. Major Grove of the Constabulary affirmed under oath at the trial of thesame case that on April 4, 1913, when Dr. Vidal and the accused CastorGarcia were in his house, Dr. Vidal had told him that he had not the least doubtthat Castor Garcia was the person whom he had examined in San Pablo.

Attorney O'Brien, among other things, stated in a sworn declaration, thatPonciano Remegio interviewed him in his office about March 15, 1914, tellinghim that the signatures affixed to the original application for insurance and thesupplementary application signed before Dr. Vidal at the time of the physicalexamination were false, and then indicated where he could get documentswith authentic signatures of the said Albay. Remegio further told him that he(Remegio) was disgusted with his accomplices because they could not reachan agreement regarding the distribution among them of the amount of thepolicy when it should be collected. All the statements of said Remegio weremade before him under oath as a notary public in the presence of Jose D. Arce, which statements were annotated in the memorandum, Exhibit 3, hebeing unable to draw up a formal document for signature as the day of theinterview was Sunday and he had no stenographer in his office. Jose D. Arcecorroborated the statements of the foregoing witness, and added that as thesaid Ponciano Remegio lodged in his house, the latter had told him the detailsof the substitution of another person in place of Dominador Albay at theexamination made by the physician of the company, and that the cedulas ofsaid Albay and two letters (pp. 171, 173), in which authentic signatures of Albay appeared, were delivered to Attorney O'Brien by Albay's mother, namedManuela Flores. Captain Barrows of the Constabulary testified how PoncianoRemegio had promised O'Brien in a conference held by the three in his housein Santa Cruz some ten days before the trial, that on the day of the trial hewould testify that the person who had signed the applications with the name ofDominador Albay was Castor Garcia, who was then outside of the PhilippineIslands (p. 35). It is true that said Remegio denied all this in his testimonygiven at the trial in favor of the plaintiff, but it is to be observed that the saidRemegio in March, 1914, told Attorney O'Brien in his office in Manila that theperson who had signed the insurance applications had left the Philippines, butafterwards he stated to said O'Brien and Barrows that the person mentioned

was Castor Garcia, and it the said Remegio did not so testify at the trial it wasthrough fear of being prosecuted for perjury.

Dr. Getrudo Reyes stated at the trial that in March, 1912, he had beenconsulted by Dominador Albay regarding the cough he had and after amedical examination witness had reached the conclusion that the personconcerned was suffering from tuberculosis in the first stage, although it doesnot appear that said physician made a microscopic analysis of the patient'ssputum; but there is circumstantial evidence in the case that the saidDominador Albay died of tuberculosis, for his own mother, Manuela Flores, soaffirmed in the affidavit (p. 199) drawn up before a notary on April 17, 1914,

although said affidavit was not admitted as evidence because she repudiatedits contents in the courts. The motive for this change of front on the part of thesaid Manuela Flores seems to have been due to the fact that the insurancecompany was unwilling to give her and her husband money for the statementsthey would make in the court, for in the letter (Exhibit 9) of Leodegario v.Lambonga, Manuela Flores' husband, addressed to Jose D. Arce on August26, 1914, Lambonga informed Arce that they would not appear the next day,not saying where, because they first wanted to agree upon the sum that theywould receive in the event Smith, defendant's agent, should win the case, andaccordingly it execute "an instrument we can hold to" (literal). Jose Valenciatestified that on November 27, 1912, he went to the municipal building to signa declaration in the name Dominador Albay because the latter was then ill andcould not leave his house— a fact admitted by plaintiff. Attorney O'Brien alsotestified that Ponciano Remegio had assured him that Dominador Albay wassuffering from tuberculosis, and also that Remegio had told him that there was

a physician in Santa Cruz, who must have been Dr. Reyes, that could attestthat said Albay was really affected with tuberculosis.

It is immaterial that Albay may have died of intestinal occlusion, as Dr.Kamatoy affirms in the death certificate (p. 154), because said aliment doesnot demonstrate that Albay was not suffering from some other chronicdisease; or that in the month of October, 1912, when he applied for insuranceon his life, he was not affected by malady that would have been sufficientcause for his rejection by the physician of the insurance company.

To secure the insurance on the life of Dominador Albay, the parties interestedused a person who signed the name of Dominador Albay as the insured in allthe documents connected with his application, for the signatures to the letterand the document relating to the insurance, exhibited at the trial, and signedby "Dominador Albay" (pp. 50, 54), are different from the authentic signaturesof the real Dominador Albay which appear in the official documents and theinstruments of conveyance of reality. (Exhibits 12, 13, 15, pp. 207-211 of therecord.) The signatures on these three documents of an official nature, as wellas those on the letters (Exhibits 7 and 8; pp. 171, 173) addressed by the said Albay to his sister Odang and his relative Lambonga, although not admitted asevidence because they were not duly identified are the genuine signatures ofthe real Dominador Albay, who was accustomed to fashion his letters in theSpanish style of handwriting. The documents exhibited under the letters D, E,F, and G by the plaintiff, which bear the signatures of "Dominador Albay,"appear to have been signed by the same hand, and therefore it is not strangethat the signatures on these documents are similar, for it is to be observed thatthe characters in these signatures are firm and strong, demonstrating that theperson who made them had learned to write in American schools where thestyle of handwriting taught is very different from the Spanish.

Plaintiff's claim is based especially on the genuineness of the signatures of"Dominador Albay" in the elector's oath (Exhibit G, p. 124), which was signedbefore the board of election inspectors on May 4, 1912, and duly identified bythe chairman of said board, Proceso Maximo, on the contention that if thesignature on said elector's oath is genuine, those which appear on theinsurance applications, Exhibits E and F (pp. 125-127), and that affixed to theletter, Exhibit D (p. 50), addressed by said Albay in November, 1912, to theinsurance also be true and genuine. But if the real Dominador Albay wrote inthe Spanish style in the months of January, February, and March of the year1912, as demonstrated by the signatures affixed to the documents Exhibits 7,8, and 15, it is impossible to believe that he should have radically changed hisform of writing, two months later by adopting a different handwriting, as can beseen in the alleged signature, said to be authentic, in the elector's oath, ExhibitG, written on May 4, 1912, and subsequently imitated in the months of Octoberand November of the same year in the Exhibits E, F, and D. The signaturesthat appear on the papers referring to the insurance are so different from thosewhich appear on the other documents which unquestionably bear thesignature of the real Dominador Albay that, in consideration of the short timewhich elapsed between the last genuine signature in March, 1912, when hesold a tract of land, and his oath as elector in May of the same year 1912, andthe great difference that exist between the two signatures, we can do no lessthan reach the conclusion already stated that there was a person who passedhimself off as Dominador Albay and said person was the one who went tosigning the documents relating to the alleged insurance of Dominador Albaywho died on December 6, 1912.

Moreover, Dominador Albay's age, according to the alleged insuranceapplication and the insurance policy, was 40 years in 1912, while according tohis personal cedulas he was only 32 years of age in 1911, so that when he

was insured he must have been only 33.

It is therefore proven that the signatures on the insurance applications reading"Dominador Albay" are false and forged; that the person who presented

8/12/2019 Insurance Meeting 6 Cases

http://slidepdf.com/reader/full/insurance-meeting-6-cases 16/28

MEETING 6 CASES - INSURANCE

16

himself to Dr. Vidal to be examined was not the real Dominador Albay, butanother different person; that at the time of the application for insurance andthe issuance of the policy which is the subject matter of this suit the realDominador Albay was informed of all those machinations, wherefore it is plainthat the insurance contract between the defendant and Dominador Albay isnull and void because it is false, fraudulent and illegal.

Article 1269 of the Civil Code states:

There is deceit when by words or insidious machinations on the part of one ofthe contracting parties the other is induced to execute a contract which withoutthem he would not have made.

It is essential to the nature of the deceit, to which the foregoing article refers,that said deceit be prior to or contemporaneous with the consent that is anecessary requisite for perfecting the contract, but not that it may haveoccurred or happened thereafter. A contract is therefore deceitful, for theexecution whereof the consent of one of the parties has been secured bymeans of fraud, because he was persuaded by words or insidiousmachinations, statements or false promises, and a defective consent wrungfrom him, even though such do not constitute estafa or any other criminalsubject to the penal law.

The defendant company accepted the application for insurance made byDominador Albay and executed the contract comprised under articles 416 ofthe Code of Commerce, although for the perfecting thereof the insured, Albay,as he was not in good health, by connivance with the insurance company'sagent, presented Castor Garcia to the physician Vidal, who wascommissioned by the company to examine applicants for li fe insurance and inview of the favorable report of the said physician, who reported and certifiedthat the person examined by him under the name of Dominador Albay was ingood health and possessed the qualifications required by said insurancecompany for perfecting the contract, so the company freely and willinglyconsented to the execution thereof, effectively induced thereto by the result ofthe medical examination and of the favorable professional report issued inview of the appearance of an individual who was in good health, but differentfrom the individual who was seeking to be insured and who died one monthand twenty-three days after the insurance had been granted.

The fraud which gave rise to the mistaken consent, given by the defendantcompany to the application for insurance made by Albay and to the executionof the contract through deceit, is plain and unquestionable. This fraudconsisted in the substitution at the examination of Castor Garcia in place of theinsured Dominador Albay, and as the deceit practiced in the said contract is ofa serious nature, the same is ipso facto void and ineffective, in accordancewith the provisions of article 1270 of the Civil Code.

If there had been no substitution, if the insured Dominador Albay had been theperson who appeared and was examined by the physician Vidal, said Albaybeing manifestly different from Castor Garcia, the said physician would nothave affirmed at the trial that it was Garcia who presented himself for thephysical examination, accompanied by the insurance company's agent, at hisresidence in San Pablo, and he would have failed to recognize him when hesaw him in the court, nor is any mistake on the physician's part possible as theinspection and physical examination of the individual lasted for something likethe space of an hour.

The supposition that Dominador Albay was not ill in October, 1912, would notexplain why he did not present himself in person to be examined by thephysician Vidal; and when he failed to do so and by agreement with the agentRemegio was willing to be substituted by Castor Garcia to the end that in anyevent no defect or personal quality should be discovered to hinder theperfecting of the insurance contract, such a change in the person constitutesone of the means of fraud which, although it may not partake of the nature of acrime, essentially nullifies the insurance contract executed.

With this array of circumstantial evidence derived from facts duly proven as aresult of the present suit, we get, if not a moral certainly, at least a fullconviction that when Castor Garcia presented himself to be examined by thephysician Vidal in place of Dominador Albay, serious deceit occurred inperfecting the insurance contract, for had the agent of the company not beendeceived it would not have granted the insurance applied for by Albay, norwould it have executed the contract by virtue of whereof payment is claimed ofthe value of policy obtained through fraud; and consequently on suchassumptions it is improper, nor is it permitted by the law, to order collection ofthe amount claimed.

With reference to the effect produced by the final judgment rendered in the caefor estafa in connection with this suit, it is unquestionable that said judgmentdoes not give rise to the presumption of res adjudicata, applicable to thepresent case (art. 1252, Civil Code), nor does it constitute an estoppel to thematters litigated in the said criminal case for estafa and consequently there

cannot be applied in the present suit the principle laid down in the decision ofPeñalosa vs. Tuason (22 Phil. Rep., 303), for the reason that said case wasinstituted by virtue of an information on the charge that the deceitful actsexecuted by the company's agent and others interested in the result of the

fraud constitute the crime of estafa to the injury of the said insurancecompany, even though the court acquitted the accused on account of the lackof satisfactory proof of the acts ascribed to them and of their guilt, while theexception taken by the defendant company is based on the nullity of theinsurance contract because deceit occurred in the perfecting and executionthereof.

In view of that acquittal the beneficiary of the insurance, Francisca Eguaras,

instituted the present suit against The Great Eastern Life AssuranceCompany, Ltd., claiming payment of the sum of P5,000, the value of policy No.5592, Exhibit A, which claim the defendant opposed with the contention thatsaid policy was void and illegal because it had been obtained by means ofdeceit and fraud.

The judgment of acquittal rendered in the criminal case for estafa against thesaid Francisca Eguaras does not produce the effect of res adjudicata in thepresent suit to the extent that because she was acquitted of the crime of estafashe has necessarily acquired as a plaintiff the right to collect the value of theinsurance, or that the insurance company cannot contend that the insurancecontract is null and void because it was executed by means of deceit, whichupon being proven, as it has been in this case, invalidates the contract thatgave rise to the obligation to pay the value of said policy.

In the said criminal case the question raised was whether the acts performedby Eguaras and her co-accused partook of the nature of the crime of estafa,and when it was decided in the negative, the said Eguaras was not thereforeunquestionably entitled to collect the value of the insurance, for after deceithad once been proven in the contract, no obligation rested upon the insurancecompany to pay the sum stipulated.

In the present civil suit it is not a question whether the acts performed byEguaras and others interested in the proceeds of the insurance were criminal,but whether in taking out the insurance on the life of Dominador Albay thereoccurred in the operation deceit and fraud of a civil nature, in the form andunder the conditions defined by the Civil Code.

In a contract executed with the requisites fixed in article 1261, one of thecontracting parties may have given his consent through error, violence,intimidation, or deceit, and in any of such cases the contract is void, eventhough, despite this nullity, no crime was committed. (Article 1265, Civil Code.)There may not have been estafa in the case at bar, but it was conclusivelydemonstrated by the trial that deceit entered into the insurance contract,fulfillment whereof is claimed, and therefore the conclusions reached by thecourt in the judgment it rendered in the criminal proceedings for estafa do notaffect this suit, nor do they influence the decision proper herein, nor can theyproduce in the present suit, over the exception of the defendant, the force ofres adjudicata.

For all the foregoing reasons the first part of the judgment appealed from, withreference to the payment of P5,000 to the plaintiff, must be reversed and thedefendant, The Great Eastern Life Assurance Company, Ltd., absolved fromthe complaint, as we do absolve it; and we affirm the second part of said judgment in so far it absolves W. G. Smith and dismissed the petition fordamages; without special finding as to the costs in both instances. So ordered.

Arellano, C.J., Johnson. Carson, Moreland and Trent, JJ. concur.

INSURANCE CASE DIGEST: QUA CHEE GAN V. LAW UNION AND ROCKINSURANCE CO., LTD. (1955)G.R. NO.L-4611 DECEMBER 17, 1955LESSONS APPLICABLE: AMBIGUOUS PROVISIONS INTERPRETEDAGAINST INSURER (INSURANCE)

FACTS:Qua Chee Gan, a merchant of Albay, owned four bodegas which he insuredwith Law Union & Rock Insurance Co., Ltd (Law Union) since 1937 and thelose made payable to the Philippine National Bank (PNB) as mortgage of thehemp and crops, to the extent of its interestJuly 21, 1940 morning: fire broke out in bodegas 1,2 and 4 which lasted foralmost a week.Qua Chee Gan informed Law Union by telegramLaw Union rejected alleging that it was a fraudulent claim that the fire hadbeen deliberately caused by the insured or by other persons in connivancewith himQue Chee Gan, with his brother, Qua Chee Pao, and some employees of his,were indicted and tried in 1940 for the crime of arson but was subsequentlyacquittedDuring the pendency of the suit, Que Chee Gan paid PNBLaw Union states that ff. assignment of errors:1. memo of warranty requires 11 hydrants instead of 2

2. violation of hemp warranty against storage of gasoline since it prohibits oils3. fire was due to fraud4. burned bodegas could not possibly have contained the quantities of copraand hemp stated in the fire claims

8/12/2019 Insurance Meeting 6 Cases

http://slidepdf.com/reader/full/insurance-meeting-6-cases 17/28

MEETING 6 CASES - INSURANCE

17

ISSUE: W/N Qua Chee Gan should be allowed to claim.

HELD: YES. Affirmed.1. It is a well settled rule of law that an insurer which with knowledge of factsentitling it to treat a policy as no longer in force, receives and accepts a preiumon the policy, estopped to take advantage of the forfeiture2. oils (animal and/or vegetable and/or mineral and/or their liquid productshaving a flash point below 300o Fahrenheit", and is decidedly ambiguous and

uncertain; for in ordinary parlance, "Oils" mean "lubricants" and not gasoline orkeroseneby reason of the exclusive control of the insurance company over the termsand phraseology of the contract, the ambiguity must be held strictly against theinsurer and liberraly in favor of the insured, specially to avoid a forfeiture3. trial Court found that the discrepancies were a result of the insured'serroneous interpretation of the provisions of the insurance policies and claimforms, caused by his imperfect knowledge of English, and that themisstatements were innocently made and without intent to defraud.4. Similarly, the 20 per cent overclaim on 70 per cent of the hemo stock, wasexplained by the insured as caused by his belief that he was entitled to includein the claim his expected profit on the 70 per cent of the hemp, because thesame was already contracted for and sold to other parties before the fireoccurred

G.R. No. L-9370 March 31, 1915

K. S. YOUNG, plaintiff-appellee,vs.THE MIDLAND TEXTILE INSURANCE COMPANY, defendant-appellant.

Bruce, Lawrence, Ross and Block for appellant.Thos D. Aitken for appellee.

JOHNSON, J.:

The purpose of the present action is to recover the sum of P3,000 upon aninsurance policy. The lower court rendered a judgment in favor of the plaintiffand against the defendant for the sum of P2,708.78, and costs. From that judgment the defendant appealed to this court.

The undisputed facts upon which said action is based are as follows:

1. The plaintiff conducted a candy and fruit store on the Escolta, in the city ofManila, and occupied a building at 321 Calle Claveria, as a residence andbodega (storehouse).

2. On the 29th of May, 1912, the defendant, in consideration of the payment ofa premium of P60, entered into a contract of insurance with the plaintiff (policyNo. 509105) by the terms of which the defendant company, upon certainconditions, promised to pay to the plaintiff the sum of P3,000, in case saidresidence and bodega and contends should be destroyed by fire.

3. On the conditions of said contract of insurance is found in "warranty B" andis as follows: "Waranty B.— It is hereby declared and agreed that during thependency of this policy no hazardous goods stored or kept for sale, and nohazardous trade or process be carried on, in the building to which thisinsurance applies, or in any building connected therewith."

4. On the 4th or 5th of February, 1913, the plaintiff placed in said residenceand bodega three boxes, 18 by 18 by 20 inches measurement, whichbelonged to him and which were filed with fireworks.

5. On the 18th day of March, q913, said residence and bodega and thecontents thereof were partially destroyed by fire.

6. Said fireworks had been given to the plaintiff by the former owner of theLuneta Candy Store; that the plaintiff intended to use the same in thecelebration of the Chinese new year; that the authorities of the city of Manilahad prohibited the use of fireworks on said occasion, and that the plaintiff thenplaced the same in said bodega, where they remained from the 4th or 5th ofFebruary, 1913, until after the fire of the 18th of March, 1913.

7. Both of the parties agree that said fireworks come within the phrase"hazardous goods," mentioned in said "warranty B" of the policy.

8. That said fireworks were found in a part of the building not destroyed by thefire; that they in no way contributed to the fire, or to the loss occasionedthereby.

The only question presented by the parties is whether or not the placing of saidfireworks in the building insured, under the conditions above enumerated, theybeing "hazardous goods," is a violation of the terms of the contract of

insurance and especially of "warranty B." "Warranty B" provides that "nohazardous goods be stored" in the building insured. It is admitted by bothparties that the fireworks are "hazardous goods." The defendant alleged thatthey were "stored." The plaintiff contends that under all the facts and

circumstances of the case, they were not "stored" in said building, and that theplacing of them in the building was not a violation of the terms of the contract.Both the plaintiff and defendant agree that if they were "hazardous goods,"and if they were "stored," then the act of the plaintiff was a violation of theterms of the contract of insurance and the defendant was justified inrepudiating its liability thereunder.

This leads us to a consideration of the meaning of the accord "stored" as used

in said "warranty B." While the word "stored" has been variously defined byauthors, as well as by courts, we have found no case exactly analogous to thepresent. The plaintiff says that he placed said f ireworks in the bodega after hehad been notified that he could not use them on the Chinese new year, inorder that he might later send them to a friend in the provinces. Whether aparticular article is "stored" or not must, in some degree, depend upon theintention of the parties. The interpretation of the word "stored" is quite difficult,in view of the many decisions upon the various conditions presented. Nearlyall of the cases cited by the lower court are cases where the article was beingput to some reasonable and actual use, which might easily have beenpermitted by the terms of the policy, and within the intention of the parties, andexcepted from the operation of the warranty, like the present. Said decisionare upon cases like:

1. Where merchants have had or kept the "hazardous" articles in small

quantities, and for actual daily use, for safe, such as gasoline, gunpowder,etc.;

2. Where such articles have been brought on the premises for actual usethereon, and in small quantities, such as oil, paints, etc; and

3. Where such articles or goods were used for lighting purpose, and in smallquantities.

The author of the Century Dictionary defines the world "store" to be a depositin a store or warehouse for preservation or safe keeping; o place in awarehouse or other place of deposit for safe keeping. See also the definitionsgiven by the Standard Dictionary, to the same effect.

Said definitions, of course, do not include a deposit in a store, in smallquantities, for daily use. "Daily use" precludes the idea of a deposit forpreservation or safe keeping, as well as a deposit for future consumption, orsafe keeping.

In the present case no claim is made that the "hazardous goods" were placedin the bodega for present or daily use. It is admitted that they were placed inthe bodega "for future use," or for future consumption, or for safe keeping. Theplaintiff makes no claim that he deposited them there with any other idea than"for future use"— for future consumption. It seems clear to us that the"hazardous goods" in question were "stored" in the bodega, as that word isgenerally defined. That being true, suppose the defendant had made anexamination of the premises, even in the absence of a fire, and had found he"hazardous goods" there, under the conditions above described, would it nothave been justified, then and there, in declaring the policy null and of no effectby reason of a violation of its terms on he par of the plaintiff? If it might, thenmay it no repudiate is liability, even after the fire? If the "warranty" is a term ofthe contract, will not its violation cause a breach and justify noncompliance ora repudiation?

Contracts of insurance are contracts of indemnity upon the terms andconditions specified in the policy. The parties have a right to impose suchreasonable conditions at the time of the making of the contract as they maydeem wise and necessary. The rate of premium is measured by the characterof the risk assumed. The insurance company, for a comparatively smallconsideration, undertakes to guarantee the insured against loss or damage,upon the terms and conditions agreed upon, and upon no other, and whencalled upon to pay, in case of loss, the insurer, therefore, may justly insist upona fulfillment of these terms. If the insured cannot bring himself within theconditions of the policy, he is not entitled to recover for the loss. The terms ofthe policy constitute the measure of the insurer's liability, and in order torecover the insured must show himself within those terms; and if it appearsthat the contract has been terminated by a violation, on the part of the insured,of its conditions, then there can be no right of recovery. The compliance of theinsured with the terms of the contract is a condition precedent to the right ofrecovery. If the insured has violated or failed to perform the conditions of thecontract, and such a violation or want of performance has not been waived bythe insurer, then the insured cannot recover. Courts are not permitted to makecontracts for the parties. The function and duty of the courts consist simply inenforcing and carrying out he contracts actually made. While it is true, as ageneral rule, that contracts of insurance are construed most favorably to theinsured, yet contracts of insurance, like other contracts, are to be construedaccording to the sense and meaning of the terms which the parties themselveshave used. If such terms are clear and unambiguous they must be taken andunderstood in their plain, ordinary and popular sense. (Imperial Fire Ins. Co.

vs. County of Coos, 151 U. S., 542; Kyte vs. Commercial Union AssuranceCo., 149 Mass., 116, 122.) The conditions of contracts of insurance, whenplainly expressed in a policy, are binding upon the parties and should beenforced by the courts, if the evidence brings the case clearly within their

8/12/2019 Insurance Meeting 6 Cases

http://slidepdf.com/reader/full/insurance-meeting-6-cases 18/28

MEETING 6 CASES - INSURANCE

18

meaning and intent. It tends to bring the law itself into disrepute when, byastute and subtle distinctions, a plain case is attempted to be taken without theoperation of a clear, reasonable, and material obligation of the contract. (Mackvs. Rochester German Ins. Co., 106 N. Y., 560, 564.)

The appellant argues, however, that in view of the fact that the "storing" of thefireworks on the premises of the insured did not contribute in any way to thedamage occasioned by the fire, he should be permitted to recover— that the

"storing" of the "hazardous goods" in no way caused injury to the defendantcompany. That argument, however, is beside the question, if the "storing" wasa violation of the terms of the contract. The violation of the terms of thecontract, by virtue of the provisions of the policy itself, terminated, at theelection of either party, he contractual relations. (Kyte vs. Commercial Union Assurance Co., 149 Mass., 116, 122.) The plaintiff paid a premium basedupon the risk at the time the policy was issued. Certainly it cannot be deniedthat the placing of the firecrackers in the building insured increased the risk.The plaintiff had not paid a premium based upon the increased risk, neitherhad the defendant issued a policy upon the theory of a different risk. Theplaintiff was enjoying, if his contention may be allowed may be allowed, thebenefits of an insurance policy upon one risk, whereas, as a matter of fact, itwas issued upon an entirely different risk. The defendant had neither beenpaid nor had issues a policy to cover the increased risk. An increase of riskwhich is substantial and which is continued for a considerable period of t ime, is

a direct and certain injury to the insurer, and changes the basis upon which thecontract of insurance rests. (Kyte vs. Commercial Union Assurance Co.(supra); Frost's Detroit Lumber Works vs. Millers' Mutual Ins. Co., 37 Minn.,300, 302; Moore vs. Phoenix Ins. Co., 62 N. H., 240; Ferree vs. Oxford Fire &Life Ins. Co., 67 Pa. State, 373.)

Therefore and for the foregoing reasons, the judgment of the lower court ishereby revoked and the defendant is hereby relieved from any responsibilityunder said complaint, and, without any finding as to costs, it is so ordered.

Arellano, C.J., Torres, Carson, Trent and Araullo, JJ., concur.Moreland, J., concurs in the result.

G.R. No. L-36232 December 19, 1974

PIONEER INSURANCE AND SURETY CORPORATION,petitioner-appellant,vs.OLIVA YAP, represented by her attorney-in-fact, CHUA SOON POONrespondent-appellee.

Eriberto D. Ignacio for petitioner-appellant.

Paculdo, Miranda, Marquez, Sibal & Associates for respondent-appellee.

FERNANDEZ, J.:p

This is an appeal by certiorari from the decision of the Court of Appeals datedDecember 16, 1972, in CA-G.R. No. 36669-R, affirming the judgment of theCourt of First Instance of Manila (Branch VI) in Civil Case No. 54508, whichlatter court declared plaintiff Oliva Yap, herein respondent, entitled to recoverfrom defendant Pioneer Insurance & Surety Corporation, herein petitioner, thefull amount of the damage inquired in Policy No. 4219, which is P25,000.00,plus 12% of said sum from the date of filing of the complaint until full payment,in addition to the sum of P6,000.00 for attorney's fees, and costs.

Respondent Oliva Yap was the owner of a store in a two-storey buildinglocated at No. 856 Juan Luna Street, Manila, where in 1962 she sold shoppingbags and footwear, such as shoes, sandals and step-ins. Chua Soon PoonOliva Yap's son-in-law, was in charge of the store.

On April 19, 1962, respondent Yap took out Fire Insurance Policy No. 4216from petitioner Pioneer Insurance & Surety Corporation with a face value ofP25,000.00 covering her stocks, office furniture, fixtures and fittings of everykind and description. Among the conditions in the policy executed by theparties are the following:

The Insured shall give notice to the Company of any insurance or insurancesalready effected, or which may subsequently be effected, covering any of theproperty hereby insured, and unless such notice be given and the particularsof such insurance or insurances be stated in, or endorsed on this Policy by oron behalf of the Company before the occurrence of any loss or damage, allbenefits under this Policy shall be forfeited. (emphasis supplied)

It is understood that, except as may be stated on the face of this policy there isno other insurance on the property hereby covered and no other insurance isallowed except by the consent of the Company endorsed hereon. Any false

declaration or breach or this condition will render this policy null and void.

At the time of the insurance on April 19, 1962 of Policy No. 4219 in favor ofrespondent Yap, an insurance policy for P20,000.00 issued by the Great

American Insurance Company covering the same properties was noted onsaid policy as co-insurance (Annex "1-E"). Later, on August 29, 1962, theparties executed Exhibit "1-K", as an endorsement on Policy No. 4219, stating:

It is hereby declared and agreed that the co-insurance existing at presentunder this policy is as follows: P20,000.00— Northwest Ins., and not asoriginally stated. (emphasis supplied)

Except as varied by this endorsement, all other terms and conditions remainunchanged.

Still later, or on September 26, 1962, respondent Oliva Yap took out anotherfire insurance policy for P20,000.00 covering the same properties, this timefrom the Federal Insurance Company, Inc., which new policy was, however,procured without notice to and the written consent of petitioner PioneerInsurance & Surety Corporation and, therefore, was not noted as aco-insurance in Policy No. 4219.

At dawn on December 19, 1962, a fire broke out in the building housingrespondent Yap's above-mentioned store, and the said store was burned.Respondent Yap filed an insurance claim, but the same was denied inpetitioner's letter of May 17, 1963 (Exhibit "G"), on the ground of "breachand/or violation of any and/or all terms and conditions" of Policy No. 4219.

On July 17, 1963, Oliva Yap filed with the Court of First Instance of Manila thepresent complaint, asking, among others, for payment of the face value of herfire insurance policy. In its answer, petitioner alleged that no propertybelonging to plaintiff Yap and covered by the insurance policy was destroyedby the fire; that Yap's claim was filed out of time; and that Yap took out aninsurance policy from another insurance company without petitioner'sknowledge and/or endorsement, in violation of the express stipulations inPolicy No. 4219, hence, all benefits accruing from the policy were deemedforfeited.

As already stated at the beginning of this opinion, the trial court decided forplaintiff Oliva Yap; and its judgment was affirmed in full by the Court of Appeals.

The vital issue in this appeal is whether or not petitioner should be absolvedfrom liability on Fire Insurance Policy No. 4219 on account of any violation byrespondent Yap of the co-insurance clause therein. In resolving this problem,the Court of Appeals stated in its decision:

5. The plaintiff-appellee has not violated the other insurance clause(Exhibit 1-F) of the insurance Policy No. 4219 that would justify thedefendant-appellant, as insurer, to avoid its liability thereunder. It appears onthe face of said policy that a co-insurance in the amount of P20,000.00 wassecured from the Great American Insurance and was declared by theplaintiff-appellee and recognized by the defendant-appellant. This was later onsubstituted for the same amount and secured by the Federal InsuranceCompany. Chua Soon Poon on being cross-examined by counsel for thedefendant-appellant, declared that the Great American Insurance policy wascancelled because of the difference in the premium and the same waschanged for that of the Federal (t.s.n., hearing of December 1, 1964, pp.35-36). Contrary to the assertion of the defendant-appellant, the Great American Insurance policy was not substituted by the Northwest Insurancepolicy. As admitted by the defendant-appellant in its brief (p. 48), the fireinsurance policy issued by the Great American Insurance Company forP20,000.00 (Exhibit 1-E) was cancelled on August 29, 1962. On the otherhand, the fire insurance policy issued by the Northwest Insurance & SuretyCompany for P20,000.00 (Exhibit 1-K) was taken out on July 23, 1962. Howthen can the Northwest Insurance policy issued on July 23, 1962, beconsidered as having substituted the Great American policy which wascancelled only on August 29, 1962? The defendant-appellant can beconsidered to have waived the formal requirement of indorsing the policy ofco-insurance since there was absolutely no showing that it was not aware ofsaid substitution and preferred to continue the policy (Gonzales La O vs. YekTong Lin Fire and Marine Insurance Co., 55 Phil. 386). Even assuming that thedefendant-appellant did not indorse the Federal Insurance policy, there is noquestion that the same was only a substitution and did not in any way increasethe amount of the declared co-insurance. In other words, there was noincrease in the risk assumed by the defendant-appellant.

We do not agree with the conclusion of the Court of Appeals.

There was a violation by respondent Oliva Yap of the co-insurance clausecontained in Policy No. 4219 that resulted in the avoidance of petitioner'sliability. The insurance policy for P20,000.00 issued by the Great AmericanInsurance Company covering the same properties of respondent Yap and dulynoted on Policy No. 4219 as c-insurance, ceased, by agreement of the parties(Exhibit "1-L"), to be recognized by them as a co-insurance policy. The Courtof Appeals says that the Great American Insurance policy was substituted by

the Federal Insurance policy for the same amount, and because it was a merecase of substitution, there was no necessity for its endorsement on Policy No.4219. This finding, as well as reasoning, suffers from several flaws. There isno evidence to establish and prove such a substitution. If anything was

8/12/2019 Insurance Meeting 6 Cases

http://slidepdf.com/reader/full/insurance-meeting-6-cases 19/28

MEETING 6 CASES - INSURANCE

19

substituted for the Great American Insurance policy, it could only be theNorthwest Insurance policy for the same amount of P20,000.00. Theendorsement (Exhibit "1-K") quoted above shows the clear intention of theparties to recognize on the date the endorsement was made (August 29,1962), the existence of only one co-insurance, and that is the NorthwestInsurance policy, which according to the stipulation of the parties during thehearing, was issued on August 20, 1962 (t.s.n., January 12, 1965, pp. 3-4) andendorsed only on August 20, 1962. The finding of the Court of Appeals that the

Great American Insurance policy was substituted by the Federal Insurancepolicy is unsubstantiated by the evidence of record and indeed contrary to saidstipulation and admission of respondent, and is grounded entirely onspeculation, surmises or conjectures, hence, not binding on the SupremeCourt. 1

The Court of Appeals would consider petitioner to have waived the formalrequirement of endorsing the policy of co-insurance "since there wasabsolutely no showing that it was not aware of said substitution and preferredto continue the policy." The fallacy of this argument is that, contrary to Section1, Rule 131 of the Revised Rules of Court, which requires each party to provehis own allegations, it would shift to petitioner, respondent's burden of provingher proposition that petitioner was aware of the alleged substitution, and withsuch knowledge preferred to continue the policy. Respondent Yap citesGonzales La O vs. Yek Tong Lin Fire and Marine Insurance Co., Ltd. 2 to

justify the assumption but in that case, unlike here, there was knowledge bythe insurer of violations of the contract, to wit: "If, with the knowledge of theexistence of other insurances which the defendant deemed violations of thecontract, it has preferred to continue the policy, its action amounts to a waiverof the annulment of the contract ..." A waiver must be express. If it is to beimplied from conduct mainly, said conduct must be clearly indicative of a clearintent to waive such right. Especially in the case at bar where petitioner isassumed to have waived a valuable right, nothing less than a clear, positivewaiver, made with full knowledge of the circumstances, must be required.

By the plain terms of the policy, other insurance without the consent ofpetitioner would ipso facto avoid the contract. It required no affirmative act ofelection on the part of the company to make operative the clause avoiding thecontract, wherever the specified conditions should occur. Its obligationsceased, unless, being informed of the fact, it consented to the additionalinsurance.

The validity of a clause in a fire insurance policy to the effect that theprocurement of additional insurance without the consent of the insurer rendersipso facto the policy void is well-settled:

In Milwaukee Mechanids' Lumber Co., vs. Gibson, 199 Ark. 542, 134 S. W. 2d521, 522, a substantially identical clause was sustained and enforced, thecourt saying: "The rule in this state and practically all of the states is to theeffect that a clause in a policy to the effect that the procurement of additionalinsurance without the consent of the insurer renders the policy void is a validprovision. The earlier cases of Planters Mutual Insurance Co., vs. Green, 72 Ark. 305, 80 S.W. 92, are to the same effect." And see Vance, Insurance, 2ndEd., 725. (Reach vs. Arkansas Farmers Mut. Fire Ins. Co., [Ark. Nov. 14, 1949]224 S. W. 2d 48, 49.)

2. Where a policy contains a clause providing that the policy shall bevoid if insured has or shall procure any other insurance on the property, theprocurement of additional insurance without the consent of the insurer avoidsthe policy." (Planters' Mut. Ins. Ass'n vs. Green [Supreme Court of Arkansas,March 19, 1904] 80 S.W. 151.)

3. The policy provided that it should be void in case of other insurance"without notice and consent of this company. ..." It also authorized thecompany to terminate the contract at any time, at its option, by giving noticeand refunding a ratable proportion of the premium. Held, that additionalinsurance, unless consented to, or unless a waiver was shown, ipso factoavoided the contract, and the fact that the company had not, after notice ofsuch insurance, cancelled the policy, did not justify the legal conclusion that ithad elected to allow it to continue in force." (Johnson vs. American Fire Ins.,Co., [Supreme Court of Minnesota, Aug. 12, 1889] 43 N.W., 59)

The aforecited principles have been applied in this jurisdiction in GeneralInsurance & Surety Corporation vs. Ng Hua 3. There, the policy issued by theGeneral Insurance & Surety Corporation in favor of respondent Ng Huacontained a provision identical with the provisions in Policy No. 4219 quotedabove. 4 This Court, speaking thru Justice Cesar P. Bengson, in reversing the judgment of the Court of Appeals and absolving the insurer from liability underthe policy, held:

... And considering the terms of the policy which required the insured todeclare other insurances, the statement in question must be deemed to be astatement (warranty) binding on both insurer and insured, that there were noother insurance on the property. ...

The annotation then, must be deemed to be a warranty that the property wasnot insured by any other policy. Violation thereof entitled the insurer to rescind.(Sec. 69, Insurance Act.) Such misrepresentation is fatal in the light of our

views in Santa Ana vs. Commercial Union Assurance Company, Ltd., 55 Phil .329. The materiality of non-disclosure of other insurance policies is not opento doubt.

Furthermore, even if the annotations were overlooked the defendant insurerwould still be free from liability because there is no question that the policyissued by General Indemnity has not been stated in nor endorsed on PolicyNo. 471 of defendant. And as stipulated in the above-quoted provisions of

such policy "all benefit under this policy shall be forfeited. (Emphasis supplied)The obvious purpose of the aforesaid requirement in the policy is to preventover-insurance and thus avert the perpetration of fraud. The public, as well asthe insurer, is interested in preventing the situation in which a fire would beprofitable to the insured. According to Justice Story: "The insured has no rightto complain, for he assents to comply with all the stipulation on his side, inorder to entitle himself to the benefit of the contract, which, upon reason orprinciple, he has no right to ask the court to dispense with the performance ofhis own part of the agreement, and yet to bind the other party to obligations,which, but for those stipulation would not have been entered into." 5

In view of the above conclusion, We deem it unnecessary to consider the otherdefenses interposed by petitioner.

WHEREFORE, the appealed judgment of the Court of Appeals is reversedand set aside, and the petitioner absolved from all liability under the policy.Costs against private respondent.

SO ORDERED.

Fernando (Chairman), Barredo, Antonio and Aquino, JJ., concur.

Insurance Case Digest: New Life Enterprises V. Court Of Appeals (1992)G.R. No. 94071 March 31, 1992

Lessons Applicable: Requisites of Double insurance (Insurance)

FACTS:

May 15, 1981: Western Guaranty Corporation issued Fire Insurance Policy toNew Life Enterprises foar P350,000renewed on May, 13, 1982July 30,1981: Reliance Surety and Insurance Co., Inc. issued Fire InsurancePolicy to New Life Enterprises for P300,000November 12, 1981; Additional P700,000February 8, 1982: Equitable Insurance Corporation issued Fire InsurancePolicy to New Life Enterprises for P200,000October 19, 1982 2 am: fire electrical in nature destroyed the stock in tradeworth P1,550,000Julian Sy went to Reliance to claim but he was refused. Same thinghappened with the others who were sister companies.Sy violated the "Other Insurance Clause"RTC: favored New Life and against the three insurance companiesCA: reversed -failure to state or endorse the other insurance coverageISSUE: W/N Sy can claim against the three insurance companies for violatingthe "Other Insurance Clause"

HELD: NO.The terms of the contract are clear and unambiguous.The insured is specifically required to disclose to the insurer any otherinsurance and its particulars which he may have effected on the same subjectmatter.The knowledge of such insurance by the insurer's agents, even assuming theacquisition thereof by the former, is not the "notice" that would estop theinsurers from denying the claim.conclusion of the trial court that Reliance and Equitable are "sister companies"is an unfounded conjecture drawn from the mere fact that Yap Kam Chuanwas an agent for both companies which also had the same insurance claimsadjuster Availmentof the services of the same agents and adjusters by differentcompanies is a common practice in the insurancebusiness and such facts donot warrant the speculative conclusion of the trial court.The conformity of the insured to the terms of the policy isimplied from hisfailure to express any disagreement with what is provided for.a clear misrepresentation and a vital one because where the insured had beenasked to reveal but did not, that was deception - guilty of clear fraudtotal absence of such notice nullifies the policyassuming arguendo that petitioners felt the legitimate need to be clarified as tothe policy condition violated, there was a considerable lapse of time from theirreceipt of the insurer's clarificatory letter dated March 30, 1983, up to the timethe complaint was filed in court on January 31, 1984. The one-yearprescriptive period was yet toexpire on November 29, 1983, or about eight (8)

months from the receipt of the clarificatory letter, but petitioners let the periodlapse without bringing their action in court

8/12/2019 Insurance Meeting 6 Cases

http://slidepdf.com/reader/full/insurance-meeting-6-cases 20/28

MEETING 6 CASES - INSURANCE

20

G.R. No. 94071 March 31, 1992

NEW LIFE ENTERPRISES and JULIAN SY, petitioners,vs.HON. COURT OF APPEALS, EQUITABLE INSURANCE CORPORATION,RELIANCE SURETY AND INSURANCE CO., INC. and WESTERNGUARANTY CORPORATION, respondents.

REGALADO, J.:

This appeal by certiorari seeks the nullification of the decision 1 of respondentCourt of Appeals in CA-G.R. CV No. 13866 which reversed the decision of theRegional Trial Court, Branch LVII at Lucena City, jointly deciding Civil CasesNos. 6-84, 7-84 and 8-84 thereof and consequently ordered the dismissal ofthe aforesaid actions filed by herein petitioners.

The undisputed background of this case as found by the court a quo andadopted by respondent court, being sustained by the evidence on record, wehereby reproduce the same with approval. 2

The antecedents of this case show that Julian Sy and Jose Sy Bang have

formed a business partnership in the City of Lucena. Under the business nameof New Life Enterprises, the partnership engaged in the sale of constructionmaterials at its place of business, a two storey building situated at Iyam,Lucena City. The facts show that Julian Sy insured the stocks in trade of NewLife Enterprises with Western Guaranty Corporation, Reliance Surety andInsurance. Co., Inc., and Equitable Insurance Corporation.

On May 15, 1981, Western Guaranty Corporation issued Fire Insurance PolicyNo. 37201 in the amount of P350,000.00. This policy was renewed on May,13, 1982.

On July 30,1981, Reliance Surety and Insurance Co., Inc. issued FireInsurance Policy No. 69135 in the amount of P300,000.00 (Renewed underRenewal Certificate No. 41997) An additional insurance was issued by thesame company on November 12, 1981 under Fire Insurance Policy No. 71547in the amount of P700,000.00.

On February 8, 1982, Equitable Insurance Corporation issued Fire InsurancePolicy No. 39328 in the amount of P200,000.00.

Thus when the building occupied by the New Life Enterprises was gutted byfire at about 2:00 o'clock in the morning of October 19, 1982, the stocks in thetrade inside said building were insured against fire in the total amount ofP1,550,000.00. According to the certification issued by the Headquarters,Philippine Constabulary /Integrated National Police, Camp Crame, the causeof fire was electrical in nature. According to the plaintiffs, the building and thestocks inside were burned. After the fire, Julian Sy went to the agent ofReliance Insurance whom he asked to accompany him to the office of thecompany so that he can file his claim. He averred that in support of his claim,he submitted the fire clearance, the insurance policies and inventory of stocks.He further testified that the three insurance companies are sister companies,and as a matter of fact when he was following-up his claim with EquitableInsurance, the Claims Manager told him to go first to Reliance Insurance and ifsaid company agrees to pay, they would also pay. The same treatment wasgiven him by the other insurance companies. Ultimately, the three insurancecompanies denied plaintiffs' claim for payment.

In its letter of denial dated March 9, 1983, (Exhibit "C" No. 8-84) WesternGuaranty Corporation through Claims Manager Bernard S. Razon told theplaintiff that his claim "is denied for breach of policy conditions." RelianceInsurance purveyed the same message in its letter dated November 23, 1982and signed by Executive Vice-President Mary Dee Co (Exhibit "C" No. 7-84)which said that "plaintiff's claim is denied for breach of policy conditions." Theletter of denial received by the plaintiff from Equitable Insurance Corporation(Exhibit "C" No. 6-84) was of the same tenor, as said letter dated February 22,1983, and signed by Vice-President Elma R. Bondad, said "we find that certainpolicy conditions were violated, therefore, we regret, we have to deny yourclaim, as it is hereby denied in its entirety."

In relation to the case against Reliance Surety and Insurance Company, acertain Atty. Serafin D. Dator, acting in behalf of the plaintiff, sent a letter datedFebruary 13, 1983 (Exhibit "G-l" No 7-84) to Executive Vice-President MaryDee Co asking that he be informed as to the specific policy conditionsallegedly violated by the plaintiff. In her reply-letter dated March 30, 1983,Executive Vice-President Mary Dee Co informed Atty. Dator that Julian Syviolated Policy Condition No. "3" which requires the insured to give notice ofany insurance or insurances already effected covering the stocks in trade. 3

Because of the denial of their claims for payment by the three (3) insurance

companies, petitioner filed separate civil actions against the former before theRegional Trial Court of Lucena City, which cases were consolidated for trial,and thereafter the court below rendered its decision on December 19, l986with the following disposition:

WHEREFORE, judgment in the above-entitled cases is rendered in thefollowing manner, viz:

1. In Civil Case No. 6-84, judgment is rendered for the plaintiff NewLife Enterprises and against the defendant Equitable Insurance Corporationordering the latter to pay the former the sum of Two Hundred Thousand(P200,000.00) Pesos and considering that payment of the claim of the insured

has been unreasonably denied, pursuant to Sec. 244 of the Insurance Code,defendant is further ordered to pay the plaintiff attorney's fees in the amount ofTwenty Thousand (P20,000.00) Pesos. All sums of money to be paid by virtuehereof shall bear interest at 12% per annum (pursuant to Sec. 244 of theInsurance Code) from February 14, 1983, (91st day from November 16, 1982,when Sworn Statement of Fire Claim was received from the insured) until theyare fully paid;

2. In Civil Case No. 7-84, judgment is rendered for the plaintiff JulianSy and against the defendant Reliance Surety and Insurance Co., Inc.,ordering the latter to pay the former the sum of P1,000,000.00 (P300,000.00under Policy No. 69135 and P700,000.00 under Policy No. 71547) andconsidering that payment of the claim of the insured has been unreasonablydenied, pursuant to Sec. 244 of the Insurance Code, defendant is furtherordered to pay the plaintiff the amount of P100,000.00 as attorney's fees.

All sums of money to be paid by virtue hereof shall bear interest at 12% perannum (pursuant to Sec. 244 of the Insurance Code) from February 14, 1983,(91st day from November 16, 1982 when Sworn Statement of Fire Claim wasreceived from the insured) until they are fully paid;

3. In Civil Case No. 8-84, judgment is rendered for the plaintiff NewLife Enterprises and against the defendant Western Guaranty Corporationordering the latter to pay the sum of P350,000.00 to the Consolidated Bankand Trust Corporation, Lucena Branch, Lucena City, as stipulated on the faceof Policy No. 37201, and considering that payment of the aforementioned sumof money has been unreasonably denied, pursuant to Sec. 244 of theInsurance Code, defendant is further ordered to pay the plaintiff attorney'sfees in the amount of P35,000.00.

All sums of money to be paid by virtue hereof shall bear interest at 12% perannum (pursuant to Sec. 244 of the Insurance Code) from February 5, 1982,(91st day from 1st week of November 1983 when insured filed formal claim forfull indemnity according to adjuster Vetremar Dela Merced) until they are fullypaid. 4

As aforestated, respondent Court of Appeals reversed said judgment of thetrial court, hence this petition the crux wherein is whether or not ConditionsNos. 3 and 27 of the insurance contracts were violated by petitioners therebyresulting in their forfeiture of all the benefits thereunder.

Condition No. 3 of said insurance policies, otherwise known as the "OtherInsurance Clause," is uniformly contained in all the aforestated insurancecontracts of herein petitioners, as follows:

3. The insured shall give notice to the Company of any insurance orinsurances already effected, or which may subsequently be effected, coveringany of the property or properties consisting of stocks in trade, goods inprocess and/or inventories only hereby insured, and unless such notice begiven and the particulars of such insurance or insurances be stated therein orendorsed on this policy pursuant to Section 50 of the Insurance Code, by or onbehalf of the Company before the occurrence of any loss or damage, allbenefits under this policy shall be deemed forfeited, provided however, thatthis condition shall not apply when the total insurance or insurances in force atthe time of loss or damage not more than P200,000.00. 5

Petitioners admit that the respective insurance policies issued by privaterespondents did not state or endorse thereon the other insurance coverageobtained or subsequently effected on the same stocks in trade for the loss ofwhich compensation is claimed by petitioners. 6 The policy issued byrespondent Western Guaranty Corporation (Western) did not declarerespondent Reliance Surety and Insurance Co., Inc. (Reliance) andrespondent Equitable Insurance Corporation (Equitable) as co-insurers on thesame stocks, while Reliance's Policies covering the same stocks did notlikewise declare Western and Equitable as such co-insurers. It is furtheradmitted by petitioners that Equitable's policy stated "nil" in the space thereonrequiring indication of any co-insurance although there were three (3) policiessubsisting on the same stocks in trade at the time of the loss, namely, that ofWestern in the amount of P350,000.00 and two (2) policies of Reliance in thetotal amount of P1,000,000.00. 7

In other words, the coverage by other insurance or co-insurance effected orsubsequently arranged by petitioners were neither stated nor endorsed in thepolicies of the three (3) private respondents, warranting forfeiture of all

benefits thereunder if we are to follow the express stipulation in theaforequoted Policy Condition No. 3.

8/12/2019 Insurance Meeting 6 Cases

http://slidepdf.com/reader/full/insurance-meeting-6-cases 21/28

MEETING 6 CASES - INSURANCE

21

Petitioners contend that they are not to be blamed for the omissions, allegingthat insurance agent Leon Alvarez (for Western) and Yap Kam Chuan (forReliance and Equitable) knew about the existence of the additional insurancecoverage and that they were not informed about the requirement that suchother or additional insurance should be stated in the policy, as they have noteven read policies. 8 These contentions cannot pass judicial muster.

The terms of the contract are clear and unambiguous. The insured is

specifically required to disclose to the insurer any other insurance and itsparticulars which he may have effected on the same subject matter. Theknowledge of such insurance by the insurer's agents, even assuming theacquisition thereof by the former, is not the "notice" that would estop theinsurers from denying the claim. Besides, the so-called theory of imputedknowledge, that is, knowledge of the agent is knowledge of the principal, asidefrom being of dubious applicability here has likewise been roundly refuted byrespondent court whose factual findings we find acceptable.

Thus, it points out that while petitioner Julian Sy claimed that he had informedinsurance agent Alvarez regarding the co-insurance on the property, hecontradicted himself by inexplicably claiming that he had not read the terms ofthe policies; that Yap Dam Chuan could not likewise have obtained suchknowledge for the same reason, aside from the fact that the insurance withWestern was obtained before those of Reliance and Equitable; and that the

conclusion of the trial court that Reliance and Equitable are "sister companies"is an unfounded conjecture drawn from the mere fact that Yap Kam Chuanwas an agent for both companies which also had the same insurance claimsadjuster. Availment of the services of the same agents and adjusters bydifferent companies is a common practice in the insurance business and suchfacts do not warrant the speculative conclusion of the trial court.

Furthermore, when the words and language of documents are clear and plainor readily understandable by an ordinary reader thereof, there is absolutely noroom for interpretation or construction anymore. 9 Courts are not allowed tomake contracts for the parties; rather, they will intervene only when the termsof the policy are ambiguous, equivocal, or uncertain. 10 The parties mustabide by the terms of the contract because such terms constitute the measureof the insurer's liability and compliance therewith is a condition precedent tothe insured's right of recovery from the insurer. 11

While it is a cardinal principle of insurance law that a policy or contract ofinsurance is to be construed liberally in favor of the insured and strictly againstthe insurer company, yet contracts of insurance, like other contracts, are to beconstrued according to the sense and meaning of the terms which the partiesthemselves have used. If such terms are clear and unambiguous, they mustbe taken and understood in their plain, ordinary and popular sense. 12Moreover, obligations arising from contracts have the force of law between thecontracting parties and should be complied with in good faith. 13

Petitioners should be aware of the fact that a party is not relieved of the duty toexercise the ordinary care and prudence that would be exacted in relation toother contracts. The conformity of the insured to the terms of the policy isimplied from his failure to express any disagreement with what is provided for.14 It may be true that the majority rule, as cited by petitioners, is that injuredpersons may accept policies without reading them, and that this is notnegligence per se. 15 But, this is not without any exception. It is and wasincumbent upon petitioner Sy to read the insurance contracts, and this can bereasonably expected of him considering that he has been a businessmansince 1965 16 and the contract concerns indemnity in case of loss in hismoney-making trade of which important consideration he could not have beenunaware as it was pre-in case of loss in his money-making trade of whichimportant consideration he could not have been unaware as it was preciselythe reason for his procuring the same.

We reiterate our pronouncement in Pioneer Insurance and Surety Corporationvs. Yap: 17

. . . And considering the terms of the policy which required the insured todeclare other insurances, the statement in question must be deemed to be astatement (warranty) binding on both insurer and insured, that there were noother insurance on the property. . . .

The annotation then, must be deemed to be a warranty that the property wasnot insured by any other policy. Violation thereof entitled the insurer to rescind(Sec. 69, Insurance Act). Such misrepresentation is fatal in the light of ourviews in Santa Ana vs. Commercial Union Assurance Company, Ltd., 55 Phil.329. The materiality of non-disclosure of other insurance policies is not opento doubt.

xxx xxx xxx

The obvious purpose of the aforesaid requirement in the policy is to preventover-insurance and thus avert the perpetration of fraud. The public, as well as

the insurer, is interested in preventing the situation in which a fire would beprofitable to the insured. According to Justice Story: "The insured has no rightto complain, for he assents to comply with all the stipulations on his side, inorder to entitle himself to the benefit of the contract, which, upon reason or

principle, he has no right to ask the court to dispense with the performance ofhis own part of the agreement, and yet to bind the other party to obligations,which, but for those stipulations, would not have been entered into."

Subsequently, in the case of Pacific Banking Corporation vs. Court of Appeals,et al., 18 we held:

It is not disputed that the insured failed to reveal before the loss three other

insurances. As found by the Court of Appeals, by reason of said unrevealedinsurances, the insured had been guilty of a false declaration; a clearmisrepresentation and a vital one because where the insured had been askedto reveal but did not, that was deception. Otherwise stated, had the insurerknown that there were many co-insurances, it could have hesitated or plainlydesisted from entering into such contract. Hence, the insured was guilty ofclear fraud (Rollo, p. 25).

Petitioner's contention that the allegation of fraud is but a mere inference orsuspicion is untenable. In fact, concrete evidence of fraud or false declarationby the insured was furnished by the petitioner itself when the facts alleged inthe policy under clauses "Co-Insurances Declared" and "Other InsuranceClause" are materially different from the actual number of co-insurances takenover the subject property. Consequently, "the whole foundation of the contractfails, the risk does not attach and the policy never becomes a contract

between the parties." Representations of facts are the foundation of thecontract and if the foundation does not exist, the superstructure does not arise.Falsehood in such representations is not shown to vary or add to the contract,or to terminate a contract which has once been made, but to show that nocontract has ever existed (Tolentino, Commercial Laws of the Philippines, p.991, Vol. II, 8th Ed.,) A void or inexistent contract is one which has no forceand effect from the very beginning, as if it had never been entered into, andwhich cannot be validated either by time or by ratification (Tongoy vs. C.A.,123 SCRA 99 (1983); Avila v. C.A., 145 SCRA, 1986).

As the insurance policy against fire expressly required that notice should begiven by the insured of other insurance upon the same property, the totalabsence of such notice nullifies the policy.

To further warrant and justify the forfeiture of the benefits under the insurancecontracts involved, we need merely to turn to Policy Condition No. 15 thereof,which reads in part:

15. . . . if any false declaration be made or used in support thereof, . . .all benefits under this Policy shall be forfeited . . . . 19

Additionally, insofar as the liability of respondent Reliance is concerned, it isnot denied that the complaint for recovery was filed in court by petitioners onlyon January 31, 1984, or after more than one (1) year had elapsed frompetitioners' receipt of the insurers' letter of denial on November 29, 1982.Policy Condition No. 27 of their insurance contract with Reliance provides:

27. Action or suit clause.— If a claim be made and rejected and anaction or suit be not commenced either in the Insurance Commission or anycourt of competent jurisdiction of notice of such rejection, or in case ofarbitration taking place as provided herein, within twelve (12) months after duenotice of the award made by the arbitrator or arbitrators or umpire, then theclaim shall for all purposes be deemed to have been abandoned and shall notthereafter be recoverable hereunder. 20

On this point, the trial court ruled:

. . . However, because of the peculiar circumstances of this case, we hesitatein concluding that plaintiff's right to ventilate his claim in court has been barredby reason of the time constraint provided in the insurance contract. It is evidentthat after the plaintiff had received the letter of denial, he still found itnecessary to be informed of the specific causes or reasons for the denial of hisclaim, reason for which his lawyer, Atty. Dator deemed it wise to send a letterof inquiry to the defendant which was answered by defendant's ExecutiveVice-President in a letter dated March 30, 1983, . . . . Assuming, gratuitously,that the letter of Executive Vice-President Mary Dee Co dated March 30, 1983,was received by plaintiff on the same date, the period of limitation should startto run only from said date in the spirit of fair play and equity. . . . 21

We have perforce to reject this theory of the court below for being contrary towhat we have heretofore declared:

It is important to note the principle laid down by this Court in the case of Angvs. Fulton Fire Insurance Co. (2 SCRA 945 [1961]) to wit:

The condition contained in an insurance policy that claims must be presentedwithin one year after rejection is not merely a procedural requirement but animportant matter essential to a prompt settlement of claims against insurancecompanies as it demands that insurance suits be brought by the insured while

the evidence as to the origin and cause of destruction have not yetdisappeared.

8/12/2019 Insurance Meeting 6 Cases

http://slidepdf.com/reader/full/insurance-meeting-6-cases 22/28

MEETING 6 CASES - INSURANCE

22

In enunciating the above-cited principle, this Court had definitely settled therationale for the necessity of bringing suits against the Insurer within one yearfrom the rejection of the claim. The contention of the respondents that theone-year prescriptive period does not start to run until the petition forreconsideration had been resolved by the insurer, runs counter to the declaredpurpose for requiring that an action or suit be filed in the InsuranceCommission or in a court of competent jurisdiction from the denial of the claim.To uphold respondents' contention would contradict and defeat the very

principle which this Court had laid down. Moreover, it can easily be used byinsured persons as a scheme or device to waste time until any evidence whichmay be considered against them is destroyed.

xxx xxx xxx

While in the Eagle Star case (96 Phil. 701), this Court uses the phrase "finalrejection", the same cannot be taken to mean the rejection of a petition forreconsideration as insisted by respondents. Such was clearly not the meaningcontemplated by this Court. The insurance policy in said case provides thatthe insured should file his claim first, with the carrier and then with the insurer.The "final rejection" being referred to in said case is the rejection by theinsurance company. 22

Furthermore, assuming arguendo that petitioners felt the legitimate need to be

clarified as to the policy condition violated, there was a considerable lapse oftime from their receipt of the insurer's clarificatory letter dated March 30, 1983,up to the time the complaint was filed in court on January 31, 1984. Theone-year prescriptive period was yet to expire on November 29, 1983, orabout eight (8) months from the receipt of the clarificatory letter, but petitionerslet the period lapse without bringing their action in court. We accordingly findno "peculiar circumstances" sufficient to relax the enforcement of the one-yearprescriptive period and we, therefore, hold that petitioners' claim was definitelyfiled out of time.

WHEREFORE, finding no cogent reason to disturb the judgment ofrespondent Court of Appeals, the same is hereby AFFIRMED.

SO ORDERED.

Melencio-Hererra and Nocon, JJ., concur.

Paras, J., took no part.

Padilla, J., took no part.

FIRST DIVISION

PRUDENTIAL GUARANTEE and ASSURANCE INC.,Petitioner,

- versus -

TRANS-ASIA SHIPPING LINES, INC.,Respondent.

G.R. No. 151890

x- - - - - - - - - - - - - - - - - - - - - - - - - x

TRANS-ASIA SHIPPING LINES,INC.,

Petitioner,

- versus –

PRUDENTIAL GUARANTEE and ASSURANCE INC.,Respondent.

G.R. No. 151991

Present:

PANGANIBAN, C.J.Chairperson,

YNARES-SANTIAGO, AUSTRIA-MARTINEZ,CALLEJO, SR., andCHICO-NAZARIO, JJ.

Promulgated:

June 20, 2006x----------------------------------------------------------------------------------------x

D E C I S I O N

CHICO-NAZARIO, J:

This is a consolidation of two separate Petitions for Review on Certiorari filedby petitioner Prudential Guarantee and Assurance, Inc. (PRUDENTIAL) inG.R. No. 151890 and Trans-Asia Shipping Lines, Inc. (TRANS-ASIA) in G.R.No. 151991, assailing the Decision[1] dated 6 November 2001 of the Court of Appeals in CA G.R. CV No. 68278, which reversed the Judgment[2] dated 6June 2000 of the Regional Trial Court (RTC), Branch 13, Cebu City in CivilCase No. CEB-20709. The 29 January 2002 Resolution[3] of the Court of

Appeals, denying PRUDENTIAL‘s Motion for Reconsideration andTRANS- ASIA‘s Partial Motion for Reconsideration of the 6 November 2001Decision, is likewise sought to be annulled and set aside.

The Facts

The material antecedents as found by the court a quo and adoptedby the appellate court are as follows:

Plaintiff [TRANS-ASIA] is the owner of the vessel M/V Asia Korea.In consideration of payment of premiums, defendant [PRUDENTIAL] insuredM/V Asia Korea for loss/damage of the hull and machinery arising from perils,inter alia, of fire and explosion for the sum of P40 Million, beginning [from] theperiod [of] July 1, 1993 up to July 1, 1994. This is evidenced by Marine PolicyNo. MH93/1363 (Exhibits ―A‖ to ―A-11‖). On October 25, 1993, while thepolicy was in force, a fire broke out while [M/V Asia Korea was] undergoingrepairs at the port of Cebu. On October 26, 1993 plaintiff [TRANS-ASIA] filedits notice of claim for damage sustained by the vessel. This is evidenced by aletter/formal claim of even date (Exhibit ―B‖). Plaintiff [TRANS-ASIA]reserved its right to subsequently notify defendant [PRUDENTIAL] as to thefull amount of the claim upon final survey and determination by averageadjuster Richard Hogg International (Phil.) of the damage sustained by reasonof fire. An adjuster‘s report on the fire in question was submitted by RichardHogg International together with the U-Marine Surveyor Report (Exhibits ―4‖ to―4-115‖).

On May 29, 1995[,] plaintiff [TRANS-ASIA] executed a documentdenominated ―Loan and Trust receipt‖, a portion of which read (sic):

―Received from Prudential Guarantee and Assurance, Inc., thesum of PESOS THREE MILLION ONLY (P3,000,000.00) as a loan withoutinterest under Policy No. MH 93/1353 [sic], repayable only in the event and tothe extent that any net recovery is made by Trans-Asia Shipping Corporation,from any person or persons, corporation or corporations, or other parties, onaccount of loss by any casualty for which they may be liable occasioned by the25 October 1993: Fire on Board.‖ (Exhibit ―4‖)

In a letter dated 21 April 1997 defendant [PRUDENTIAL] deniedplaintiff‘s claim (Exhibit ―5‖). The letter reads:

―After a careful reviewand evaluation of your claim arising fromthe above-captioned incident, it has been ascertained that you are in breach ofpolicy conditions, among them ―WARRANTED VESSEL CLASSED ANDCLASS MAINTAINED‖. Accordingly, we regret to advise that your claim isnot compensable and hereby DENIED.‖

This was followed by defendant‘s letter dated 21 July 1997requesting the return or payment of the P3,000,000.00 within a period of ten(10) days from receipt of the letter (Exhibit ―6‖).[4]

Following this development, on 13 August 1997, TRANS-ASIA filed aComplaint[5] for Sum of Money against PRUDENTIAL with the RTC of CebuCity, docketed as Civil Case No. CEB-20709, wherein TRANS-ASIA soughtthe amount of P8,395,072.26 from PRUDENTIAL, alleging that the samerepresents the balance of the indemnity due upon the insurance policy in thetotal amount of P11,395,072.26. TRANS-ASIA similarly sought interest at42% per annum citing Section 243[6] of Presidential Decreee No. 1460,otherwise known as the ―Insurance Code,‖ as amended.

In its Answer,[7] PRUDENTIAL denied the material allegations of theComplaint and interposed the defense that TRANS-ASIA breached insurancepolicy conditions, in particular: ―WARRANTED VESSEL CLASSED AND

8/12/2019 Insurance Meeting 6 Cases

http://slidepdf.com/reader/full/insurance-meeting-6-cases 23/28

8/12/2019 Insurance Meeting 6 Cases

http://slidepdf.com/reader/full/insurance-meeting-6-cases 24/28

MEETING 6 CASES - INSURANCE

24

THE COURT OF APPEALSERRED IN HOLDING THAT THE ―LOAN ANDTRUST RECEIPT‖ EXECUTED BY TRANS-ASIA IS AN ADVANCE ON THEPOLICY, THUS CONSTITUTING PARTIAL PAYMENT THEREOF.

VII.

THE COURT OF APPEALS ERRED IN HOLDING THAT THE ACCEPTANCEBY PRUDENTIAL OF THE FINDINGS OF RICHARDS HOGG IS INDICATIVE

OF A WAIVER ON THE PART OF PRUDENTIAL OF ANY VIOLATION BYTRANS-ASIA OF THE WARRANTY.

VIII.

THE COURT OF APPEALS ERRRED (sic) IN REVERSING THE TRIALCOURT, IN FINDING THAT PRUDENTIAL ―UNJUSTIFIABLY REFUSED‖ TOPAY THE CLAIM AND IN ORDERING PRUDENTIAL TO PAY TRANS-ASIAP8,395,072.26 PLUS DOUBLE INTEREST FROM 13 AUGUST 1996, UNTIL[THE] SAME IS FULLY PAID.[15]

Similarly, TRANS-ASIA, disagreeing in the ruling of the Court of Appeals filed a Petition for Review docketed as G.R. No. 151991, raising the

following grounds for the allowance of the petition, to wit:

I.THE HONORABLE COURT OF APPEALS ERRED IN NOT AWARDING ATTORNEY‘S FEES TO PETITIONER TRANS-ASIA ON THE GROUNDTHAT SUCH CAN ONLY BE AWARDED IN THE CASES ENUMERATED IN ARTICLE 2208 OF THE CIVIL CODE, AND THERE BEING NO BAD FAITHON THE PART OF RESPONDENT PRUDENTIAL IN DENYING HEREINPETITIONER TRANS- ASIA‘S INSURANCE CLAIM.

II.

THE ―DOUBLE INTEREST‖ REFERRED TO IN THE DECISION DATED 06NOVEMBER 2001 SHOULD BE CONSTRUED TO MEAN DOUBLEINTEREST BASED ON THE LEGAL INTEREST OF 12%, OR INTEREST ATTHE RATE OF 24% PER ANNUM.[16]

In our Resolution of 2 December 2002, we granted TRANS- ASIA‘sMotion for Consolidation[17] of G.R. Nos. 151890 and 151991;[18] hence, theinstant consolidated petitions.

In sum, for our main resolution are: (1) the liability, if any, ofPRUDENTIAL to TRANS-ASIA arising from the subject insurance contract; (2)the liability, if any, of TRANS-ASIA to PRUDENTIAL arising from thetransaction between the parties as evidenced by a document denominated as―Loan and Trust Receipt,‖ dated 29 May 1995; and (3) the amount of interestto be imposed on the liability, if any, of either or both parties.

Ruling of the Court

Prefatorily, it must be emphasized that in a petition for review, onlyquestions of law, and not questions of fact, may be raised.[19] This rule maybe disregarded only when the findings of fact of the Court of Appeals arecontrary to the findings and conclusions of the trial court, or are not supportedby the evidence on record.[20] In the case at bar, we find an incongruencebetween the findings of fact of the Court of Appeals and the court a quo, thus,in our determination of the issues, we are constrained to assess the evidenceadduced by the parties to make appropriate findings of facts as are necessary.

I.

A. PRUDENTIAL failed to establish that TRANS-ASIA violated andbreached the policy condition on WARRANTED VESSEL CLASSED ANDCLASS MAINTAINED, as contained in the subject insurance contract.

In resisting the claim of TRANS-ASIA, PRUDENTIAL posits thatTRANS-ASIA violated an express and material warranty in the subjectinsurance contract, i.e., Marine Insurance Policy No. MH93/1363, specificallyWarranty Clause No. 5 thereof, which stipulates that the insured vessel, ―M/V ASIA KOREA‖ is required to be CLASSED AND CLASS MAINTAINED. According to PRUDENTIAL, on 25 October 1993, or at the time of theoccurrence of the fire, ―M/V ASIA KOREA‖ was in violation of the warranty as itwas not CLASSED AND CLASS MAINTAINED. PRUDENTIAL submits thatWarranty Clause No. 5 was a condition precedent to the recovery ofTRANS-ASIA under the policy, the violation of which entitled PRUDENTIAL torescind the contract under Sec. 74[21] of the Insurance Code.

The warranty condition CLASSED AND CLASS MAINTAINED wasexplained by PRUDENTIAL‘s Senior Manager of the Marine and AviationDivision, Lucio Fernandez. The pertinent portions of his testimony on directexamination is reproduced hereunder, viz:

ATTY. LIM

Q Please tell the court, Mr. Witness, the result of the evaluation of thisclaim, what final action was taken?

A It was eventually determined that there was a breach of the policycondition, and basically there is a breach of policy warranty condition and on

that basis the claim was denied.Q To refer you (sic) the ―policy warranty condition,‖ I am showing toyou a policy here marked as Exhibits ―1‖, ―1- A‖ series, please point to thewarranty in the policy which you said was breached or violated by the plaintiffwhich constituted your basis for denying the claim as you testified.

A Warranted Vessel Classed and Class Maintained.

ATTY. LIM

Witness pointing, Your Honor, to that portion in Exhibit ―1- A‖ which is thesecond page of the policy below the printed words: ―Clauses, Endorsements,Special Conditions and Warranties,‖ below this are several typewrittenclauses and the witness pointed out in particular the clause reading:

―Warranted Vessel Classed and Class Maintained.‖

COURT

Q Will you explain that particular phrase?

A Yes, a warranty is a condition that has to be complied with by theinsured. When we say a class warranty, it must be entered in theclassification society.

COURT

Slowly.

WITNESS(continued)

A A classification society is an organization which sets certainstandards for a vessel to maintain in order to maintain their membership in theclassification society. So, if they failed to meet that standard, they areconsidered not members of that class, and thus breaching the warranty, thatrequires them to maintain membership or to maintain their class on thatclassification society. And it is not sufficient that the member of thisclassification society at the time of a loss, their membership must becontinuous for the whole length of the policy such that during the effectivity ofthe policy, their classification is suspended, and then thereafter, they getreinstated, that again still a breach of the warranty that they maintained theirclass (sic). Our maintaining team membership in the classification societythereby maintaining the standards of the vessel (sic).

ATTY. LIMQ Can you mention some classification societies that you know?

A Well we have the Bureau Veritas, American Bureau of Shipping,D&V Local Classification Society, The Philippine Registration of ShipsSociety, China Classification, NKK and Company Classification Society, andmany others, we have among others, there are over 20 worldwide. [22]

At the outset, it must be emphasized that the party which alleges a fact as amatter of defense has the burden of proving it. PRUDENTIAL, as the partywhich asserted the claim that TRANS-ASIA breached the warranty in thepolicy, has the burden of evidence to establish the same. Hence, on the partof PRUDENTIAL lies the initiative to show proof in support of its defense;otherwise, failing to establish the same, it remains self-serving. Clearly, if noevidence on the alleged breach of TRANS-ASIA of the subject warranty isshown, a fortiori, TRANS-ASIA would be successful in claiming on the policy.It follows that PRUDENTIAL bears the burden of evidence to establish the factof breach.

In our rule on evidence, TRANS-ASIA, as the plaintiff below, necessarily hasthe burden of proof to show proof of loss, and the coverage thereof, in thesubject insurance policy. However, in the course of trial in a civil case, onceplaintiff makes out a prima facie case in his favor, the duty or the burden ofevidence shifts to defendant to controvert plaintiff‘s prima facie case,otherwise, a verdict must be returned in favor of plaintiff.[23] TRANS-ASIAwas able to establish proof of loss and the coverage of the loss, i.e., 25October 1993: Fire on Board. Thereafter, the burden of evidence shifted toPRUDENTIAL to counter TRANS- ASIA‘s case, and to prove its special and

affirmative defense that TRANS-ASIA was in violation of the particularcondition on CLASSED AND CLASS MAINTAINED.

8/12/2019 Insurance Meeting 6 Cases

http://slidepdf.com/reader/full/insurance-meeting-6-cases 25/28

MEETING 6 CASES - INSURANCE

25

We sustain the findings of the Court of Appeals that PRUDENTIAL was notsuccessful in discharging the burden of evidence that TRANS-ASIA breachedthe subject policy condition on CLASSED AND CLASS MAINTAINED.

Foremost, PRUDENTIAL, through the Senior Manager of its Marine and Aviation Division, Lucio Fernandez, made a categorical admission that at thetime of the procurement of the insurance contract in July 1993, TRANS- ASIA‘svessel, ―M/V Asia Korea‖ was properly classed by Bureau Veritas, thus:

Q Kindly examine the records particularly the policy, please tell us ifyou know whether M/V Asia Korea was classed at the time (sic) policy wasprocured perthe (sic) insurance was procured that Exhibit ―1‖ on 1st July 1993(sic).

WITNESS

A I recall that they were classed.

ATTY. LIM

Q With what classification society?

A I believe with Bureau Veritas.[24]

As found by the Court of Appeals and as supported by the records, BureauVeritas is a classification society recognized in the marine industry. As it isundisputed that TRANS-ASIA was properly classed at the time the contract ofinsurance was entered into, thus, it becomes incumbent upon PRUDENTIALto show evidence that the status of TRANS-ASIA as being properly CLASSEDby Bureau Veritas had shifted in violation of the warranty. Unfortunately,PRUDENTIAL failed to support the allegation.

We are in accord with the ruling of the Court of Appeals that the lack of acertification in PRUDENTIAL‘s records to the effect that TRANS- ASIA‘s ―M/V Asia Korea‖ was CLASSED AND CLASS MAINTAINED at the time of theoccurrence of the fire cannot be tantamount to the conclusion thatTRANS-ASIA in fact breached the warranty contained in the policy. Withmore reason must we sustain the findings of the Court of Appeals on theground that as admitted by PRUDENTIAL, it was likewise the responsibility ofthe average adjuster, Richards Hogg International (Phils.), Inc., to secure acopy of such certification, and the alleged breach of TRANS-ASIA cannot begleaned from the average adjuster‘s survey report, or adjustment of particularaverage per ―M/V Asia Korea‖ of the 25 October1993 fire on board.

We are not unmindful of the clear language of Sec. 74 of the Insurance Codewhich provides that, ―the violation of a material warranty, or other materialprovision of a policy on the part of either party thereto, entitles the other torescind.‖ It is generally accepted that ―[a] warranty is a statement or promiseset forth in the policy, or by reference incorporated therein, the untruth ornon-fulfillment of which in any respect, and without reference to whether theinsurer was in fact prejudiced by such untruth or non-fulfillment, renders thepolicy voidable by the insurer.‖[25] However, it is similarly indubitable that forthe breach of a warranty to avoid a policy, the same must be duly shown by theparty alleging the same. We cannot sustain an allegation that is unfounded.Consequently, PRUDENTIAL, not having shown that TRANS-ASIA breachedthe warranty condition, CLASSED AND CLASS MAINTAINED, it remains thatTRANS-ASIA must be allowed to recover its rightful claims on the policy.

B. Assuming arguendo that TRANS-ASIA violated the policy condition onWARRANTED VESSEL CLASSED AND CLASS MAINTAINED,PRUDENTIAL made a valid waiver of the same.

The Court of Appeals, in reversing the Judgment of the RTC which held thatTRANS-ASIA breached the warranty provision on CLASSED AND CLASSMAINTAINED, underscored that PRUDENTIAL can be deemed to have madea valid waiver of TRANS- ASIA‘s breach of warranty as alleged, ratiocinating,thus:

Third, after the loss, Prudential renewed the insurance policy of Trans-Asia fortwo (2) consecutive years, from noon of 01 July 1994 to noon of 01 July 1995,and then again until noon of 01 July 1996. This renewal is deemed a waiverof any breach of warranty.[26]

PRUDENTIAL finds fault with the ruling of the appellate court when it ruled thatthe renewal policies are deemed a waiver of TRANS- ASIA‘s alleged breach,averring herein that the subsequent policies, designated as MH94/1595 andMH95/1788 show that they were issued only on 1 July 1994 and 3 July 1995,respectively, prior to the time it made a request to TRANS-ASIA that it befurnished a copy of the certification specifying that the insured vessel ―M/V

Asia Korea‖ was CLASSED AND CLASS MAINTAINED. PRUDENTIALposits that it came to know of the breach by TRANS-ASIA of the subjectwarranty clause only on 21 April 1997. On even date, PRUDENTIAL sentTRANS-ASIA a letter of denial, advising the latter that their claim is not

compensable. In fine, PRUDENTIAL would have this Court believe that theissuance of the renewal policies cannot be a waiver because they were issuedwithout knowledge of the alleged breach of warranty committed byTRANS-ASIA.[27]

We are not impressed. We do not find that the Court of Appealswas in error when it held that PRUDENTIAL, in renewing TRANS- ASIA‘sinsurance policy for two consecutive years after the loss covered by Policy No.

MH93/1363, was considered to have waived TRANS- ASIA‘s breach of thesubject warranty, if any. Breach of a warranty or of a condition renders thecontract defeasible at the option of the insurer; but if he so elects, he maywaive his privilege and power to rescind by the mere expression of anintention so to do. In that event his liability under the policy continues asbefore.[28] There can be no clearer intention of the waiver of the allegedbreach than the renewal of the policy insurance granted by PRUDENTIAL toTRANS-ASIA in MH94/1595 and MH95/1788, issued in the years 1994 and1995, respectively.

To our mind, the argument is madeeven more credulous by PRUDENTIAL‘slack of proof to support its allegation that the renewals of the policies weretaken only after a request was made to TRANS-ASIA to furnish them a copy ofthe certificate attesting that ―M/V Asia Korea‖ was CLASSED ANDCLASSMAINTAINED. Notwithstanding PRUDENTIAL‘s claim that no certification

was issued to that effect, it renewed the policy, thereby, evidencing anintention to waive TRANS- ASIA‘s alleged breach. Clearly, by granting therenewal policies twice and successively after the loss, the intent was to benefitthe insured, TRANS-ASIA, as well as to waive compliance of the warranty.

The foregoing finding renders a determination of whether thesubject warranty is a rider, moot, as raised by the PRUDENTIAL in itsassignment of errors. Whether it is a rider will not effectively alter the resultfor the reasons that: (1) PRUDENTIAL was not able to discharge the burden ofevidence to show that TRANS-ASIA committed a breach, thereof; and (2)assuming arguendo the commission of a breach by TRANS-ASIA, the samewas shown to have been waived by PRUDENTIAL.

II.

A. The amount of P3,000,000.00 granted by PRUDENTIAL to TRANS- ASIA via a transaction between the parties evidenced by a documentdenominated as ―Loan and Trust Receipt,‖ dated 29 May 1995 constitutedpartial payment on the policy.

It is undisputed that TRANS-ASIA received from PRUDENTIAL the amount ofP3,000,000.00. The same was evidenced by a transaction receiptdenominated as a ―Loan and Trust Receipt,‖ dated 29 May 1995, reproducedhereunder:

LOAN AND TRUST RECEIPT

Claim File No. MH-93-025 May 29,1995P3,000,000.00Check No. PCIB066755

Received FROM PRUDENTIAL GUARANTEE AND ASSURANCE INC., thesum of PESOS THREE MILLION ONLY (P3,000,000.00) as a loan withoutinterest, under Policy No. MH93/1353, repayable only in the event and to theextent that any net recovery is made by TRANS ASIA SHIPPING CORP., fromany person or persons, corporation or corporations, or other parties, onaccount of loss by any casualty for which they may be liable, occasioned bythe 25 October 1993: Fire on Board.

As security for such repayment, we hereby pledge to PRUDENTIALGUARANTEE AND ASSURANCE INC. whatever recovery we may make anddeliver to it all documents necessary to prove our interest in said property.We also hereby agree to promptly prosecute suit against such persons,corporation or corporations through whose negligence the aforesaid loss wascaused or who may otherwise be responsible therefore, with all due diligence,in our own name, but at the expense of and under the exclusive direction andcontrol of PRUDENTIAL GUARANTEE AND ASSURANCE INC.

TRANS-ASIA SHIPPINGCORPORATION[29]

PRUDENTIAL largely contends that the ―Loan and Trust Receipt‖ executed bythe parties evidenced a loan of P3,000,000.00 which it granted toTRANS-ASIA, and not an advance payment on the policy or a partial paymentfor the loss. It further submits that it is a customary practice for insurancecompanies in this country to extend loans gratuitously as part of good

business dealing with their assured, in order to afford their assured the chanceto continue business without embarrassment while awaiting outcome of thesettlement of their claims.[30] According to PRUDENTIAL, the ―Trust andLoan Agreement‖ did not subrogate to it whatever rights and/or actions

8/12/2019 Insurance Meeting 6 Cases

http://slidepdf.com/reader/full/insurance-meeting-6-cases 26/28

MEETING 6 CASES - INSURANCE

26

TRANS-ASIA may have against third persons, and it cannot by no means betaken that by virtue thereof, PRUDENTIAL was granted irrevocable power ofattorney by TRANS-ASIA, as the sole power to prosecute lies solely with thelatter.

The Court of Appeals held that the real character of the transaction betweenthe parties as evidenced by the ―Loan and Trust Receipt‖ is that of an advancepayment by PRUDENTIAL of TRANS- ASIA‘s claim on the insurance, thus:

The Philippine Insurance Code (PD 1460 as amended) was derived from theold Insurance Law Act No. 2427 of the Philippine Legislature during the American Regime. The Insurance Act was l ifted verbatim from the law ofCalifornia, except Chapter V thereof, which was taken largely from theinsurance law of New York. Therefore, ruling case law in that jurisdiction is toUs persuasive in interpreting provisions of our own Insurance Code. Inaddition, the application of the adopted statute should correspond infundamental points with the application in its country of origin x x x.

x x x x

Likewise, it is settled in that jurisdiction that the (sic)notwithstanding recitals in the Loan Receipt that the money was intended as aloan does not detract from its real character as payment of claim, thus:

―The receipt of money by the insured employers from a surety company forlosses on account of forgery of drafts by an employee where no provision orrepayment of the money was made except upon condition that it be recoveredfrom other parties and neither interest nor security for the asserted debts wasprovided for, the money constituted the payment of a liability and not a mereloan, notwithstanding recitals in the written receipt that the money wasintended as a mere loan.‖

What is clear from the wordings of the so-called ―Loan and Trust Receipt Agreement‖ is that appellant is obligated to hand over to appellee ―whateverrecovery (Trans Asia) may make and deliver to (Prudential) all documentsnecessary to prove its interest in the said property.‖ For all intents andpurposes therefore, the money receipted is payment under the policy, withPrudential having the right of subrogation to whatever net recovery Trans-Asiamay obtain from third parties resulting from the fire. In the law on insurance,subrogation is an equitable assignment to the insurer of all remedies which theinsured may have against third person whose negligence or wrongful actcaused the loss covered by the insurance policy, which is created as the legaleffect of payment by the insurer as an assignee in equity. The loss in the firstinstance is that of the insured but after reimbursement or compensation, itbecomes the loss of the insurer. It has been referred to as the doctrine ofsubstitution and rests on the principle that substantial justice should beattained regardless of form, that is, its basis is the doing of complete,essential, and perfect justice between all the parties without regard toform.[31]

We agree. Notwithstanding its designation, the tenor of the ―Loan and TrustReceipt‖ evidences that the real nature of the transaction between the par tieswas that the amount of P3,000,000.00 was not intended as a loan wherebyTRANS-ASIA is obligated to pay PRUDENTIAL, but rather, the same was apartial payment or an advance on the policy of the claims due to TRANS-ASIA.

First, the amount of P3,000,000.00 constitutes an advance payment toTRANS- ASIA by PRUDENTIAL, subrogating the former to the extent of ―anynet recovery made by TRANS ASIA SHIPPING CORP., from any person orpersons, corporation or corporations, or other parties, on account of loss byany casualty for which they may be liable, occasioned by the 25 October 1993:Fire on Board.‖[32]

Second, we find that per the ―Loan and Trust Receipt,‖ even as TRANS-ASIAagreed to ―promptly prosecute suit against such persons, corporation orcorporations through whose negligence the aforesaid loss was caused or whomay otherwise be responsible therefore, with all due diligence‖ in its name, theprosecution of the claims against such third persons are to be carried on ―atthe expense of and under the exclusive direction and control of PRUDENTIALGUARANTEE AND ASSURANCE INC.‖[33] The clear import of the phrase―at the expense of and under the exclusive direction and control‖ as used inthe ―Loan and Trust Receipt‖ grants solely to PRUDENTIAL the power toprosecute, even as the same is carried in the name of TRANS-ASIA, therebymaking TRANS-ASIA merely an agent of PRUDENTIAL, the principal, in theprosecution of the suit against parties who may have occasioned the loss.

Third, per the subject ―Loan and Trust Receipt,‖ the obligation ofTRANS-ASIA to repay PRUDENTIAL is highly speculative and contingent,i.e., only in the event and to the extent that any net recovery is made byTRANS-ASIA from any person on account of loss occasioned by the fire of 25October 1993. The transaction, therefore, was made to benefit

TRANS-ASIA, such that, if no recovery from third parties is made,PRUDENTIAL cannot be repaid the amount. Verily, we do not think that thisis constitutive of a loan.[34] The liberality in the tenor of the ―Loan and TrustReceipt‖ in favor of TRANS-ASIA leads to the conclusion that the amount of

P3,000,000.00 was a form of an advance payment on TRANS- ASIA‘s claim onMH93/1353.

III.

A. PRUDENTIAL is directed to pay TRANS-ASIA the amount ofP8,395,072.26, representing the balance of the loss suffered by TRANS-ASIAand covered by Marine Policy No. MH93/1363.

Our foregoing discussion supports the conclusion that TRANS-ASIA is entitledto the unpaid claims covered by Marine Policy No. MH93/1363, or a totalamount of P8,395,072.26.

B. Likewise, PRUDENTIAL is directed to pay TRANS-ASIA, damages inthe form of attorney‘s fees equivalent to 10% of P8,395,072.26.

The Court of Appeals denied the grant of attorney‘s fees. It held thatattorney‘s fees cannot be awarded absent a showing of bad faith on the part ofPRUDENTIAL in rejecting TRANS- ASIA‘s claim, notwithstanding that therejection was erroneous. According to the Court of Appeals, attorney‘s feescan be awarded only in the cases enumerated in Article 2208 of the Civil Code

which finds no application in the instant case.

We disagree. Sec. 244 of the Insurance Code grants damages consisting ofattorney‘s fees and other expenses incurred by the insured after a finding bythe Insurance Commissioner or the Court, as the case may be, of anunreasonable denial or withholding of the payment of the claims due.Moreover, the law imposes an interest of twice the ceiling prescribed by theMonetary Board on the amount of the claim due the insured from the datefollowing the time prescribed in Section 242[35] or in Section 243,[36] as thecase may be, until the claim is fully satisfied. Finally, Section 244 considersthe failure to pay the claims within the time prescribed in Sections 242 or 243,when applicable, as prima facie evidence of unreasonable delay in payment.

To the mind of this Court, Section 244 does not require a showing of bad faithin order that attorney‘s fees be granted. As earlier stated, under Section 244,a prima facie evidence of unreasonable delay in payment of the claim iscreated by failure of the insurer to pay the claim within the time fixed in bothSections 242 and 243 of the Insurance Code. As established in Section 244,by reason of the delay and the consequent filing of the suit by the insured, theinsurers shall be adjudged to pay damages which shall consist of attorney‘sfees and other expenses incurred by the insured.[37]

Section 244 reads:

In case of any litigation for the enforcement of any policy or contract ofinsurance, it shall be the duty of the Commissioner or the Court, as the casemay be, to make a finding as to whether the payment of the claim of theinsured has been unreasonably denied or withheld; and in the affirmativecase, the insurance company shall be adjudged to pay damages which shallconsist of attorney‘s fees and other expenses incurred by the insured personby reason of such unreasonable denial or withholding of payment plus interestof twice the ceiling prescribed by the Monetary Board of the amount of theclaim due the insured, from the date following the time prescribed in sectiontwo hundred forty-two or in section two hundred forty-three, as the case maybe, until the claim is fully satisfied; Provided, That the failure to pay any suchclaim within the time prescribed in said sections shall be considered primafacie evidence of unreasonable delay in payment.

Sections 243 and 244 of the Insurance Code apply when the court finds anunreasonable delay or refusal in the payment of the insurance claims.

In the case at bar, the facts as found by the Court of Appeals, and confirmedby the records show that there was an unreasonable delay by PRUDENTIALin the payment of the unpaid balance of P8,395,072.26 to TRANS-ASIA.On 26 October 1993, a day after the occurrence of the fire in ―M/V Asia Korea‖,TRANS-ASIA filed its notice of claim. On 13 August 1996, the adjuster,Richards Hogg International (Phils.), Inc., completed its survey reportrecommending the amount of P11,395,072.26 as the total indemnity due toTRANS-ASIA.[38] On 21 April 1997, PRUDENTIAL, in a letter[39] addressedto TRANS- ASIA denied the latter‘s claim for the amount of P8,395,072.26representing the balance of the total indemnity. On 21 July 1997,PRUDENTIAL sent a second letter[40] to TRANS-ASIA seeking a return of theamount of P3,000,000.00. On 13 August 1997, TRANS-ASIA wasconstrained to file a complaint for sum of money against PRUDENTIALpraying, inter alia, for the sum of P8,395,072.26 representing the balance ofthe proceeds of the insurance claim.

As can be gleaned from the foregoing, there was an unreasonable

delay on the part of PRUDENTIAL to pay TRANS-ASIA, as in fact, it refutedthe latter‘s right to the insurance claims, from the time proof of loss was shownand the ascertainment of the loss was made by the insurance adjuster.

8/12/2019 Insurance Meeting 6 Cases

http://slidepdf.com/reader/full/insurance-meeting-6-cases 27/28

MEETING 6 CASES - INSURANCE

27

Evidently, PRUDENTIAL‘s unreasonable delay in satisfying TRANS- ASIA‘sunpaid claims compelled the latter to file a suit for collection.

Succinctly, an award equivalent to ten percent (10%) of the unpaidproceeds of the policy as attorney‘s fees to TRANS-ASIA is reasonable underthe circumstances, or otherwise stated, ten percent (10%) of P8,395,072.26.In the case of Cathay Insurance, Co., Inc. v. Court of Appeals,[41] where afinding of an unreasonable delay under Section 244 of the Insurance Code

was made by this Court, we grant an award of attorney‘s fees equivalent to tenpercent (10%) of the total proceeds. We find no reason to deviate from this judicial precedent in the case at bar.

C. Further, the aggregate amount (P8,395,072.26 plus 10% thereof asattorney‘s fees) shall be imposed double interest in accordance with Section244 of the Insurance Code.

Section 244 of the Insurance Code is categorical in imposing aninterest twice the ceiling prescribed by the Monetary Board due the insured,from the date following the time prescribed in Section 242 or in Section 243, asthe case may be, until the claim is fully satisfied. In the case at bar, we findSection 243 to be applicable as what is involved herein is a marine insurance,clearly, a policy other than life insurance.

Section 243 is hereunder reproduced:

SEC. 243. The amount of any loss or damage for which aninsurer may be liable, under any policy other than life insurance policy, shall bepaid within thirty days after proof of loss is received by the insurer andascertainment of the loss or damage is made either by agreement betweenthe insured and the insurer or by arbitration; but if such ascertainment is nothad or made within sixty days after such receipt by the insurer of the proof ofloss, then the loss or damage shall be paid within ninety days after suchreceipt. Refusal or failure to pay the loss or damage within the timeprescribed herein will entitle the assured to collect interest on the proceeds ofthe policy for the duration of the delay at the rate of twice the ceiling prescribedby the Monetary Board, unless such failure or refusal to pay is based on theground that the claim is fraudulent.

As specified, the assured is entitled to interest on the proceeds for the durationof the delay at the rate of twice the ceiling prescribed by the Monetary Boardexcept when the failure or refusal of the insurer to pay was founded on theground that the claim is fraudulent.

D. The term ―double interest‖ as used in the Decision of the Court of Appeals must be interpreted to mean 24% per annum.

PRUDENTIAL assails the award of interest, granted by the Court of Appeals,in favor of TRANS-ASIA in the assailed Decision of 6 November 2001. It isPRUDENTIAL‘s stance that the award is extortionate and grosslyunsconscionable. In support thereto, PRUDENTIAL makes a reference toTRANS- ASIA‘s prayer in the Complaint filed with the court a quo wherein thelatter sought, ―interest double the prevailing rate of interest of 21% per annumnow obtaining in the banking business or plus 42% per annum pursuant to Article 243 of the Insurance Code x x x.‖[42]

The contention fails to persuade. It is settled that an award of double interestis lawful and justified under Sections 243 and 244 of the Insurance Code.[43]In Finman General Assurance Corporation v. Court of Appeals,[44] this Courtheld that the payment of 24% interest per annum is authorized by theInsurance Code.[45] There is no gainsaying that the term ―double interest‖ asused in Sections 243 and 244 can only be interpreted to mean twice 12% perannum or 24% per annum interest, thus:

The term ―ceiling prescribed by the Monetary Board‖ means thelegal rate of interest of twelve per centum per annum (12%) as prescribed bythe Monetary Board in C.B. Circular No. 416, pursuant to P.D. No. 116,amending the Usury Law; so that when Sections 242, 243 and 244 of theInsurance Code provide that the insurer shall be liable to pay interest ―twicethe ceiling prescribed by the Monetary Board‖, it means twice 12% per annumor 24% per annum interest on the proceeds of the insurance.[46]

E. The payment of double interest should be counted from 13 September1996.

The Court of Appeals, in imposing double interest for the duration of the delayof the payment of the unpaid balance due TRANS-ASIA, computed the samefrom 13 August 1996 until such time when the amount is fully paid. Although

not raised by the parties, we find the computation of the duration of the delaymade by the appellate court to be patently erroneous.

To be sure, Section 243 imposes interest on the proceeds of the policy for theduration of the delay at the rate of twice the ceiling prescribed by the MonetaryBoard. Significantly, Section 243 mandates the payment of any loss ordamage for which an insurer may be liable, under any policy other than lifeinsurance policy, within thirty days after proof of loss is received by the insurerand ascertainment of the loss or damage is made either by agreementbetween the insured and the insurer or by arbitration. It is clear that underSection 243, the insurer has until the 30th day after proof of loss and

ascertainment of the loss or damage to pay its liability under the insurance,and only after such time can the insurer be held to be in delay, therebynecessitating the imposition of double interest.

In the case at bar, it was not disputed that the survey report on theascertainment of the loss was completed by the adjuster, Richard HoggsInternational (Phils.), Inc. on 13 August 1996. PRUDENTIAL had thirty daysfrom 13 August 1996 within which to pay its liability to TRANS-ASIA under theinsurance policy, or until 13 September 1996. Therefore, the double interestcan begin to run from 13 September 1996 only.

IV.

A. An interest of 12% per annum is similarly imposed on the TOTALamount of liability adjudged in section III herein, computed from the time of

finality of judgment until the full satisfaction thereof in conformity with thisCourt‘s ruling in Eastern Shipping Lines, Inc. v. Court of Appeals.

This Court in Eastern Shipping Lines, Inc. v. Court of Appeals,[47] inscribedthe rule of thumb[48] in the application of interest to be imposed onobligations, regardless of their source. Eastern emphasized beyond cavilthat when the judgment of the court awarding a sum of money becomes finaland executory, the rate of legal interest, regardless of whether the obligationinvolves a loan or forbearance of money, shall be 12% per annum from suchfinality until its satisfaction, this interim period being deemed to be by then anequivalent to a forbearance[49] of credit.

We find application of the rule in the case at bar proper, thus, a rate of 12% perannum from the finality of judgment until the full satisfaction thereof must beimposed on the total amount of liability adjudged to PRUDENTIAL. It is clearthat the interim period from the finality of judgment until the satisfaction of thesame is deemed equivalent to a forbearance of credit, hence, the imposition ofthe aforesaid interest.

Fallo

WHEREFORE, the Petition in G.R. No. 151890 is DENIED.However, the Petition in G.R. No. 151991 is GRANTED, thus, we award thegrant of attorney‘s fees and make a clarification that the term ―double interest‖as used in the 6 November 2001 Decision of the Court of Appeals in CA GRCV No. 68278 should be construed to mean interest at the rate of 24% perannum, with a further clarification, that the same should be computed from 13September 1996 until fully paid. The Decision and Resolution of the Court of Appeals, in CA-G.R. CV No. 68278, dated 6 November 2001 and 29 January2002, respectively, are, thus, MODIFIED in the following manner, to wit:

1. PRUDENTIAL is DIRECTED to PAY TRANS-ASIA theamount of P8,395,072.26, representing the balance of the loss suffered byTRANS-ASIA and covered by Marine Policy No. MH93/1363;

2. PRUDENTIAL is DIRECTED further to PAY TRANS-ASIAdamages in the form of attorney‘s fees equivalent to 10% of the amount ofP8,395,072.26;

3. The aggregate amount (P8,395,072.26 plus 10% thereofas attorney‘s fees) shall be imposed double interest at the rate of 24% perannum to be computed from 13 September 1996 until fully paid; and

4. An interest of 12% per annum is similarly imposed on the TOTALamount of liability adjudged as abovestated in paragraphs (1), (2), and (3)herein, computed from the time of finality of judgment until the full satisfactionthereof.

No costs.

SO ORDERED.

8/12/2019 Insurance Meeting 6 Cases

http://slidepdf.com/reader/full/insurance-meeting-6-cases 28/28

MEETING 6 CASES - INSURANCE

DIGEST

Young v Midland March 31, 1915 G.R. No. L-9370J. Johnson

Facts:

Young owned a candy and fruit store in Manila. Midland issued a policy for thepayment of a premium of P60. The indemnity was 3,000 if the place wasdestroyed by fire. One clause claimed:―Waranty B. – It is hereby declared and agreed that during the pendency of

this policy no hazardous goods stored or kept for sale, and no hazardous tradeor process be carried on, in the building to which this insurance applies, or inany building connected therewith.‖ Young then placed three boxes of fireworks. The plaintiff intended to use themfor Chinese New Year, but the authorities prohibited the use. The bodega wasdestroyed by fire.Both of the parties agree that said fireworks come within the phrase―hazardous goods,‖ mentioned in said ―warranty B‖ of the policy. But it wasfound out that the fireworks were found in a part of the building not destroyedby the fire, and that they in no way contributed to the fire.

Issue: Whether or not the placing of said fireworks in the building insured is aviolation of the terms of the contract of insurance and especially of ―warrantyB‖

Held: Yes. Petition dismissed.

Ratio:Both the plaintiff and defendant agree that if they were ―hazardous goods,‖and if they were ―stored,‖ then the act of the plaintiff was a violation of theterms of the contract of insurance and the defendant was justified inrepudiating its liability.This leads us to a consideration of the meaning of the accord ―stored‖ as usedin said ―warranty B.‖ Whether a particular article is ―stored‖ or not must, insome degree, depend upon the intention of the parties. Nearly all of the casescited by the lower court are cases where the article was being put to somereasonable and actual use, which might easily have been permitted by theterms of the policy, and within the intention of the parties, and excepted fromthe operation of the warranty, like the present.The author of the Century Dictionary defines the world ―store‖ to be a depositin a store or warehouse for preservation or safe keeping; o place in awarehouse or other place of deposit for safe keeping.Said definitions, of course, do not include a deposit in a store, in smallquantities, for daily use. ―Daily use‖ precludes the idea of a deposit forpreservation or safe keeping, as well as a deposit for future consumption, orsafe keeping.

The plaintiff makes no claim that he deposited them there with any other ideathan ―for future use‖ – for future consumption. It seems clear to us that the―hazardous goods‖ in question were ―stored‖ in the bodega, as that word isgenerally defined. That being true, suppose the defendant had made anexamination of the premises, even in the absence of a fire, and had found the―hazardous goods‖ there, under the conditions above described, would it nothave been justified, then and there, in declaring the policy null and of no effectby reason of a violation of its terms?The appellant argues, however, that in view of the fact that the ―storing‖ of thefireworks on the premises of the insured did not contribute in any way to thedamage occasioned by the fire, he should be permitted to recover – that the―storing‖ of the ―hazardous goods‖ in no way caused injury to the defendantcompany. The ―storing‖ was a violation of the terms of the contract by virtue ofthe provisions of the policy itself, terminated the contractual relations.The plaintiff paid a premium based upon the risk at the time the policy wasissued. Certainly it cannot be denied that the placing of the firecrackers in thebuilding insured increased the risk. The plaintiff had not paid a premium basedupon the increased risk, neither had the defendant issued a policy upon thetheory of a different risk. The plaintiff was enjoying, if his contention may beallowed may be allowed, the benefits of an insurance policy upon one risk,whereas, as a matter of fact, it was issued upon an entirely different risk. Thedefendant had neither been paid nor had issues a policy to cover theincreased risk. An increase of risk which is substantial and which is continuedfor a considerable period of time, is a direct and certain injury to the insurer,and changes the basis upon which the contract of insurance rests.


Recommended